Dharma is Independent of Culture
Ngakma Shardröl Du-nyam Wangmo interviews Ngak'chang Rinpoche and Khandro Déchen
on the published views of the Carreon's.
I read the text of Another View on Whether Tibetan Buddhism is Working in the West, by Tara Carreon and her husband, to Ngak'chang Rinpoche and Khandro Déchen and requested their responses. I mentioned specific quotes on which I particularly wanted comment and clarification in terms of Vajrayana; however, Ngak'chang Rinpoche and Khandro Déchen declined to address some of the quotes from the article since they considered them to be too pointless or trivial even to refute.

Ngakma Shardröl: Rinpoche, Khandro Déchen, I would like to ask you if you would comment on this article. I know you don't like to comment about specific people - but could you perhaps respond to some aspects of what's being presented here in order that I can get a better understanding of how people arrive at their own forms of confusion?
Khandro Déchen: Most of those who make pronouncements on this matter know who the representatives are on either side - but, if we may restrict ourselves to the concepts presented, we have no great objection to the sources of the comments being apparent.
Rinpoche: To a degree, it is no longer possible to speak without reference to individuals, especially when they themselves are commenting on other writers. We skimmed the interview with Alan Wallace in Tricycle, and... we can see why some people would take objection to it.
Firstly we would say that whilst there is certainly room to criticise Western people's approach to Dharma - there should also be room to look at the many fine practitioners of Vajrayana in the West. There are many, but they tend to be low key in terms of making public statements. We have a number of excellent students who have been with us for almost twenty years, and we have seen no signs of fickleness in them. From our association with various Tibetan Lamas in the West, we can say quite definitely that they have a high regard for their committed Western disciples. Dung-sé Thrin-lé Norbu Rinpoche in particular has been highly supportive of Western Lamas in connection with Tharchin Rinpoche and Chag'düd Tulku Rinpoche. As far as we are concerned, Vajrayana is working extremely well in certain places in the West. One only has to look to Tsogyelgar in Ann Arbor, Michigan to see the magnificent way in which Traktung Rinpoche and A'dzom Rinpoche are integrating Vajrayana into a Western setting. There are certainly problems - and some Western students are exactly as infantile and fickle as described in Alan Wallace's interview. People are people wherever they are, and it is not useful to make hard-line generalisations. But one of the major dangers with regard to Tibetan Buddhism in the West is the 'new age' appropriation of its teachings and the falsification of its functions by people such as the author under discussion.
Secondly we must say that we were concerned with regard to Allan Wallace's commentary on the way in which Vajrayana was introduced to the west by Tibetan Lamas. The picture he gives would appear to lean heavily in the direction of an institutionalised approach in which monastic training - or something approaching it - is deemed indispensable.
Q: From reading the interview I noticed that he was once a monk. It seems that people like Allan Wallace and Stephen Batchelor and some of the others who make crucial comment on Tibetan Lamas are ex-monks. At the Naropa Conference and in other places it seems to be some kind of qualification to have once been a monk of a nun. I don't really understand that as being a qualification.
R: Well, quite - but I would rather not comment on this. I am more concerned with the way in which Alan Wallace speaks as if he possessed the grand overview. It is not that I entirely disagree with what he describes as the desperate nature of the introduction of Vajrayana - it is simply that it is a partial account and makes no reference to the fact that many thoroughgoing programmes of study and practice were established alongside the fleeting visits of high Lamas. Many people who met these 'briefly visiting' Lamas went out to India and Nepal to study with them - and to a certain extent this replicates the situation as it was in Tibet. Personally - I see no great room for complaint in terms of the manner in which Vajrayana was introduced, nor do I think that anyone has sufficient historical perspective at this point in time to make any useful analysis. If I may make a final observation on the Alan Wallace interview, it would be a fundamental concern with the idea that the East and the West are so very different from each other that we all need to have ongoing conferences and discussions about it. I feel that this makes much too much of the issue - and plays into the hands of those who seem intent on leading Buddhist up a psychotherapeutic backwater.
KD: To speak of cultures as being so problematically different is to neglect the Buddhist view that we are all busy creating samara from the ground of being. There may be a Tibetan-style samsara, and an English-style samsara and an American-style samsara - but they have one thing in common which is addressed by Buddhist practice. I have the feeling that it is not useful to approach Buddhist practice by concentrating on our special status in being different.
R: There are cultural differences, but these differences seem to be more concerned with linguistics - and very few people seem interested in addressing that point. We try to write and teach in contemporary language - but we appreciate that we could be criticised for this in the same way in which we have criticised placing too much importance on cultural difference. However that may be, we took our lead from Chögyam Trungpa Rinpoche's writings - and so we feel that this tangent has some basis in necessity beyond our own need to be different.
Before you begin to ask us about your particular chosen quotes from this article by Mrs. Carreon, I would like to comment on one statement which piqued my interest. The author says, 'Only people who don't have a reputation or position to protect can speak the truth. That precludes people with vested interests in the existing system from saying anything meaningful...' or words to that effect. Now this is an excellent point at which to begin because Ngakpa Chögyam does not have a good reputation or any position at all, as far as Tibetan Buddhism is concerned - especially not in the West. Ngakpa Chögyam is regarded with suspicion and resentment in many quarters, and so I have nothing at all to lose - whatever I say. Khandro Déchen and I are pariahs with little welcome in most places. There is one notable Buddhist publisher and distributor who will not even list our books - so no one can say we have any position to defend.
Khandro Déchen and I function as independent Lamas - that is to say, independent of any directly imposed hierarchic direction. This - although common in the Nyingma tradition - is maybe not a common situation for Western Lamas. Our own Lamas, Kyabjé Künzang Dorje Rinpoche and Jomo Sam'phel, are not well-known in the West and have left us to evolve our own situation with students. As Western Lamas, we are aware of certain cultural issues which effect the integration of Vajrayana into the West - but it is perfectly clear to us that Dharma is independent of culture. Dharma may be presented through cultural forms to a certain degree - but these are only the container of Dharma. All the great Lamas we have known have stood outside culture as teachers of Dharma. Kyabjé Künzang Dorje Rinpoche in particular, of all the Lamas I have known, stressed continually that I must understand the principle and function of practice, and this has helped me not only in understanding the nine yanas - but also in understanding the value of other religions and systems of human evolution. Whatever we have to say on this subject is said without fear of praise or blame. If we receive blame - we are well used to it, and if we receive praise - it will be something of a surprise. So - to your questions.
Q: This, as you know, was a website response to an interview which appeared in Tricycle. Tara Carreon - the author - is a disgruntled ex-student of a Tibetan Lama. She seems to want to justify her disgruntlement by discrediting Tibetan Buddhism and Tibetan Lamas so that no blame can be attached to her, and she can view herself as a victim. She makes quite a few statements, some of which make unhelpful generalisations and others which seem to be the result of personal prejudice. For example, she begins by saying that 'American Tibetan Buddhists have made the understandable decision to adopt traditional Tibetan Buddhist beliefs because they seem authoritative and reliable. This decision has been a mistake.'
R: An understandable decision? That is strange... Why would this be expected? It could only be understood if the people of whom she is writing were alienated from their own culture. Maybe this is true of the hippie generation. Although I am of that generation, I was never exactly a card-carrying hippie, and so my own interest in Vajrayana never excluded the richness of my own culture. I never really related well to the 'born-again-Tibetans' who went to India and Nepal - and to be honest they didn't relate well to me either.
KD: I was too young to be a hippy and a little too conservative to have had any interest in the 'new age' - so rejection of Western culture passed me by. My interest in Vajrayana arose accidentally through meeting Rinpoche in social circumstances rather than through having an interest in Eastern religion. I was not a 'spiritual seeker' and had never considered the need to find the 'inner peace and self-understanding' for which Tara Carreon was searching. Why look to an Eastern religion for 'inner peace and self-understanding' when the religions of your own culture already contain teachings which address this? The Quakers and the Unitarians have the egalitarianism she wants, and the silent sitting can be found in the Julian Movement within Christianity.
R: Both Christianity and Judæism have esoteric teachings too - so why not look a little deeper into Western culture before approaching teachers of Buddhism with regard to needs that might not be met according to your preset notions of what you expect? I never wanted 'inner peace and self-understanding' either - so maybe that is why Khandro Déchen and I have not been disappointed.
Q: It sounds as though she took a kind of retail approach to her spiritual life - like she had this goal of inner peace and self-understanding, which is not actually even a Buddhist concept as I have ever heard it expressed, and then she felt cheated when the product she bought didn't deliver what she wanted.
R: That is certainly the impression she creates. I would also say that finding your home in any religion is the surest way of being at ease. Lack of 'inner peace' is caused by feeling driven and embattled. It ought to be possible simply to be happy to be 'a practitioner without ambition'. I have never judged Vajrayana because of my own lack of capacity. I am quite content to be a loser who has failed to understand himself. If I understood myself, there would then need to be a self to understand, and I think that I'm better off without that. I know that I'm greedy, irritable, obsessive, quirky, and occluded. These are the five obstacles we all have to one degree or another - so self-knowledge is not really the issue. As to the nature of Mind - that is empty, and so there is nothing to know or not know. There is only the immediate experience. There is only the transmission I have received, and continually returning to that.
Q: It seems as though she thinks that being a Buddhist means accepting a belief system, on someone else's authority, when I thought the whole point was to practise in order to experience things for yourself.
R: Yes. One is given the structure and the means of verifying that structure. One has to take the structure on faith - through inspiration - in order to gain personal realisation of the structure - and, discover that it is not a structure. I suppose in some sense what she says is true - many people do seem to have accepted Buddhism as if it were a belief system - but that is reflective more of those persons themselves than of Buddhism. Buddhism is not a belief system. Buddhism does advocate faith, but not blind faith. Buddhism advocates reasonable faith - that is to say, faith based on experience. Faith must be based on study and practice, so that reasonable faith gradually becomes direct knowledge. That must be clearly understood. It should also be understood that faith is required in every aspect of life - you even need faith to bite into a pastrami baguette. How do you know it's not poisonous? How do you know you'll like it? Someone you like and respect says: "This is a good place to buy a pastrami baguette." And so you proceed on faith to buy one and take the first bite out of it.
There is never any certainty about any act or a course of action until we enter into it. It is only the foolhardy who plunge into projects without intelligent research. It is also dubious to accept information because it seems authoritative and reliable. Door-to-door sales representatives turn up every day seeming authoritative and reliable. I have had such people tell me that their plastic window frames are superior to my old wooden sash windows - but I don't believe a word of it. We need more than something 'seeming' to be 'authoritative and reliable' before we buy it. I have actually never met a Lama who has not continually underlined the need for study and practice - and if one studies and practises then one develops intelligent faith in the lineage one follows.
KD: We say this to students all the time - that they should continually compare their study and practice in order to make sure that they are confident in how they proceed. So to proceed without experience born out of study and practice is a mistake.
Q: So basically she's making a tautological statement - she's saying that people made a mistake in making a mistake.
KD: That's a good way to put it - but more that that, you have to ask why people made that mistake in the first place and why they persevered in their mistake. There must have been some kind of reward for persevering with mistaken beliefs.
Q: Well, the writer says that she 'received the entire transmission of Nyingma teachings from beginning to end, including Trekchod and Togal teachings.'
R: Then that is likely to be the answer. Her perseverance would appear to have lasted until the final Dharma commodity had been acquired.
Q: The retail approach again.
R: Quite. Does she say that she did more than receive these transmissions? Does she make any comment on what she gained from practice?
Q: Quite the reverse - she says that she was unable 'to see any "truly" enlightened developments in [her] psyche' after 22 years of effort.
R: That seems an extremely long time to persevere without reward... It strikes me that there must have been other rewards which she does not discuss - because no one proceeds with anything for which there is no reward. That is not psychologically feasible. Does she mention anything at all?
Q: Well, she says that she was 'extremely devout' and 'did her practice compulsively'. She seems to have been involved in fund-raising and building gompas and transcribing teachings. She says she edited a book of teachings and held practices at her house to which other people came, so I guess she must have had some kind of status in the group.
R: This 'extremely devout'... now, there's a phrase... I wonder what that means, and on what her devoutness was based? Then the 'compulsive' nature of her practice - that is also strange. It seems like a rationalisation in retrospect. If one was genuinely devout, then compulsivity would not come into it. One is either devout or compulsive - the two terms are mutually contradictory. One can only say that one was compulsive in hindsight - and then at the same time one would have to say that one's devoutness was equally pathological. One cannot make the two statements at once. I am sure that to some degree the writer has looked at herself and that she has come to what she feels are honest conclusions about her experience - but remodelling one's history to suit one's present mindset is a common self-protective phenomenon and often carried out almost unconsciously. It is not possible for me to say how honest or dishonest the writer may be - all I can say is that there are some serious experiential and logical discrepancies in her account.
KD: I am simply amazed that anyone would put so much effort into something which then seemed hollow. Surely there must have been some more gradual process involved unless she was in severe denial - and then, I would tend to feel that the process of denial is now merely operating in reverse: she is unable to see any of the benefits she received. So what went wrong?
Q: She doesn't really say. She doesn't actually give any kind of adequate explanation of what it was that caused her to change her mind so radically about something that had been so important to her for 22 years. It's the same in the autobiographical material she and her husband present on their website. They give a history of the typical 1960s seeking in which they meet there Lama - and then they skip 22 years to the point when they leave for no accountable reason apart from the accusations she makes against Tibetans. She says that 'Tibetans suffer from ethnocentricity and cultural arrogance that blinds them to the virtues of Western culture and predisposes them to favour all things Tibetan.'
KD: This of course could be said of any nationality. I am sure that some Tibetans suffer from this as well as Americans or the British. There are French people who would like to see all English words expunged from their language.
R: This is nothing unique to Tibetans - but more to the point, we are not really discussing culture when we discuss Vajrayana. Vajrayana is not Tibetan. Vajrayana is not English or American either. I am not really concerned with cultural differences or whether one culture is better than another. But actually I've heard many Tibetan Lamas be highly appreciative of Western culture - Chögyam Trungpa Rinpoche in particular loved Handel's Messiah and was notably keen on Pavarotti. I wouldn't say that there are no Tibetan cultural supremacists - but I can say that I have never met such a person. On the other hand, I have met plenty of Western people who disparage Western culture - in fact it was a highly popular position to adopt in the 1960's and early 1970's. I once had a sitar myself. [pause] You know . . . It's often Western people who gives their Tibetan teachers a picture of the West. Many Western people speak of their own culture in a disparaging way and so it is not entirely surprising if some Lamas come to accept their students' descriptions.
Q: Yes - at the conference of Western Buddhist teachers in McLeod Ganj Yvonne Rand told the Dala'i Lama quite a story about the horrors of rock music, how it was terribly loud and disturbing and that people became mindlessly devoted slaves to the musicians. There were other comments like that too - all negative about the West.
R: Yes [laughs] I remembered a certain degree of derision when I responded that I liked cowboy movies, in answer to a 'round-the-table' question on favourite films.
Q: It's almost as if a lot of Western Buddhists are embarrassed about being Westerners. They long to leave their rapacious materialistic culture behind and find something simple and pure and foreign. Maybe the author of the article was one of these people and she's now swinging back all the way in the other direction, rejecting everything foreign and going back to her American roots. That leads on to the next quote: she finds 'American students far too willing to abandon the advantages of our intellectual training and democratic culture in favour of medieval concepts...'
R: I would tend to agree with that in part. Among some Westerner people there would seem to be a romantic intrigue with the 'ancient world' which was born out of the idea that the 'modern world' was corrupt and that science had in some way 'gone too far'. I think that Western people often do abandon their intelligence in favour of a wide variety of 'new age' and or anachronistic pastimes. But this is not actually possible with Buddhism. Buddhism demands the most rigorous employment of intelligence that I could imagine. I find rather that most people with a grudge against Vajrayana are actually severely lacking in intellectual discipline. The arguments they put forward have been - as far as Khandro Déchen and I have seen - logically deficient to a surprising degree. So - how does she suggest we proceed?
Q: She says that we should 'abandon Tibetan cultural belief systems, stripping Buddhism to its core values...' [interrupted]
R: So she is still interested in Buddhism then? That is singular... I wonder what basis she has for continuing with Buddhism after 22 years of admittedly fruitless practice? Sorry to interrupt - pray continue.
Q: '... stripping Buddhism to its core values of straightforward inquiry and insight into emptiness, supplementing these values with Western virtues of optimism, creativity, and scientific method. Such a change in spiritual approach can lead to real cause for optimism and freedom from outmoded notions that merely lead to psychological subjugation.' It's interesting that she sees the scientific method as antithetical to Buddhism. It seems to me that in practice Buddhism is actually quite scientific - in terms of each practitioner verifying what they have been taught by practising it themselves and achieving or not achieving results.
R: Yes. She is evidently unhappy - and I am sorry for that, it is sad to see anyone in such obvious pain, but if she has not seen that Dzogchen Trek gÇod can already be said to strip Buddhism to the core - then she never understood the teachings she received. I find it improbable that anyone who ever studied Dzogchen - let alone practised it - could write in such a way. It makes no sense to me whatsoever.
Q: It seems that Surya Das agrees with her, and he advertises himself as a teacher of Dzogchen. Part of the same internet material contains a letter from Surya Das in which he says: 'Thank you for speaking out clearly, honestly and well. You should publish this somewhere.'
KD: That does not surprise us so much anymore.
R: I don't like to be censorious with regard to individuals . . . but it would seem that Surya Das could not be said to be lacking in having something of a vested interest in the publication of such critical material . . .
Q: Yes - Surya Das has positioned himself at the forefront of a movement which seems to be out to corner the market on Buddhism in the West. It looks like some sort of takeover bid by those who failed to get what they wanted to get from Buddhism and who are now attempting to become the new gurus of dissatisfaction.
R: This material so far, has all the trappings of victim-as-hero or victim-as-heroine. All you need in certain circles is to cry pain in a passably articulate manner - and you have an immediate audience. I am not saying that no one was ever abused - I have emphasised this before - but we do live in a society where you can sue McDonald's because you gripped a Styrofoam cup of coffee between your thighs and scalded yourself. That is not exactly the most brilliant and worthwhile aspect of Western culture. If she's so keen on Western culture - let her emulate the real heroes and heroines of our culture. Let her emulate Florence Nightingale, Mother Theresa, Joan of Arc, Boadicea, or Annie Oakley. Why give credence and respect to someone who allowed herself to be psychologically subjugated for 22 years? If she had 'shed the trappings of orthodoxy', gained liberation and come back with her story - I might buy it as an intriguing read. But what is being offered here is merely a thesis based on failure and subsequent psychological processing.
It is not that I have no sympathy for suffering - Khandro Déchen and I have often given time to people who have been badly treated in the spiritual context - but on all but a few occasions our time was not particularly well spent. Our experience has mainly been that those who like to complain are merely seeking a new audience from whom they can squeeze personal value as a result of their suffering.
Q: It seems like her main point is that she feels she wasted 22 years of her life on something she got nothing out of and now, rather than look at herself and why she did that, and what were the hidden benefits that kept her there, she's decided that there can be nothing of value in Tibetan Buddhism for anybody. Somehow she's not content just to say it wasn't right for her, she wants to discredit the whole thing in order to avoid having to look at herself... [interrupted]
KD: Before we go on, I would like to say something about her idea of optimism being a Western virtue. I think it's actually rare to find any work of modern Western literature that has a happy ending. Jane Austen wrote happy endings but that no longer seems to be the trend. A great deal of Western art seems to revel in suffering - so I find her statement quite disconnected from Western culture. Tibetans, on the other hand, have always seemed to me to be surprisingly happy and positive in spite of their condition as refugees.
R: Yes. When I was first in India in 1970 I was inordinately impressed with the Tibetan temperament in general - and I lived amongst them for almost seven years. To me, the Bodhisattva vow is amongst the most optimistic, positive, and creative drives that could be imagined - and that seemed to be the vector of sheer chutzpah that I found so often amongst the yogis and yoginis I have known. It also exists amongst the ordinary Tibetan people - whereas in the West it's often the so-called intelligentsia who make a virtue of depression. Take Woody Allen for example - some people find him funny...
KD: I'm afraid I don't find depression and despair funny. And I'm not saying this because I don't appreciate Western humour which looks at the negative aspects of being human - I find John Cleese hysterically funny in 'Fawlty Towers'.
R: I appreciate the tragic both in opera and blues. I value a great deal in terms of Western creativity, so I simply find her statements unacceptable as a reflection of the reality of Tibetan culture. It seems to me that many of the statements I remember from your reading of the article are as naively positive about Western culture as she must have been originally about Tibetan culture. Neither culture, however, can be regarded as innately superior, and Dharma can flourish in both. Of that I am completely certain.
Q: The next line that stood out for me was her suggestion that 'sincere spiritual seekers return to themselves and appreciate the good aspects of our own culture in order to achieve spiritual satisfaction'.
R: It's hard to know where to start with a comment such as this. What, for example, does it mean to return to yourself? That is what we do continually in order to justify ourselves.
KD: Perhaps it would be better if she really returned to her own culture and took up the Christianity or Judæism of her parents or grandparents. But maybe that would be too demanding in terms of 'spiritual satisfaction'...
R: That is quite likely. Christianity and Judaism are also not as accommodating in terms of allowing people to be special and unusual - so interest in these religions would require more humility than is perhaps available.
Q: I can't help thinking that her phrase 'spiritual satisfaction' once again sounds as if she's talking about some sort of business transaction: 'I failed to experience spiritual satisfaction; I want my money back'. The next comment really surprised me - she describes His Holiness the Dala'i Lama as 'the most progressive of all Tibetans.' What do you think that means?
R: It's hard to say. I do not know how the word 'progressive' is being used. When I was young 'progressive blues' meant that it had gone electric. I think that progress - in Western terms - relates to things like scientific and sociological advances. It is not fitting that I comment on His Holiness the Dala'i Lama in this or any other respect - what is more to the point is how this idea reflects upon the author of the article in question. As I recall from your reading, she quotes His Holiness the Dala'i Lama in the context of world-wide social concerns, rather than spiritual issues. In this context the question of democracy is highly pertinent - especially with regard to the Tibetan people and the Chinese occupation of Tibet. It is obviously important for people to discuss such issues. There is not much more I can say on that subject as my knowledge of politics is severely limited.
Q: The next quote I have for you is '... lamas are sure they know best, and will likely not be impressed with your own speculations or reflections about spirituality.' It sounds as if she wants Lamas and their students to be in some sort of chummy discussion group 'sharing' their ideas and insights with each other as if they were all equally interesting and valuable.
R: Our five year old son Robert often likes to tell us his ideas about reality - and it's really quite charming and delightful to hear.
KD: Why would a brain surgeon be interested in unqualified untrained speculations and reflections on the best way to remove a tumour? I'm sure brain surgeons are also sure that they know best - and I'm sure that we also know that they know best, otherwise we would not have sought their help in the first place.
R: Yes... If you go to a Lama for instruction - particularly in the context of Vajrayana - you are obviously in a position in which you know precious little about the subject and are assuming that the Lama is someone who knows more. Also - if a Lama does not know fundamentally that he or she 'knows best', then he or she should not be accepting students. I think that the author is attempting to inculcate some kind of 'we're all in it together' ethos - as if democracy had any application to spirituality. Democracy has its place in the ordering of mundane human arrangements such as governments and food co-ops - but it has as little place in spirituality as it does in science. You can't vote about the nature of scientific discovery, as if the majority were right. "OK folks which is it to be? Flat earth or round earth - cast your ballots." This whole issue of democracy - I believe - is a means of creating a knee-jerk response in people. Accuse Vajrayana of being against democracy and it will upset people. Let me make this very clear: Vajrayana has no issue against democracy in terms of the way in which it is employed to order human affairs. But it has no place in the relationship between a Lama and his or her disciples.
Q: She seems to feel that there is some problem at the level of human freedom, though. She says: 'Buddha, presumably, was an individual, who through the exercise of his own mind, found freedom. Yet Thinley Norbu criticizes Americans for having "freedom habit". Must we choose between Buddhism or freedom? Perhaps in some brand of Buddhism appropriate to a feudal system, peasants do not ask these questions. Americans, however, would probably choose freedom, and thereby, I believe, true dharma as well.'
R: Not quite The Gettysburg Address - but a rousing speech nonetheless. There are several points here. Firstly, when Dung-sé Thrin-lé Norbu Rinpoche commented on 'freedom habit', I understood him to mean just that: the habit of having to be free. The habit of having to be free is not freedom. I can choose the freedom to have my entire body tattooed. I am free to blow my brains out with an eight-gauge shotgun. I am free to damage myself in countless ways and to engage in any manner of idiocy I choose. I am also free to accept discipline in order to change. If I want to lose weight, I am no longer free to eat whatever I want and as much as I want. So as to 'Must we choose between Buddhism or freedom?' - this is a manipulative construction - an artificial choice. The answer, if we have to answer it, is that those who take Refuge choose both. Discipline of any kind requires the temporary relinquishment of freedom. Even to engage in silent sitting requires that we temporarily relinquish the freedom to engage in some other use of our time. To get married is to relinquish the freedom to have other relationships - yet no one who is happily married would say that was an imposition on their freedom. I could go on. This whole idea of freedom is another red herring to decoy the unwary. My Lamas never trapped me in any way. All the vows I took, I asked to take - and my Lamas asked me to think seriously before taking them.
The second point is that it is not so simple to describe the cultures of those countries which followed Vajrayana Buddhism as feudal and their inhabitants as peasants. Take this quote from an inhabitant of Golok:
To advice of strangers we will not hearken. Nor will we obey ought but the heart with which each Golokpa enters the world. This is why we have remained as free in the past as we are now. We are slaves of none - neither Khan nor Dala'i Lama. Our tribe is the mightiest in the land of snows, and it is our birth-right to disdain Chinese and Tibetans. We regard them both with contempt.
This hardly sounds like a downtrodden peasant - yet this man was also devoted to his Lama.
Q: Is there a reference for that Rinpoche - that I can quote?
R: Yes... it comes form a book called 'Civilised Shamans' although I cannot give you the chapter or page. I have seen it quoted elsewhere too. Traktung Rinpoche once quoted it as well when he was talking about Golok and the family history of DoKhyentsé Yeshé Dorje - so what I have quoted here is probably a hybrid version from various sources.
KD: I imagine this may be a surprising quote for many Western people - in terms of common ideas about Tibet - but it illustrates the fact that it is not feasible to describe Vajrayana as suitable only for 'feudal peasants'.
R: The final point is that Dharma is true freedom and that to artificially separate them is merely another of the author's ploys to disparage Vajrayana.
Q: She talks at one point about lack of inspiration and says that 'We should be careful about adopting a world view that equates the outer world with ugliness and evil (samsara) and which urges "retreat" into "meditation" as the only refuge from a doomed existence.' It sounds as though she has about as much understanding of Buddhism as the Pope, when he wrote that book about how Buddhism was negative and anti-life.
R: Yes - and Dung-sé Thrin-lé Norbu Rinpoche addressed that issue extremely precisely - in his book 'Welcoming Flowers', so there is no reason why anyone should be unaware this life-negative view is utterly inaccurate - especially someone who studied with Gyaltrül Rinpoche, and would therefore have read this book.
KD: It was published in 1997 - and written in highly accessible English - so we can really say that this life and world negative issue has been concluded.
R: Yes - and into the bargain, this is a person who has received transmission of Dzogchen Trek gCod? I find that implausible. The equal purity of samsara and nirvana is fundamental to Dzogchen teaching. Also fundamental to Dzogchen teaching is Lhundrüp - the meditation in which everything is integrated into the non-dual state. If it is true that the author has received teachings on Dzogchen, then all I can assume is that she is deliberately ignoring every aspect of the teachings which do not suit her argument. This is a typical manœuvre in political polemic - but not one which gives rise to sympathy or respect in terms of spiritual integrity.
Q: And I suppose Surya Das must have been aware of this when he praised this piece of writing. There is another point I forgot to mention. When I compared Mrs. Carreon's article with the interview with Alan Wallace, I found that she had not quoted him accurately in various places - and that her misquotes always seemed to serve the purpose of furthering her argument.
KD: Can you give us an example?
Q: Yes - it's when she quotes Alan Wallace as saying that 'The finest lamas are now refusing even to come to the west..." Alan Wallace does not say that - what he says is: 'So a few of the finest Lamas are now refusing even to come to the West.'
KD: I think at this point we have probably addressed all we need to address.
R: I think you are absolutely right. There is little more that can be achieved by giving this material any further time. It is said that if one argues with a fool one becomes a fool oneself - so maybe if one argues with a political manipulator one has to enter too far into their world. I am sorry for whatever pain this lady has experienced - but I do not believe that she will help herself by taking the course she has adopted. I believe that if she were actually through with Vajrayana, that she would be getting on with her life. If she has now discovered happiness - why is she wasting her time writing about her pain? She may feel it benefits others, and if that is her intention then that at least is good in some way - but I find it slightly hard to believe that she does not realise that what she has written will be disturbing and hurtful to the many good people who have no problems with their Lamas or their spiritual home in Vajrayana Buddhism.

*********************

A Seamless Process:
Practice On and Off the Cushion
An Interview with Kamala Masters & Steve Armstrong
Kamala Masters and Steve Armstrong have been teaching mindfulness retreats together, both at IMS and worldwide, for more than ten years. They live on Maui, where they are developing a Dharma sanctuary and hermitage Ho'omalamalama (the ground for awakening). In talking with Insight Newsletter, they offer their perspective on the relationship between intensive practice and householder life.

Over the last decade, you've offered introductory courses for those new to the practice, as
well as longer retreats for advancing students. How do you meet the needs of students in different stages of meditation experience?
Steve: Kamala and I respect and teach from a Burmese model of understanding how students progress in their Dharma practice. The distinguished Burmese meditation master Mahasi Sayadaw, a pioneer in teaching meditation to those in a lay or householder life, outlined this model about 55 years ago. Prior to that, meditation instruction was not easily available to those outside the ordained community of monks and nuns. He taught that householders can develop the essential practice of the paramis, those positive forces or skillful qualities of mind such as generosity, renunciation, effort, truthfulness, lovingkindness and balance of mind.
These are the forces which, through various Dharma practices, can become our default setting - our first response to situations - rather than emotional reactivity, resulting in less harm and suffering. In Burma, householders practice the paramis in their everyday life, raising children, on the job and among their neighbors and friends. As these qualities ripen through ordinary daily activities, the ground is prepared for the deeply liberating insight that arises through sustained vipassana retreat practice.
Kamala: So we encourage the cultivation of the paramis at home, at work and in our social and civic interactions. While one or more paramisare practiced, other virtuous qualities are simultaneously nurtured. For example, whenever we remind ourselves to practice patience - the parami the Buddha called "the supreme virtue" - we not only create harmony in the outer conditions of our lives, but in addition support the growing inner presence of lovingkindness, equanimity and renunciation.
We also encourage students to sit a silent vipassana retreat every year, whether it's for nine days, or a month, or longer. When we take the time to temporarily disengage from household activities and move into a semi-monastic environment, the momentum towards progressively deeper levels of freedomin the mind joins forces with the momentum gained from strengthening the paramis in our heart.
Steve: The Dharma lifestyle of householder parami practice combined with intensive meditation retreats is a middle path between a full monastic way of life, and a full householder life without any spiritual practice. It is a form that seems to suit Western lay people. We have noticed significant maturation in our students who sit annual retreats, not only in the depth of their liberating insight, but also in their skillful application of Dharma understanding in daily life.
Kamala, can you say a little about the impact of your practice in Burma?
Kamala: Over the last two years, I've traveled to Burma three times - twice to undertake some months of intensive practice with my teacher and meditation master, Sayadaw U Pandita. The third trip involved sutta and Pali study at Sitagu Monastery.
For a long time prior to that, as I reflected on and practiced the paramis, the quality of renunciation resonated deeply in my heart. As I gave this more energy and attention, the wish to ordain arose - to shave my head, don the nun's robes and utterly simplify my life. I was interested to see if outer renunciation would help the inner renunciation of abandoning greed, hatred and delusion. So, I traveled to Burma in 2001 and ordained for two months with Sayadaw U Pandita. I did this again early this year. These two times have been amongst the happiest of my life; the simplification of my activities together with his monastery's clearly defined practice regimen allowed the constrictions of heart and mind to easily let go.
While in Burma I realized how supportive and strengthening it is for me, as one who guides others, to have my own guide. Of course, the bottom line is that the Dharma is our truest guide. There is, however, immense value in turning to someone much wiser who can, with fierce compassion, tell me where to refine the practice; a teacher who can direct and help me reach new horizons without embellishment or coddling. Two important things Sayadaw U Pandita said to me were, "You must be willing to be admonished," and "You must be willing to invest everything you have in the practice."
Is it possible to undertake intensive mindfulness practice while in a household situation,
fulfilling parent, partner and financial responsibilities?
Kamala: Being a mother and respecting family needs are primal forces within me. Nevertheless, while raising my children I also honored a deep need to take time out now and then for intensive retreat. Sometimes, I had to plan for this up to three years in advance. I didn't shirk that inner responsibility. So, yes, with careful preparation it is possible to fulfill both worldly obligations and intensive practice needs.
Recently, on the way to Burma, I visited India, and had the opportunity to talk with Dipa, the daughter of an extraordinary and wonderful Indian woman, Dipa Ma, whose teachings and practice have contributed greatly to my own development. I asked Dipa what was the most awesome thing that she remembered about her mother. She told me that when she was a child she and her mother were at the Mahasi meditation center in Burma. Her mother's teacher, Munindraji, instructed her mother to practice for three days, while Dipa was being cared for by friends there. This was not a weekend retreat of sitting and walking, with breaks in between; it meant Dipa Ma sat down for three days solid, without moving. Dipa said "She didn't even get up to go to the bathroom or to eat!"
This story inspires me, because it validates my own efforts to be both a devoted parent and a devoted meditation student - though I'm not sure I'll ever sit for three days without moving!
Another simple yet profound practice that helps create a seamless process between householder life and retreat life is known as 'Mindfulness of the Four Postures' - sitting, walking, lying down and standing. Munindraji, my other teacher, who passed away in October last year, often reminded me to practice a general awareness of the entire body as it sits, walks, bends or turns throughout everyday activities. I have found this technique brings about a significant continuity of mindfulness.
Steve, you were a monk in Burma for many years. How did this prepare you for developing a new sanctuary on Maui?
Steve: I spent five years in robes, doing intensive practices with Sayadaw U Pandita in Yangon. This was while Kamala was raising her family. Now, while she is undertaking intensive practice, I find myself much more involved in householder activities. Our roles have reversed.
We are in the process of building a Dharma sanctuary on Maui, which requires a lot of raising money, managing finances, and communicating with supporters, neighbors, contractors and the local government. The practices I did in Asia provide essential tools for skillful interaction in these often challenging situations. My household work is the test of my Dharma practice.
Do you see the teachings of Burmese masters continuing to influence Western Buddhism?
Steve: Most definitely! While Kamala and I were in Burma two years ago, we heard about a book in Burmese by the late Mahasi Sayadaw that had never been translated into English. It is a two volume work called Practicing Vipassana; a definitive book from the Buddhist teachings on how to practice vipassana, and much of its content is generally unavailable to Western students. In it, Mahasi Sayadaw discusses the preparatory practices that householders can develop and experience both in theireveryday lives and on retreat.
We have undertaken subsidy of the book's translation and publication. In January, Kamala asked Sayadaw U Pandita what he thought about making it available in the West. He responded, in English, "The sooner, the better." Not only will it provide a valuable resource for Dharma students and teachers alike, it will also establish a baseline in the West for reviewing our own practice and gauging its authenticity.
How relevant is the Buddhist understanding of mind today?
Steve: In our view, its relevance is just beginning to be appreciated, at least here in the West. Last September, Kamala and I attended the Mind and Life Conference with the Dalai Lama in Cambridge, Massachusetts, where Western scientists of psychology spoke with the Dalai Lama and senior Tibetan and Theravada monks and scholars about the nature of the mind. It was a fascinating dialogue to witness- Western scientists are just startingto document, through their scientificmethod and machinery, what Buddhistshave been experiencing for millennia, through their meditation.
For further information about Kamala and Steve's Dharma activities, please visit www.vipassanametta.org.

*********************

An Interview with Acharya Tenpa Gyaltsen
Gampo Abbey, August 23, 1998

Acharya Tenpa Gyaltsen is currently the spiritual director of Theksum Tashi Choling, the Kagyu center in Hamburg. At the Institute, he taught courses in Gyu Lama, Lorik and Takrik and Debate with much appreciated energy and clarity. At our prompting, he obligingly offered an account of his struggles and studies as a young monk. Born in western Nepal and drawn at an early age to monastic life, Acharya La was gravely disappointed by his first experiences at monasteries, which failed to provide adequate or inspired training. Help fortuitously arrived when H.E. Jamgon Kongtrul Rinpoche III advised the promising young monk to pursue higher studies at H.H. Karmapa's newly established shedra. Acharya La joined the first class at the shedra shortly after the Karmapa's parinirvana in 1981. This group of twenty-five or so young monks and tulkus received a classical and extremely rigorous training, under the guidance of Khenchen Thrangu Rinpoche and the Venerable Khenpo Tsultrim Gyamtso Rinpoche. This training produced a generation of exceptional teachers, who are highly in demand today in Asia and the West. The Dzogchen Ponlop, Rinpoche graduated number one in this class. While the Rumtek Shedra ultimately proved a great blessing, the rigors of study, including a daunting schedule, a weighty curriculum of texts and a very limited diet, were initially almost overwhelming. Many times he considered dropping out. Fortunately, Acharya La's classmate and friend, The Dzogchen Ponlop, Rinpoche, provided much welcome help and guidance.
Noting some similarities between Acharya La's exhausting studies at the shedra, and our own experiences at Nitartha Institute with difficult subjects, schedule, tiredness and food, we were moved to ask Acharya La for some pertinent advice for Western students of Dharma. The reply we received was surprisingly frank and very much to the point, touching on a number of vital issues for Western Buddhist practitioners. The following exchange, which climaxed our discussion, seems well worth pondering.
Translator and scholar Michelle Martin graciously sat in on our conversation and added her insightful comments. The Dzogchen Ponlop, Rinpoche visited us from time to time, as well, occasionally supplementing Acharya La's stories with his own remembrances of life and studies at the shedra.
Bodhi: Thank you for sharing your experiences at the shedra with us. Since that time, you've taught in the West, both in Europe and the United States. You've worked with a variety of students. Based on what you've seen from those interactions, we'd like to ask if you have any advice for Western dharma students? Although it's hard to generalize, do you have any suggestions for us?
Acharya Tenpa Gyaltsen: It's a difficult topic. My experience has been mainly in Europe, where the students are very fanatical. So far, I've met very fanatical students there. It's not blind faith exactly, but an extreme of devotion. On the one hand, that's very good. They are very devoted.
M: Is it kind of a blindness?
A: They are not interested in study.
B: You mean they are more interested in practice?
A: They have only practice. For them, study is not part of the practice. They think study is not really important. They have their own ideas. But I can see that when they begin to practice or go into retreat without an education, the students have a lot of problems. They don't have half of what is needed. There are a lot of problems. This is very clear. Without education, without having some ground of study, you cannot really know your practice. So this is my suggestion to the western student. You should of course practice. Your ideas and motivation are very good, but still you don't understand what motivation you have.
B: You don't even understand your own motivation.
A: You may have a good motivation for practice, but you don't know what practice itself is. This is what I felt. This is not completely the fault of the western student. Honestly speaking, there are some teachers who introduce practice and emphasize pujas and meditation as very important, but who don't introduce the study aspect. So their students don't have enough information about practice.
Michelle Martin: There's European lama who talks like that. A western lama. He says to just look at the nature of your mind directly. Practice, do pujas, do powa, but you don't need to study. He actually discourages people from studying.
A: So this is my advice.
B: Thank you. I think it's important to discuss this issue.
A: Sometimes I feel very sad, very sorry for new students. Somehow, they have an open heart. They have the karma to open the dharma in their hearts. But they've followed the wrong direction, met the wrong teacher. And then they close up. It happens everywhere.
M: I think this is particularly a problem in Europe.
A: I don't know America very well, but I think the same problem is everywhere.
M: I think, in general, it's a tendency of Westerners to think they don't need to study.
B: Westerners are so focused on the ordinary type of education: high school, college, and graduate school. It's so emphasized that you can't be successful in life or know how to deal with your life without this type of education. But then when we get into practice, that idea does not always carry over. I suppose we're so fascinated with meditation practice. Or maybe we feel like…
M: …we can't possibly study anymore!
A: That's one reason why, when lamas say you don't have to study, the students feel very happy. They're tired of studying for their whole life. When told they have to study, most students say, "Oh, I'm tired of studying. I've studied too much." So when you can finally forget all of those things and just sit in meditation, it seems very easy and simple. Therefore, I think it's one reason they don't want to study.
B: That reminds me of what you said when you were first thinking about going into the monastery. First, you visited your friend, and you saw how he lived. You thought, "Oh this is a nice life, a simple life." The life of a simple meditator.
A: Then, when you really get there…. I realized that you cannot just sit there. You can sit one day or two days but not indefinitely.
B: Can you discuss this further? How does study impact one's meditation practice? Your own background reflects a long period of emphasizing study. Then you entered a three-year retreat.
A: It's been very helpful to have had both experiences, I could say. I can really compare the two. When I finished studying, I didn't really appreciate or feel much gratitude for my education at the shedra. But when I went into retreat, I was really grateful and appreciated my studies at the shedra. In retreat, there was a mix of retreatants. Some were from the shedra but most of the retreatants didn't have much education. And I could really see the problems with this. The practice is not very easy. We had to meditate for long periods. And the other monks, who didn't have the education of the shedra, even the good monks, the smart monks, had difficulties with the practice. The practice itself involves reading a lot of texts. Our teachers, Khenpo Rinpoche and Bokar Rinpoche, would come and teach a text and then, of course, they left. The next day, we had to continue our practice. We had to read and follow the text, and these monks were completely lost and didn't know what to do.
Then they came to us, to Lodro Namgyal and myself. We were quite busy ourselves. However, we had to help them. Sometimes when Khenpo Rinpoche was there, for a few days it would be okay, because they could remember what Khenpo Rinpoche or Bokar Rinpoche told them. But sometimes there was quite a big gap in the teachers' visits, and then they didn't remember what they were taught. And they had some problems with their meditation. When they opened the text, they could not really follow what the text was saying. Sometimes, when we would listen to a text on tape (we had made some recordings), they could not understand the philosophical terms that were used. Even for practice, Khenpo Rinpoche and Bokar Rinpoche use philosophical terms, not ordinary terms. Without the education, you don't have the knowledge, the understanding, of what these words mean. You cannot find the meaning in a dictionary. You cannot find it anywhere.
M: Do you think it helps to have already thought about these things before, to have considered, "What is emptiness? How does samaya work? How do these mental factors work?" So you have been thinking about this for a while.
A: Yes. From our study, we know what the terms are referring to even before having put them into the practice. But without this knowledge of the meaning, you can know the words "mental factors" and "primary mind," but they are just words. You cannot really discriminate between the two. So when you meditate, if you don't know the meaning of the words, then you cannot really apply them to your practice. You're stuck on the same thing. You're just looking at your mind, and it's all the same thing.
M: So you can't analyze it. You can't break things down.
A: Right. You cannot break things down. Especially when we practice the Six Dharmas of Naropa, there are a lot of detailed explanations. At that point, if you cannot understand that detailed level of explanation, you cannot really meditate. You can just sit. Or you're just reciting the mantra. It doesn't help much. Realizing this, I really appreciated my training, and how important the shedra is.
M: Didn't Jamgon Rinpoche also change the traditional "counting retreat" to require less mantra and more study?
A: Yes. That was Jamgon Rinpoche's idea, but now I realize there are also obstacles with this.
M: Why?
A: Because I think that, before you go into retreat, your training should be done. In retreat, if you have to study and practice together, it is too busy.
B: So you should be able to just practice, and have your studies done?
A: Yes. Jamgon Rinpoche's idea was that in retreat you could practice and study at the same time. But that turned out to be too busy, and also it was not possible to finish all of the study that Rinpoche had planned. Mainly, we had to concentrate on practice, four sessions a day. And in between lunch and the third session, we had one hour of teaching. But it was hard of us to concentrate then, because we were so tired.
Sometimes I felt a little bit worried, not for myself because I didn't have to pay attention to study, but for some of the other monks. They felt the pressure of having to study. Then when they were meditating, when they were doing Vajrayogini practice, they would keep thinking about their study program. This was our experience. We talked to Khenpo Rinpoche about this. Khenpo Rinpoche decided that for six months before the monks would enter the retreat, they would study to prepare themselves. Once in the retreat, it's very important to be able to study and work with the materials related to your practice and the teachers' explanations.
M: Yes, for example, I heard that when you were doing the mandala offering, you would study the chapter on generosity from the Bodhicharyavatara. So in this way, the background text comes in to support the practice.
A: The teachers often explained to us the importance of what we read. So when we were doing Vajrayogini practice, for example, at the same time we'd be reading Namshae (phag mo'i rnam bshad).
Now I can really appreciate how much shedra actually helped. Looking back, I can say I was a little bit proud of my training, of being involved in the studies. But I wasn't genuinely appreciative of its value. So when I went into retreat, then I clearly began to see how important it is.
B: And that's that whole idea of joining study and practice.
A: I remember one lama there, in his late fifties. When we were meditating on the Four Ordinary Foundations (the Four Reflections), we had to meditate on impermanence. This lama came to me and said, "I cannot really meditate on impermanence. How do you meditate on impermanence?" He said, "I understand that 'this' should be impermanent and that 'that' should be impermanent. I think about this universe and how it will dissolve one day. My mother will die, and my father will die, but it doesn't really help." This didn't help him. It didn't really penetrate his mind.
M: It didn't change his mind.
A: It didn't really touch his mind. Then Lodro Namgyal explained to him that he should think about why these things are impermanent. You should think more about the reasons. It's not just hoping and praying that everything is impermanent. Not just hoping or praying that, but you should really work with your reasoning. Impermanence is not just based on simple meditation. It's more an analytical mediation. Then through that process, you come to know what "impermanence" means. When we know that one person has died, that is not a sufficient reason that everything is impermanent. It doesn't help. Then Lodro Namgyal explained the more subtle reasonings for why things are impermanent. And then this lama practiced in that way and later said, "This is really helpful."
M: The reasoning enabled him to go more deeply.
A: Yes. He said that it made sense. In a similar way, without studies, you don't know what "impermanence" means. When one person dies, or one house collapses, this is not really a sufficient reason to test your mind. When you really practice, then you have to really think about all of the reasons why something is impermanent on a very subtle level. You have to go more deeply. Then this helps.
B: I think we've gotten some idea that successful practice is not thinking.
A: This is true. Successful practice is non-thinking, but in order to arrive at non-thinking you have to think. This is like the saying, "if you want to get water from ice, you have to melt the ice." If you want to get water without melting, completely melting the ice, you can't do it, right? In order to get that water, you have to melt the ice. If there is no ice, that's fine. Then we don't have to melt it. But somehow, we have the ice, and we're thinking, "I don't want to melt it, but I just want to drink the water." This is the stupid way we have of thinking.
M: You need some heat.
A: This is what all the teachings are saying. When you have already produced the heat, already melted the ice, then trying melt it again is also an obstacle.
M: So you need a certain amount of thinking, enough to melt the ice. But once you've melted the ice, then you don't think anymore. Then it's non-conceptual. And if you continue trying to melt the ice, then you're overdoing it.
A: Same thing if you want to go upstairs. You're standing here, on the bottom floor, and you say, "I don't want to start from this lower vehicle, from the first step. The real thing is, I want to go up there." If you don't want to start from here, you're never going to reach there. This is stupid, not thinking of this. But when you have already reached the upper level, if you're still thinking that you have to start from the first step, that is also useless. So first, in meditation you have to think and think and think and think. But at the second level, you don't have to think. You've already reached that level.
If you want to be just a simple meditator, then maybe it's not necessary to go through all of this study. If you just want to be a simple meditator. Just sit in meditation. Just remain simple, which is not a Buddhist meditation.
M: You want to relax a little bit.
A: For this kind of meditation, you don't need to go through all of this study. Even though you won't go any further, you will develop some calmness. But if you really want to follow the Buddhist path, then you have to study. For example, if you want to do some shamatha meditation, shamatha is not just calming your mind. And when you go on to meditate on lhagthong (vispashyana), how exactly do you meditate? And when you meditate on mahamudra, you should think about what you are meditating on. We're not just sitting blind and closed to our experience. We're not blocking our thoughts. We have to work with our awareness.
When you think about a simple kye rim (generation stage) practice, when you visualize Vajrayogini, you are visualizing Vajrayogini as a form. First, this is just an illustration. Through generating this illustration, we see that Vajrayogini has two hands. What do these two hands symbolize? What does her crown symbolize? What do the third eye, necklace, feet, and corpse each symbolize? You have to remember these details. You're not just making up a picture. Through the picture, you have to remember the meaning. The visualization itself raises a series of deeper and deeper questions. Why does Vajrayogini stand with one foot down and one foot raised? It shows that Vajrayogini is full of compassion and wisdom. Then you have the question, what is the wisdom here? What is the compassion here? Without that information, without the knowledge of compassion, without the understanding of what prajna means, your visualization has benefit, but only the benefit of creating a form, like the painting of a thangka. There is some merit, similar to building one statue of Vajrayogini. But it doesn't serve your purpose, the real purpose of your practice.
M: If you want to transform your mind, then you need to know the deeper meaning.
A: You are not just creating some form or picture. Every detail of the picture symbolizes something else. So without this knowledge from study, even if you can remember from the text that the necklace symbolizes the fifty-one mental factors, then what are the fifty-one mental factors here?
M: What do they do, and why are there fifty-one? (laughter)
A: And why do you have to transform them?
B: Good point.
A: How do you transform them? If you do not know, first of all, the fifty-one mental factors, you cannot deal with it. Actually, in the Vajrayana, the whole analytical meditation is present. This is the very, very great benefit of Vajrayana. It is an analytical meditation. At every point, you have to remember plainly the meaning along with the forms. When you are visualizing flowers or a garland of heads, you have to simultaneously remember the meaning of the fifty-one mental factors, the six paramitas, and wisdom and compassion. At the same time you're doing analytical meditation.
M: All the generation stage practices are like that, all of the visualizations. They're coded.
A: Maybe there are some beings who are very fortunate, who have fortunate karma from their previous life, who don't have to study. They can remember everything when they see the image. Otherwise, it is difficult to meditate.
B: It seems that it's simply going through the motions, hoping some kind of result will arise, just because you're doing it. But without the understanding.
A: Difficult. Difficult.
B: It's like having a simplistic understanding of buddha nature. We might have the belief that if we just sit there long enough, that perfect mind will arise and all of our problems will be over (laughter), because understanding will come with it. We are assuming that that understanding is automatically accessible through some simple form of meditation. But it doesn't quite come like that. There's a certain amount of effort needed.
M: Refinement.
A: Also, having only study can be dangerous. I can say this from my own experience. I went into retreat, and even though I didn't really practice well, I could see the gap between practice and study.
B: You've seen both sides.
A: Both sides, fortunately. So I can tell people. Study is one of the seeds for pride.
M: Study is a cause for pride.
A: It doesn't really help, actually. It doesn't really help without meditation. I could say that, without meditation, study is just "blah blah blah." It is something we're all just saying about practice.
M: It's just words.
A: Study without practice is like a bubble of water. That's a saying, which is really true. This is really true. First I can say study is "here," in our head. But I say "here" is not a buddha yet. Buddhadharma has a long way to travel from "here" to "here" (from our head to our heart). So first when we study, our knowledge is "here" (in our head). We know a lot of things. But not in "here" (in our heart). In the end, we have to bring our head to our heart. That is the purpose of meditation.
M: Getting your head in your heart.
A: If you want to bring your knowledge into your heart, you have to bring it through your head. You have to transfer it through the process of meditation. Of course, it's individual karma. Each individual is different. But for me, first I have to come through my head. And refine, refine, refine. Then I can accept. Then you can come to the heart through meditation.
M: That's what Rinpoche says about what reflection is for, no? Studying and reflecting and meditating. Khenpo Tsultrim always says that reflecting is to make things certain in your mind, to eliminate your doubts. "Oh, that's really the way it is. I understand it, and that's clear for me now." And then you can really meditate on it, because it's clear and you believe in it. There's no block.
B: Are we talking about prajna and compassion again?
A: I'm not talking about prajna and compassion. I'm talking about how Buddhist practice should be. Study and meditation. From one perspective, "Buddhist" can mean whatever dharma or philosophy you've learned. But that's not talking from here, from your heart. You're talking from your brain. And we should talk from our hearts. Whatever we have learned, we should not leave in this brain. We should bring that to our hearts. I think this is what Buddhism means.
This is how I understood that meditation helps to bring real wisdom into the heart. Meditation and study balance each other.
[end]
This article is part of the contents published in Bodhi Issue 3 (Winter 1998).

*********************

An interview with Ani Thubten Dekyong
Note: The title Ani, used for Tibetan Buddhist nuns much as the word "Sister" is used for Catholic nuns, literally means "aunt." Rinpoche, which means "most precious treasure," is a title expressing respect and affection for high Tibetan Buddhist religious figures. Similar to the Japanese -san, the suffix -la is appended to Tibetan proper names and titles to signify respect. Ani Thubten Dekyong's lay name, Tseyang, is pronounced as "tsen." This interview was conducted for Glow magazine in 1996, when Tseyang-la worked as a translator for monks from Sera Je Monastery in a U.S. tour performing David Patt's Wild Life, Tamed Mind to raise funds for the monastery.

Could you tell us where you were born, where you grew up, what the basic conditions of life were?
I was born in Tibet, in Lhasa, in 1960. We came out in 1965. My parents worked as merchants, selling and bartering farm goods. My immediate family wasn't politically active, but they were very religious. I grew up in Nepal, and then I went to a couple of schools in India. I finally ended up in a boarding school in Kalimpong, near Darjeeling in the Indian state of West Bengal, until I finished high school.
You had a brother who was a tülku (reincarnation) at Sera Je. Was he a strong influence on you?
I wouldn't say he was a strong influence, because he was much younger than I.
What was your education like?
It was a convent school run by Catholic missionary nuns. We came home just once a year for winter vacations. My parents lived in Nepal, and travelling was difficult then. It used to take three days; you had to go on a bus, get on a train, change to another train. Now travelling is more convenient and you can do it in one and a half days.
The student body was a complete mixture - Nepalis, Indians, Tibetans, Bhutanis, Sikkimis. Our schooling was completely in English. We had Hindi as a second language and, for part of the school year, up to the sixth grade, we had Nepali as a third language. But we spoke Tibetan at home. And when we came home for our winter vacations, my brothers and I occasionally had tutors who used to teach us Tibetan.
Doesn't it make you feel ripped-off that the school didn't teach you about your language, your history or your culture?
No, no. We were growing up in communities in exile, so it was interesting to know the history of the place where we were living. In fact, as immediate information, this was something more appropriate. When my parents chose to send me to missionary school, they did that politically because they felt the missionary schools offered the best education. For something that is as personal as developing your own scope of your own knowledge, I think, it's your own individual responsibility to look for it. If you're going to wait for all your knowledge to fall into your lap, it's not going to happen! You have to seek it somewhere.
When did you decide to become a nun, and what led to that?
During my high school years, I used to visit Kopan monastery. I took a couple of meditation courses that were being offered to the Westerners. It was actually much more out of Lama Yeshe's encouragement. It was much more his idea, initially, than it was mine. I just knew him as a lama who was a great friend of my father. He would make a great point of telling me to come to the meditation course, and my father would remind me that I had to do that. And if Lama told me that I had to come by, I felt it was important that I did that. So initially I did it more for him than out of my own personal interest. I didn't want to disappoint him. I wanted to fulfill the expectation that he had of me.
So at first it was more for Lama Yeshe and for your father than for yourself. Once you got there, was it for Lama Yeshe and your father, or for yourself, or for other women?
I had grown all my life in a Catholic convent, and being a nun of any kind hadn't really occurred in my mind. As I began to learn more about Buddhism, it became my own personal interest as well. Then when I finished high school, joining the monastery felt like the next best thing to do in my life.
Joining the monastery, or joining a nunnery?
It was a monastery! We didn't really have mixed monasteries. The core group was a monks' monastery and, if there were nuns, they were just Western nuns. There weren't any Tibetan nuns when I joined it. I was the first one. It was Lama Yeshe who was instrumental in bringing me in. He just said, "You be a nun here!" He and Lama Zopa created the gateway or the path for me to be accepted.
He suggested it? You didn't ask?
All my studies, all the learning of Buddhism that I did was done at that monastery, and I didn't think I had to go to some other monastery now that I had become a nun. And since they had accepted me, ordained me, I kind of presumed I was going to be in there.
For the monastery, it was a challenge to have a local Tibetan nun actually be a part of the community. Because if you accept one, you've got to accept all the others that come. In the early years, the admissions were a bit restricted, because the monastery didn't have the facilities for it. We lived separately, but we ate from the same main kitchen and we went to the same classes as the monks did. I joined in 1979 and, in '82, there were two more nuns. By '83 or '84, there were two more. By '86, there were eleven nuns. It was done very cautiously - trying out one, trying out two more, and so on. Even when I entered, I thought that a lot depended on me, on my way of behavior, to either stop or make it flow.
You had to break new ground for other people to follow in your footsteps.
I felt like that, yes! I was the first local nun, living with a core group of monks. I did feel a sense of responsibility, because I knew that if I made a mess of it, nobody else would get in. And if I made it a success, then others would be able to get in.
Were you met with a lot of hostility or resistance, like the American women who were the first to integrate the armed forces or like blacks who integrated previously white-only schools?
It was difficult for the first few years, but they didn't discriminate against me in that kind of way. Listen, I had finished high school. At least I had some basis of education, not particularly in monastic learning but, still, I had done a kind of study, which other monks hadn't done. It was difficult because here you are, you know, a very new dharma student, very enthused by the spiritual path, and you're in a community, and it's just all monks. I do remember thinking, "There must be an easier way to a spiritual life than what I'm going through."
Was Lama Yeshe as encouraging towards other would-be nuns as he was towards you? And are there other lamas or abbots or monks who are as encouraging as he was?
I don't know about other people, but I do know Lama was very encouraging. Just as he was instrumental in bringing me into the monastic community at Kopan, it was he who brought in the next two nuns, too.
How did things change for you when you went from being the only nun to being with other nuns?
What happened was that, just as the first two other nuns joined me, Lama sent me to Australia to translate. I was there until '85. While I was in Australia, Lama Zopa sent this letter about how wonderful it would be to have a nunnery, how beneficial it would be, and so forth. When I came back, there were four or five more nuns. By '86, there were eleven!
Was that just the tip of the iceberg in terms of how many women wanted to be in there?
I think so. The monastic community is pretty crowded, so it was a big step for them to make room for a community of nuns. Anyway, then in '86, Lama Zopa actually said it, about building the nunnery. That's how the idea started off. I think when he wrote to me in '84, he kind of expected me to come back and do it. But at that time, I didn't really think I was being asked to do it.
He was planting the seed in your mind.
Yes, probably he was getting me warmed up to the idea.
When you started the Kachoe Ghakyil Nunnery, was that a separate institution, or were there organizational ties to Sera Je or to Kopan?
Lama Yeshe and Lama Zopa are connected to Sera Je, and Kopan is a monastery that they founded. And the nunnery is a result of Kopan. I didn't go to Sera Je; I've never been to Sera Je. I started studying Buddhism at Kopan, first as a layperson and later on as a nun. And the nunnery happened because of Kopan. It shares teachers with Kopan. But for buying land and building, as nuns we had to find the money and everything to do it.
How many of you were there?
There were eleven of us in the beginning. By '88, we were 18. Then 20. As it became more apparent that the nuns were going to do a place for themselves, more nuns were allowed to join the community. By the time we finished building and the nuns moved into the property, which was in November of '92, there were more than 55 nuns.
How did you do it? How did you raise the money?
Oh, I don't know how I did it! I just begged for them. I asked for money from different people, wrote brochures, did interviews so that people would hear about it and give us money. I did most of the fundraising outside, and then we did a little bit within the community in Nepal. There is very little money in Nepal. But we were able to raise almost all of it to buy land and to build enough so that we could move in.
We hired local people and, for the first year, the nuns also helped. Because the initial year doesn't require much skilled labor. It's clearing dirt here and there, stacking up bricks, carrying this and that back and forth. So for the first year, all of the nuns participated. Plus, we needed the nuns to look after the construction site, make sure people were working. We had a meeting, and we decided that this would happen. It was also: then it would feel like it's part of their life, as opposed to just me running around, getting money, build it and "Here's a place!" I thought it would be good for the spirit or the morale. And since they didn't participate in the actual fundraising, I thought it was the least they could do. We all had our little jobs for the first year, except for the youngest ones; we all had roles as to who would look after this or who would look after that.. The second year, we needed more skilled labor. We built the nunnery between '90 and '91. It was only in November of '92 that we were able to move in.
Now there are over 120 nuns. Initially, when I was first planning, I was thinking, "Oh, I'm never going to get enough money to build, for a big community. Maybe I should at least aim to make the place big enough to house 50 nuns and let the community grow from there." We had 50 nuns by the time we moved in! So they've been just adding layers.
What are the nuns like, and how do they support themselves?
Their age range is from eight to forty. Everything has to be taught at the monastery, starting with the alphabet, so older nuns teach the younger ones. Sometimes their families contribute to their support, but never on a continuous basis. They give what they can, when they can. Kopan Monastery gives them some money for their food and things like that. A certain amount of money each month is allocated to them, so if they run short of it, they have to make do. If they have a surplus, it's also for them. So they get a certain lump sum each month, divided by the number of nuns that are there.
Is it anticipated that the nunnery will be self-supporting at some point?
I left the nunnery shortly after the nuns moved in. I left in early '93. So by now I'm not so involved. The two nuns that entered the monastery after me are kind of managing everything. Now there's a place for them to live and everything, and if donations happen that's great.
What's your connection with them now? Do you just drop in and say hi when you're in the neighborhood, or do you live there when you're in Nepal?
Now it's more informal. I just drop in when I'm in Nepal, offer them lunch or something like that. Offer them tea, offer them some money, and that's it. Because I've been living mostly in the States since mid-1993.
Do you plan to go back?
I don't know. I haven't decided.
A problem that David Patt brought up in my interview with him is that the best and the brightest of the lamas - and, obviously, now nuns - Tibetans who come to the States and get a taste of Western life and luxuries [Tseyang-la howls with laughter] frequently give up their robes and don't want to go back. And even those who don't give up their robes still don't want to go back. It's a great benefit to the US, but it's a real loss to the Tibetans. Not that we don't want to have you here, but Tibetan culture and people are hanging by such a thin thread that to lose their people to Western decadence (or civilization or whatever you want to call it) may be the difference between life and death for Tibetan Buddhism. These are my words, not David's. It seems to me to be a real problem that needs to be addressed: Why isn't there enough there for you to be able to stay? Or is it that you think the needs here are so much greater, or that you could do more for them here than you could there?
I know it would be more meaningful in the long run for me to stay there and help out. But sometimes it's difficult when you have to work. Because I was responsible for the nunnery, I still had to work in conjunction with the main monastery. And it got difficult to work.
How so?
I think I was too much of a problem! [laughs] I didn't quite fit the mold of what a Tibetan nun should be.
I'm sure every woman who's been the first has gone through that one, not knowing if it's because of who she is individually as a person or simply because she's female. It's usually because she's female. It doesn't have anything to do with you!
It's like, if I have to challenge, it's much more of a challenge than a monk's challenge. I stuck it out until they all got established. Had I left any earlier, the nunnery would never have been finished. But the moment everyone moved in, I said, "That's it!" I thought it would be better for our relationship if I wasn't there and they got to handle the situation. You see, when we were building the nunnery, although it was Lama Zopa who suggested it, we didn't get any financial help from Kopan in those first years. They didn't say, "Here's five cents toward the nunnery." We fundraised from scratch! Although we lived at the monastery at Kopan, it was fine as long as we were dependent. But to create this independent situation with our own land, our own property - in that, we didn't get any financial help. Generally I should say that it's harder to raise funds for nuns than for monks.
Was it that they didn't want you to be independent, to be in control of your own lives, to be making decisions for yourselves?
It's funny, because we are now lightening the burden by coming out of the community. But I think the thing was mostly that all the money I collected, I should have given to the monastery and then said, "I need money." Ten dollars here, a hundred dollars there. They wanted it like that. And I didn't see it that way. I raised the money, and I felt responsible for the money, and I was doing the best I could, from my perspective. I think that created bad feelings. And then, Lama Zopa used to tell me, "You should tell them what's happening, talk to them."
Would you say it's any of their business?
Well, first of all, they didn't show enough of an interest in us for me to feel that I would go to them and tell them my sob stories or my happy stories. You share your stories, in sadness or in happiness, with people whom you feel are concerned or would be concerned or are interested. You wouldn't go to tell anyone with whom you felt strange or who was indifferent towards you. This is what I felt; I'm sure they would say otherwise, that they had our interest in the depth of their heart. They could easily say that. But personally, I didn't feel that kind of pathway to go to them. So that didn't create a good situation.
In situations where nunneries have been run under the auspices of monasteries, haven't there sometimes been problems where the nuns were basically expected to wait on the monks, do their laundry and clean up after them, handle the cooking and cleaning and all that stuff so that the monks would be free to study?
I have absolutely no idea, because I never lived in a nunnery other than what I got myself into! There was one time in the '80s when I could see laundry hanging on the nuns' side that I didn't think belonged to any of the nuns. I told the nuns that I didn't think this was appropriate. I told them the only people's laundry that I wanted to see was their own, or that of very, very young monks that couldn't take care of themselves, or some of the older monks who shouldn't have to do laundry. I felt it was okay for the nuns to do that, because the monks are giving them teachers and they don't pay a penny toward their education. The least they can do is wash their robes, for the youngest and the oldest. But we're not washing machines!
I do feel that it was all worth it, now that it's created a space for so many people to come. It would have been good if I had felt like it was a good situation to still continue on. It would have been good because I could have done more. But right now I feel that it's better for me to give breathing space to them - or to myself!
Then there's just the concern as to whether or not they'll be able to fight for themselves and speak out in the way that you would have.
When I was there, if something came up, they would complain to me, but they would never bring it up to the monks that were in charge or had something to do with the nunnery. Initially I would bring it up to the monks but, later on, I would teach them, encourage four or five of the nuns to go visit together and let one be the spokesperson and say "This is the problem." So that it wasn't just one person. I found little ways to communicate like that.
Is this part of a problem of lack of ordination for women or basic Buddhist stuff that monks, as you say, were "in charge"? Were the monks in charge, or were the nuns in charge? Who's making the decisions, who's running this institution?
It's a Catch-22 kind of thing. How Lama Zopa and Yeshe Rinpoche's organizations are run, whether it be a monastery or a nunnery or just a dharma center, is that we all have directors. Each place has its own director. We have a board and, under the board, we have directors for each center. All the directors are hand-picked by Lama Zopa or Lama Yeshe. It's not like there's a community election. Since Rinpoche appointed me as a director, I felt like I was only answerable to him. And because we didn't have any financial dependence on the monastery to get the money for the land or the building, I felt that we were independent. They apparently didn't see it that way; they felt that we are nuns who come to the monastery to get education. They already see themselves in the role of teachers, and they feel they have some mandate over the situation.
So it won't be until we have our own women teachers that we can be independent.
I think so. I think there's always going to be a little bit of conflict of interest.
What about you personally now? Do you see yourself staying here in the US forever? Will there be a conflict between keeping your robes and living here, or will you be a nun forever?
I'm still dedicated, still want to remain as a nun for the rest of my life. That's for sure. Here, although it isn't specifically a nun's situation, I still devote a lot of time to my own practices. All the years that I had to fundraise and get the nunnery together, my own practices or studies were secondary; I kept them to the minimum. Being here in the States for the last two years has allowed me a lot of time for myself, to concentrate on my own practices, all the teachings that I've heard. To just do that. And also, I make myself available to teachers that come here in the States and need translators. There are a few dharma centers that know where I am and can call me if they ever need a translator. It's a great learning process for me, much more than just listening to a teacher. That in itself is a great blessing, and I give that first priority. Second priority is just giving myself more time for my own study, for my own practices. However long it takes me to go through the practice, I'll do that, because I know I have the time. Occasionally I give talks or things like that, depending on where I am. I feel kind of satisfied doing this. I don't feel it's necessary for me to be running a whole organization. But I do know that, had it been different for the nunnery in that situation, I know I could have made it. I'd still be there, still running around like a chicken with its head cut off.
Except that you might have been able to empower more women and delegate more responsibility so that it wasn't all on your shoulders.
I think so. And in some way, they looked up to me for an inspiration. Although sometimes they would feel that I was more Western than Tibetan.
Well, sometimes you were the scapegoat, and sometimes you were the inspiration, right?
Yes! I worked in both ways. So, I don't know, I haven't decided. I would say that for personal benefit, this is my best situation. But if you think about the good of the many, then it would be better for me to go and still live in the nunnery and do what I can for the nunnery and for the nuns.
As a result of being in Australia and here, are there things you've seen, learned, ways of operating, that you want to take back, that you would do differently now than when you were there before?
I think the organizational skills would be very helpful, because in some ways monasteries have a very autocratic way of getting things done or decided. When I was with the nuns and we used to have meetings, one of the key things I used to stress to the nuns was "Here are the jobs we've allocated to ourselves or to each other. And we all decided that this is the best way you can help out in the situation. But should you feel you are being underused or that there is a field where you can do even better, then you should say so." This was the thing we always worked on. Some said, "Oh, I think I could do this." And even when I had my doubts, I'd say, "If you feel you can do it, you're welcome to try." We were all trying and learning, because none of us were equipped with the knowledge to do everything.
Mostly the key is what you feel about yourself - that's the most important. The confidence in yourself, not how society perceives you or into what grouping or what area society has pocketed you. That's secondary. What bypasses all that is your own confidence in what you feel you are. Because of that attitude, that feeling, people give you credit. They treat you at the level at which you perceive yourself.
But there's still a difference between perception and reality. You may be strong and independent, but if they don't want you to be strong and independent or intelligent and resourceful, then you have a problem. And it's not a problem of perception; it's a problem of power, a problem of reality.
It comes down to proving them wrong, or irrelevant. To convince them that it isn't quite what they perceive or what they think it to be. Of course it's harder. When have men ever got to prove their credentials? Women have always had to prove their worth, that they are credible or whatever. In that way, there's more effort involved as a woman.
They say a female rebirth is not quite at par with a man's. When I think about all the hurdles we have to jump, I can understand. Because they hardly have to do anything. We have to jump, skip and hop all the way.

*********************

An Interview with Hannah Nydahl
In 1969, Lama Ole Nydahl and his wife Hannah became the first western students of His Holiness the XVI Gyalwa Karmapa, Rangjung Rigpai Dorje, one of the greatest yogis of this century, and the head of the Kagyu tradition of Tibetan Buddhism. His Holiness had a profound influence on their lives. He asked Hannah and Ole to bring Buddhism to the West. For the last 22 years they have been traveling non-stop, teaching and setting up meditation centers around the world.
Hannah Nydahl is a much sought after translator and interpretator of Tibetan Buddhist philosophy. She divides her time between translating for the lamas at the Karmapa International Buddhist Institute in New Delhi, India, participating in various Buddhist text translation projects, organizing schedules and visits of high Rinpoches in the lineage, and traveling around the world with Lama Ole.

Kagyu Life International: How did it happen that you spent so much time in Asia?
Hannah Nydahl: Ole and I went to Asia the first time in the 60's. We connected with Buddhism and stayed there for a few years. There was no Tibetan Buddhism in the West then, so the connection with the East was still very important. My function became to translate for the Tibetan lamas, and help them organize their schedules. Also, for many years, Ole and I arranged pilgrimage tours to the East, taking approximately 100 people at a time, every year or two. This gave me a lot of contact with Asia. For the past five years I have also been involved as a translator for Tibetan teachers at KIBI (the Karmapa International Buddhist Institute) in New Delhi.
For people starting on the Buddhist path today, the situation is quite different. You can become a Buddhist in your own country, and learn and practice everything there. It may be good for your development to go on a pilgrimage, in order to visit places carrying a special blessing such as Bodhgaya, the place of Buddha's enlightenment. But it is not necessary to go and live in the East. I only go myself when I have work to do there.
How did you learn Tibetan?
In the late 60's when we met with Buddhism, very few texts were translated, and few teachers spoke English. We had to learn Tibetan ourselves, and I started by learning the alphabet from Tarab Tulku at the University in Denmark. Then, when we stayed in India in the Himalayas and did our practice, we had to translate all the meditation texts ourselves. When we did the Ngondro practices we started with the prostration text. I looked up almost every word in the dictionary and slowly translated the text. It was the same with the other parts of Ngondro. At that time we were in a retreat-like setting and did not talk much to people, so we did not get to practice any spoken Tibetan. To practice speaking it we had to stay in the Tibetan camps where nobody knew English.
Later, we invited the lamas to Europe and there was no one to translate, so I had to learn more Tibetan in order to translate for them. It was a natural process. Translating became part of my role, and Ole went into the teaching activity. He is a born teacher, and not a born translator (Hannah laughs). If he were to translate, he would give his own teaching (still laughing). So it fits like this.
Again, the situation today is very different. Now many teachings and texts are translated and many translators are available, so one can easily practice Tibetan Buddhism without knowing Tibetan.
How and when did you decide to give up the traditional family role? Did it happen early in your marriage?
When we went to Asia on our honeymoon and met with Buddhism, we stayed there several years to learn and practice the Buddhist teachings intensively. Later, we got the position of working full time for the Dharma. H.H. the 16th Karmapa was very precise in his instructions to us. He wanted us to go back to Europe and work for the Dharma. At that time it was not possible to combine this work with normal family life - it was a matter of making a choice. The choice was easy, there are enough children in this world, and what we were doing at that time was more important than having our own children. Today it is a different situation. Becoming a Buddhist does not mean changing one's lifestyle as we did.
You and Ole were the ones who actually brought Tibetan Buddhism to Europe.
It became our responsibility because there was no Tibetan Buddhism available in Europe at that time. Our development was not a typical one, it was a specific function at a specific time.
How do you maintain your balance when so many people make demands on your time, and your every move is watched as being significant?
Making demands on one's time is OK, and actually this is not a big problem. Concerning people watching one's every move, I would like to mention something of general interest. In the West we have a tendency to become a little artificial and fanatic around our teachers. We look at the teacher, watch every move he makes, and give special meaning to each word he utters. In Europe we have this tendency quite a lot; I don't know how it is in the States. We should try to be more natural towards teachers, towards Rinpoches. Our devotion can be kept internally - it does not have to show on the outside in an extreme way. It is not necessary to be physically close to the teacher, or look at him all the time. If one has trust or devotion, this does not have to be shown outside. It is important that, as Buddhists, we give more care to the kind of impression we make on the outside world, since people already have a hard enough time understanding what Buddhism is, and we don't want to be confused with the many cults coming up these days.
How do you maintain your balance living in the shadow of such powerful men as Ole and Shamar Rinpoche?
No problem! (she laughs) I do not have ambitions in that way. I don't see myself as being in anybody's shadow. I am just myself.
In your practice of Buddhism over the last 25 years, what are the stages of development you have seen in yourself?
From the moment I first met with pure Buddhist teachings, it was like a revelation. Since childhood I always had many questions in my mind - I wondered about the meaning of existence and such things. Denmark is a Christian country, but not very religious, and the Christianity I met there did not give me the answers I was looking for. I could not accept the concept of one creator God, the rhetoric that if you did not believe in God you were doomed forever, or that people who did not believe in God were lost. This never made sense to me. I was also very concerned about what happened to the mind when one died. I wondered a lot about these things when I was very young.
Later in puberty, I was involved in a lot of mundane activities (she laughs). I got distracted, and was not so occupied with these questions. Then, I met Ole and we started taking psychedelics. For me this was a continuation of looking for answers and especially trying to explore the mind. Apart from breaking some rough concepts about the world being solid and real, and thus getting a taste of the illusory nature of things, psychedelics did not give any answers either. The problem with them was that one clung to the experiences as being real instead, which was even worse and more difficult to purify.
The first direct Buddhist teachings I read were in a book called Tibetan Yoga and Secret Doctrines in 1968. In the beginning of the book was a text by Gampopa The Garland of Precious Jewels, translated by Evans-Wenz. This is a collection of teachings presented in sets of advice that starts on an ordinary relative level and takes you through to the absolute teachings. It gave answers to all the things I had wondered about. It was a very strong experience for me, like coming home.
After that, we met our teacher the Karmapa, and started to practice. Ever since then, it has been a process of trying to integrate the teachings as much as possible. It is amazing how vast and profound the teachings are, there is no end to them. Every instruction and practice I was given always confirmed the truth of Buddha's teachings and took me deeper into understanding. Feeling how much the Dharma has helped me, and seeing how much benefit it has for other people too, I feel extremely grateful to be able to use my life the way I do.
Have you ever doubted or been discouraged about the Dharma?
No. If something unpleasant or disappointing happens, it only confirms what the teachings say about the impermanent and changing quality of everything conditioned. As a child, I had an easy life, which maybe is not always so useful for learning to deal with difficulties, but at the same time it helped me gain some inner stability which has been useful in my later work.
How can one maintain a pure view and not be naive?
This is where the Dharma helps you. If you do not know the Dharma, then you tend to live in some unrealistic illusion and think that things are what they are not; you give things a permanent existence which they do not have. You may think something is wonderful, then suddenly it is not wonderful anymore. Or you see defects, start judging and thinking everything is terrible, there is no solution and you want to commit suicide or whatever. I can understand how people can get desperate if they don't know the Dharma - those who look at the world without seeing the whole picture can be terrified by what they see. But once you know the Dharma, it is not so bad. You can see the potential in people, get the right perspective. Even when there are wars and catastrophes, you know, at least theoretically, that this is not how things really are, and it is only a question of everybody understanding the true nature of things for these sufferings to stop. If you meet people who behave strangely, you do not take it personally. You think more of how you can advise them. It is no longer a private thing. This is how the Dharma helps us. To have the pure view means to see how things are in their essence.
What about the situation when people who are close to you turn against you?
The teachings say that we can understand impermanence by seeing how friends turn into enemies and enemies turn into friends. This is very true. Of course it is sad when a good friend turns against you, something is destroyed, but this has not happened to me as much as to Ole. Because of his function and dominant appearance, he is in more situations where people either adore or hate him. Some people like to see him as their idol and try to imitate him. Sometimes it is these exact same people who turn against him out of pride and jealousy - they suddenly make Ole into the devil. Ole does not take this personally. It is a pity when it happens but we learn a lot through this. One learns about the mentality of people and about the different approaches people can have. You learn how to deal better with situations, how to relate to people, and how to prevent these things from happening again. You can see certain tendencies in people and then be more careful about the kinds of relationship you have with these people - for their own sake.
Do you have any plans to write a book?
I have had different suggestions from people about this. One suggestion is to write a book about Dharma experiences, and another is to write about being a woman in Buddhism. It might be useful, but it is a question of time, it takes time to write a book. Ole is better at utilizing every single second - he can produce books simultaneously with his other activity. I cannot do that, so we will see what becomes possible.
Could you speak about the special role of women in the Dharma?
When you practice Buddhism, it is a very individual thing and not so much a question of whether one is a man or a woman. Each individual has his or her capacity and conditions - both outer and inner. In the West I do not see a big difference between men and women. It is more in the Eastern cultures that there is a big difference in their roles. Concerning the Buddhist methods, there is not much difference, one just has to use them. Generally, attachment is more difficult for a woman to dissolve, and men have to perhaps work more with aggressions - but it is very individual. We are all human beings, and most have a combination of disturbing emotions. So there is not that big of a difference between man and woman when it comes to practicing the Dharma.
You hold so much knowledge and wisdom from being around teachers and translating for so many years. Why don't you teach more?
There is only so much time and I am involved in many different kinds of activities already. Once one starts something, one should do it properly. I do not mind teaching, but when I am together with Ole, it is more natural that he teaches. When I am not with him, I mainly translate and organize for the Tibetan lamas. Somehow, teaching has not been part of my activity yet. Also, His Holiness was very specific about the importance of Ole and I working together, and, if I was to have an independent teaching program on top of everything else, the already too little time we spend together would be reduced to zero.
Every teacher has a different style. This can be difficult since we tend to prefer one style and disapprove of others.
The confusion may have to do with us not distinguishing between the different kinds of teachers and thinking the teacher must be a Rinpoche or somebody very well known before we bother to listen to his teachings. One's main teacher will naturally be somebody one likes and in whom one has confidence. It is psychologically normal to learn better and be more attentive if the teacher has a style one feels at home with. But we don't always seem to understand that we need to study the basic teachings in order to understand and practice the path in a correct way. For this we can listen to teachers who are not necessarily enlightened or especially charismatic. In such cases the main thing is that he knows what he is talking about. If we focus more on the Dharma than on the person teaching it, we also protect ourselves against spiritual manipulation, which is good for everybody.

*********************

An Interview with Jhampa Shaneman: Buddhist Astrologer
by Lura McCallum
Jhampa Shaneman is featured at as our Buddhist astrologer. Every month he posts his analysis for our customers on our website. Click Here to read this month's edition. He has studied with many outstanding teachers and completed a traditional three-year retreat. You can also see his website at www.buddhist-astrology.com.

LURA MCCALLUM: Jhampa, you spent 14 years in India studying Tibetan Buddhism, can you tell us something about this time.
JHAMPA SHANEMAN: In 1970 I decided to travel and see the world. My journey ended up in India with the Tibetan Buddhists. I was excited with Tibetan Buddhism and what it had to offer. By 1971, I became an ordained monk and started to learn the Tibetan language. There were no English texts and very few translators available at that time.
LURA: Who were some of the teachers you studied with?
JHAMPA: My first teacher was Lama Thubten Yeshe. He was inspiring and gave me a powerful image of what it meant to be a Buddhist. As time passed I studied with his teachers, such as Geshe Rabten and Geshe Dhargaye. By my third year in India I became a pupil of the senior tutor of H.H. the Dalai Lama, Ling Rinpoche, and continued studying with him as my principal teacher until 1984. I also studied other Buddhist practices and with other lineages as the years passed. I have now translated and become familiar with all four sects of Tibetan Buddhism.
LURA: What happened after 1984?
JHAMPA: I completed a three year retreat in 1983, the same year Kyabje Ling Rinpoche died. This motivated me to return to Canada as I had lived in India for 14 years. I started a small meditation center, Thubten Choling. I had studied astrology in India and so in Canada I started to read astrology charts as a source of income. It was difficult to make ends meet so I also worked in a hardware store after a few years. I continued teaching Buddhism at the meditation center and my astrology practice while working a regular job.
LURA: You have always referred to yourself as a Buddhist astrologer, in what way is this different?
JHAMPA: After studying Western astrology while in India my teacher asked me to read his astrology chart. He was curious to see how well I had grasped the information. I surprised him with my interpretation style because it was different. I presented a Buddhist approach to the material. I used the Buddhist philosophy and practices I had gathered over my years of study. This made me realize that there was a new and unique way of presenting astrology. In Canada, I have always used a Buddhist approach to reading astrological charts. I find it practical when tied with meditation practices.
LURA: How can a Buddhist astrology chart benefit me?
JHAMPA: One does not have to be a Buddhist to benefit from the reading. The emphasis of Buddhism is with awareness and wisdom. Everyone has some level of self-awareness and wisdom. That awareness and wisdom dictate how one experiences their life. When you consider astrology, you apply mindful awareness and wisdom to astrological influences. There are different schools of astrological thought. Some believe life is predetermined and others support an expression of choice and free will. I fall into the category that emphasizes astrology is a circumstance of influence. Astrology effects everyone's life, but an individual's awareness and wisdom can filter those influences. The Buddhist interpretation would benefit anyone because it emphasizes key astrological points from a practical perspective.
A Buddhist could harmonize their practice with the interpretation. The awareness of astrological transits, cycles and fluctuations better prepare one to utilize their innate wisdom. The greater the scope of awareness generated, the better one handles life. Important decisions are made without unconscious factors of influence. One can see on a personal level the dynamics of planetary interdependence.
LURA: Can Buddhist astrology shed light on ego and the manifestation of suffering?
JHAMPA: Yes. First there is the natal chart interpretation showing the dynamics that influence the formation of personality. These dynamics mix with factors like the birth family, education and social economic influence. Astrology is just one of the factors that create the individual. The chart's interpretation continues with major astrological transits from client's birth to their present age. That section often invokes surprise. People see how interdependent they are with the world around them. Astrology plays a role in that interdependence.
A Buddhist understands samsara is fraught with uncertainty. Suffering is created by delusion. Astrological influences can increase the impact of delusion. Awareness of upcoming influences can prepare one. Specific meditation techniques can be implemented to balance things. Foreknowledge can diminish the disruptive effects of astrologically stressful times.
Another component is to utilize this foreknowledge in a positive way. One could deepen their practice and understanding with beneficial astrological transits. The planetary influence can augment love, compassion or insightfulness. Certain planets stimulate these feelings and wisdom. It all depends on the attitude and awareness of the individual.
LURA: If one has major suffering in their life, can you direct them?
JHAMPA: I am frank with my clients and if there are astrological inclinations affecting the personality that cause suffering, then I would explain these clearly. I would give strategies and Buddhist options that help one understand the causes of suffering. For example, if someone has a difficult time with their feelings, which could involve a transit of Uranus to Venus, then I identify areas that require conscious awareness. I give positive feedback to compensate for the transit and Buddhist tools to enhance the natal chart strengths.
LURA: Can you explain what astrology the Tibetans study?
JHAMPA: Tibetan astrology has a seven year program which is connected to the medical college. Tibetan astrology is based on two systems, the Indian and the Chinese. There is also an indigenous form used by farmers. They read the chart incorporating a compilation of these traditions. The Tibetan tradition connects the health and well being of any individual to their astrological chart. A doctor knows astrological transits to a certain degree and can access individuals in the medical college who are trained to read full charts. There were references made to astrology during my studies, but I focused on the meditative techniques and philosophy of Mahayana Buddhism. I was drawn to the Western astrology as it was not such a long program and more accessible to me. The western presentation is natural to our mind and attitude.
LURA: You have just completed your first book, which is based on Buddhist astrology. I believe you are working on a second book based on your interpretation of a world renowned Buddhist's astrological chart.
JHAMPA: Yes, I requested H.H. the Dalai Lama to be the principal subject for the second book. I will focus on a Buddhist interpretation of his astrological chart and the transits during his life. My interview was interesting because H.H. confided that he does not believe in Tibetan astrology. This came as a surprise to me. I explained I was working from western astrological principles and interpreting the material from a Buddhist point of view. H.H. knows his Tibetan chart and can compare it to the western style. H.H. has agreed to this and will even write the foreword for the book.
LURA: Can a Buddhist astrology chart strengthen my Buddhist practice?
JHAMPA: Yes, because we are interdependent with the world around us. H.H. says we should look at the planet earth as our home and that we are interdependent with this world. It opens our mind to the greater scope of our life and the influences that effect us. A study of astrology is a study of interdependence and that increases wisdom.
The planets, which relate to being a bodhisattva, a person wanting to work for the benefit all sentient beings, are the Moon and Jupiter. The Moon relates to compassion, receptivity and empathy, while Jupiter can inspire optimism, enthusiasm and a positive attitude towards life. Bodhicitta is an attitude that develops the ability to see the positive possibilities of all beings. It is a desire to benefit others.
Saturn relates to wisdom. A Saturn placement in the third or ninth house would be excellent for the development of wisdom. It broods and reflects on issues and this generates wisdom. Saturn is often called the planet of hard knocks. We learn valuable lessons from difficult times in our lives. Saturn can help develop our insight and the practice of guru yoga. An understanding of one's chart can assist enhancing personal awareness.
LURA: On a final note, where do you see yourself in the future?
JHAMPA: As long as I am capable I would like to continue teaching Buddhism and doing astrology readings to help direct people to a better understanding of their own nature and the influences that affect them. I give lectures for psychologists regularly and I think Buddhism, astrology and psychology all have ground for an inter-disciplinary dialogue.

*********************

An Interview with Zenmar
Tom Armstrong
In the electronic sangha of America Online, the voice of Zenmar is distinctive and his knowledge is honored. It is unusual for him to visit a Buddhist chatroom, but when he does, attention turns to him as an arbiter of theory and practice. On the BBS he frequently posts to the more-serious folders where his erudite comments clear the air or spark high-level discussion. Only a rare few though can stay toe-to-toe in discourse of high scholarship with Zenmar though. A part of his genius too, is exhibited in a pithy, sometimes-disarming humor that leaves the reader chuckling as well as challenged. Zenmar agreed to an interview using a format of exchanged E-mail, thus allowing him to give full vent to his thoughts, and allowing me to find the right follow-up question where appropriate.

Snarkyman: Talk to me a little about your development as a Zen Buddhist. Those of us who subscribe to your newsletter or have conversed with you in an E-mail discussion group have heard a little about your days as a Zen monk-I believe in Japan. Are those the "good old days," that you look back on reverently, when you had Masters whom you loved and who were near-perfect beings?
Zenmar: I think my development as a Zen Buddhist began when my philosophy professor, Dr. Lauren, introduced me to Zen Buddhism in 1964. He knew DT Suzuki quite well and studied with him as a scholar at the University of Hawaii. Incidentally, my teacher had also been a student of Martin Heidegger who had once remarked that Suzuki's essays said what he was trying to say all along! Talk about a circle. Anyway, I thank my philosophy teacher for starting me on the path. About a year later I found a Zen teacher who lived by the College of the Pacific and had taught Alan Watts a little Zen when Mr. Watts lectured at the American Academy of Asian Studies at COP around 1955 (I can't remember the exact date). At that time, I felt I was in good hands and had no need to book passage on a cattle boat to Japan! I felt that studying Zen in Japan, then trying to learn Japanese at the same time, would make me a lousy Zennist and a poor Japanese speaker. After I met my teacher, who was named Daino Doki, he convinced me that he was up to the task of training me. I had no reason to doubt him at the time. Daino Doki was a Caucasian who had studied with Hodo Tobase in San Francisco and then a number of years later was elevated to the rank of Roshi by the Primate of Soto Zen, Rosen Takashina. As for the "good old days," I must confess that some of it was fun for a while. At least I can say that my tenure with Daino Doki wasn't boring. He was quite a creative character. In his temple, the word "dullness" never came up. I found the whole Zen environment to be quite exciting. I must tell you that life there was unpredictable. Several times, I got kicked out, and just as many times I came back, which was part of the testing process. To enumerate some things I did, I sat in zazen, chanted the Heart Sutra, cleaned the garden, walked the dog, made coffee for my teacher, mowed the lawn, and made my own robes. Looking back, I think I was more of a holy houseboy. Supposedly, doing chores is a vehicle through which to practice "awareness," which for Soto Zen seems to be its main secret. There wasn't a day that went by when my teacher didn't say "Be aware!" I must admit that practicing awareness sobered me up. By paying attention to the little things, soon I saw that the big things take care of themselves. As for the last part of your question, like most beginners new to Buddhism, I was a hopeless romantic and believed in my teacher-even when he told me he was the Tathagata! However, I soon came to learn that Buddhist teachers are not "near-perfect beings"-including my teacher. Many teachers, I found out later, are very human. And some I have seen will do almost anything for money and fame. In hindsight, I learned that it is very important to stop focusing on the personality of your teacher and pay attention to the content of the teaching. Many students I have noticed today, still must learn the hard way as I did; that the worship of a teacher corrupts the Dharma. It doesn't matter if the teacher is a Tibetan Lama or a Zen Buddhist Roshi-all teachers are all-too-human, to use Nietsche's expression, and shouldn't be worshipped. The essential aim of a good teacher is to get his or her students on a path of philosophical intuition which will eventually lead to a reality that surpasses this finite world. Now, I should point out that philosophical intuition is not to be confused with interpretive intuition, nor should it be confused with having so-called "religious peak experiences" which are often quite meaningless. In Buddhism, philosophical intuition is synonymous with remembrance, of trying to wake up and recall our Buddha-nature, which has never left us. All the rest is humbug. I know this will, no doubt, offend many Buddhists. But seriously, anyone who is familiar with the Buddhist canon as a whole, has to see the Buddha as a wise old philosopher who just happened to recall the secret of the universe, which is wholly detached from the cycles of samsaric regeneration.
Snarkyman: Does training to be a monk have many similarities to being in the Army? The reputation that is "put out there" is one of precise organization and that it is very formal. So, what was it like for you?
Zenmar: Rather than being like the Army, in my case, life in a Zen temple with a Zen master was a combination of being in a Trappist monastery, a college, and a mild prison boot camp. Such an environment is heavy on formality and weak on content. I remember when Bishop Sumi came from Japan to inspect our temple; he made sure that we had the right kind of bell and that our shrine was the correct height. He couldn't have cared less about our library and whether or not we had the Buddhist canon in it. As far as my expectations of learning the content of Zen went, my whole stay in the Zen temple eventually became a big let down. I was expecting Zen to be like T'ang Zen in China, and my teacher to be like Rinzai or Joshu. That was not the case.
Snarkyman: Are you saying that you didn't have any kind of "satori" or some other kind of earth shattering experience while you trained with your teacher?
Zenmar: First, let me say something shocking about the Zen world. If your teacher likes you, you seem to answer all the koans and receive the seal of approval. If not, you figure it out. That I wasn't really liked by my teacher, had something to do with not having satori, as funny as that sounds. My teacher's favorite disciple, for some strange reason, answered all the koans, while I couldn't get to first base. I began to have doubts when my teacher give me a book to read entitled The Temple of the Golden Pavilion by Yukio Mishima. I think my teacher saw me as a kind of stuttering Mizoguchi who tried to destroy the Golden Temple in Mishima's book. After the big hint, I took another course. I decided to read every book in the library and become smarter than my teacher. I can remember when my teacher and his favorite disciple went on vacation and left me alone to take care of the temple. For a week, I had an orgy reading the Pali Suttas and the Mahayana Lankavatara Sutra! I should mention that my teacher would only let me read certain books, ones that I thought were rather inane. To make a long story short, many years later I had a profound satori. When I visited my teacher afterwards, I could see that his depth of penetration into the Dharma was quite shallow. By that time, I saw that authentic Zen was not the same as institutional Zen.
Snarkyman: What is the meaning of the name "Zenmar?"
Zenmar: I just combined the Greek god Zeus, which can be written as "Zen", with the Roman god Mars and dropped the "s." I wanted six letters for my AOL name and something quite odd that I could remember. On the whole it is rather barbaric and unBuddhist, but it has a nice ring to it. Actually, the name Zenmar is better suited to a comic book character, flying around with a huge cape! On a more serious note, I wanted Zenmar to be a cybernetic creation. I have given him a personality and a set of manners with which to deal with Buddhists in the cybersangha. This way, I don't have to become personal with anyone, inasmuch as Zenmar is fictional. Speaking for myself, I do enjoy the character "Zenmar." I should add this: I believe it is more important to understand a person's ideas rather than knowing about their peccadilloes. American Buddhists are far too concerned about the "person," which eventually turns into a cult of personality. I think it is time that we pay more attention to the teaching and less to who teaches it.
Snarkyman: A dumb question, no doubt, but once the dust-free mirror reflects back to us that most-original of all Original Faces and we've taken The Great Swandive into the timeless, spaceless ocean, will we know what the hell this crazy passion-play called "life" was all about? Or will we "merely" transcend the question?
Zenmar: In a way, the question becomes transcended. It is like a dream in which everything seems important at that moment, until you awaken from the sleep of ignorance and look back. Speaking about the "this crazy passion-play called 'life'," eventually we come to learn the cause of the passion-play. We understand that Mind, in the past, attached itself to its productions, setting the stage for its own tragic downfall into corporeal existence. Said another way, the human passion-play has its genesis in Mind's sympathy with its phenomenal side, rather than with its "suchness."
Snarkyman: Should we feel an "urgency" in our quest for enlightenment?
Zenmar: To answer your question, I would say emphatically, yes! We need to feel a sense of urgency towards the quest. The reason, I think, why we don't feel an "urgency" is because we are infatuated with the sensory-world which veils the truth and clouds our higher reason. But we must remember that such a world ends worse than it begins, according to the Buddha.
Snarkyman: Most of us American gum-chewing Buddhists who tune in to cyberspace will never be devoting the time and effort that you have. Perhaps we are simply not doing "our best." What is the role of the ordinary man? Can it be that a person is born into a lifetime where the necessary "ripeness" for finding his Buddha-nature is never going to come?
Zenmar: Let me put your question this way: Would gum-chewing ordinary American Buddhists study the Dharma if it were interesting and made perfect sense? I think the answer is yes. So far, Buddhist teachers have not clearly articulated what Buddhism is actually about. They have a lot of half-baked theories about Buddhism that are frankly quite dull and just plain humbug. I can't blame people for not doing their level best, given some of the Buddhism that I have seen over the years. The reason why Buddhism engaged the Orient in the past and caused great cultures to honor it with splendid works of art and literature, was that Buddhism was seen as a path that ennobled humanity and taught how to break the bonds of death and samsaric regeneration in a very concrete way. The Buddha actually laid out a path that teaches us the process of disembodiment; of how to cast off this mortal body, while still being able to generate it-a kind of divine "having your cake and eating it too" solution. It is interesting to see how much the word "Nirvana" still remains a riddle for most Westerners; when in fact it is just the accomplished state of disembodiment. Now, to answer the last part of your question, there is no cause for doom. If you would like to not worry about old age, suffering, and death ever again, just bear in mind that you are disembodied right now, but can't remember in what way you are free of these horrid conditions. Nothing is stopping you from realizing what the Buddha realized, except your attachment to the corporeal body. You are quite ripe now, by the way.
Snarkyman: I don't think I have ever heard it described that Nirvana is like disembodiment-that is strange. Isn't Nirvana just the extinction of the ego? Is there any mention of disembodiment in Zen?
Zenmar: I think the idea of "ego-extinction" is a modern invention. Frankly, I don't see our modern concept of egoity in Buddhism. The Buddha wasn't Freud. Nirvana really has nothing to do with the extinction of the ego. Above all, we should not understand Nirvana to be synonymous with extinction, as if to blow a flame out. Even though it is mentioned in this context once or twice, it is only an analogy used by the Buddha to describe Nirvana's indeterminate character. Supposedly, an adept when released from the corporeal body, is beyond all measure, just as the whereabouts of a flame, blown out by a gust of wind, cannot be determined. At least, that is the way I read it, in the Pali Suttas. I know this is going to sound weird, but the Greek word "utopia" is a perfect cognate for Nirvana. What does "utopia" mean? It means literally "no place"; the term "place" in Greek being "at position" or "locus." Remember, that in the scripture, the Buddha spoke of Nirvana as being a place where there is no earth or water, nor wind or space. I can't remember most of the passage, but the Buddha goes on and says that Nirvana is even beyond consciousness and any kind of dependence. Nirvana, you could say, is kind of a placeless place that is free of suffering. As to your last question-yes, disembodiment is mentioned in Zen. Rinzai does a good job using the metaphor of the "unfixed true man" for the state of disembodiment. This "man" he says, is not made of the elements and is constantly going out and coming in through our foreheads! Anyway, if you read his sermon, you can't fail to get the meaning.
Snarkyman: Are Lao Tzu's writings of value? Is not the Tao the same as the One Mind, that ocean, beneath our waves? Can we move forward on our path to locate our Buddha-nature by being Taoist?
Zenmar: I think his writings are of great value. I might add that there is a lot of wiggle room to interpret Taoism in a Buddhist way and vice versa. When Buddhism came to China, it had to be interpreted in the context of Taoism. It would be like Westerners interpreting Buddhism in a Neoplatonic context, when it first arrived on our shores. In fact, Buddhism would be a lot better off if Platonists had translated Buddhist Sutras. Now, we are stuck with translations that reflect a postmodern agenda. Taoism, I must say, didn't stop any Chinese Buddhists from realizing their Buddha-nature.
Snarkyman: Is "being in love" with Buddhism an aid to locating our Buddha-nature?
Zenmar: Yes-very much. You have to really love Buddhism in order to understand Buddha-nature. You can't practice out of fear, or just use Buddhism to make a name for yourself. You have to be in love with it. Every day, you should delight in your study of the Dharma. I find that when I am really interested in something I learn quicker. It is amazing how fast students can progress in Buddhism, when they are really in love with it. Instead of many lifetimes trying to find the Buddha-nature, if you are enthusiastic, you can realize it in thirty or forty years-maybe.
Snarkyman: Thirty or forty years! I have heard that in Zen it only takes a very short time-maybe a few years or even a few days to realize your Buddha-nature. Is that not true?
Zenmar: Mostly, that is hogwash. Take the example of Zen master Hakuin. Sure he had a lot of insights that he imagined were enlightenment, but in the end, he had to acknowledge that they were misleading. In my reading of his biography, he reached profound enlightenment only in his forty-second year. That is along tenure. I am willing to bet that his last enlightenment almost came as a let down! He simply realized that the Buddha-nature that he was searching for, was the very same one that was conducting the search for enlightenment all those years! Maybe this is why it takes so long with the truth, because we are dealing with an amazing subtlety. As an old Zen expression goes: "It is like riding an ass in search of an ass."
Snarkyman: When you post to the boards is there always a "purpose" at play to enlighten the other person with what you write?
Zenmar: I think the main purpose why I post, is to exchange ideas and to have dialogues with other Buddhists that are enriching to everyone concerned. Of course, reaching that goal is not easy. Many people don't want to exchange ideas or have a dialogue. The vast majority of those that post are just snipers. If you write more than a sentence-you are too intellectual. If you speak metaphysically, rather than dogmatically, someone tells you that you talk too much and that you need to do more zazen! All this is a waste of time, to my way of thinking. Generally the sniping reflects the postmodern view, to which most Buddhists subscribe, mainly that Buddhism should be unintelligible. D.T. Suzuki, I feel, was responsible for casting Zen into a postmodern mold. I think his understanding of Zen was dead wrong on a few points-but he was a man of his times; so what could he do?
Snarkyman: I have read a little Suzuki like most people. I have always found him interesting. He seems to know what he is talking about. Since you mentioned that he was dead wrong on some points, could you give an example?
Zenmar: The one that sticks in my mind is his idea of "emptiness." Suzuki thinks that is it some kind of Buddhist absolute that we are to experience, in some mystical way, when we get rid of all of our concepts and logic. As a result of this purging-voila!-emptiness is experienced! But is that emptiness or just an abstraction? I think the answer is clear. This kind of emptiness is only an empty standpoint, from what I can see. It is just as illusory as that which is negated. To be honest with you, I don't find an iota of evidence in the Buddhist canon which supports Suzuki's understanding of emptiness. For the Buddha, emptiness is the unreal, rather than the real. For Nagarjuna, sometimes called the second Buddha, emptiness is like an echo, which can deceive. In fact, Nagarjuna equated emptiness with dependent origination. What he is saying is that things derived by causation are empty and impotent!
Snarkyman: What are some of your insights about the cybersangha that is developing?
Zenmar: An evolving cybersangha is good for Buddhism. But not because it is providing practitioners with a lot of texts. It is good for Buddhism because, as Buddhists, we can exchange our ideas with other Buddhists, seeing which ideas make more sense and which ideas are mature or contradictory. I have found that what comes across best are solid ideas. A post, well conceived and well written, has a strong impact. On the other hand, blather gets nowhere. Nor does the typical flippant zenic "one word" response, as when some yokel writes "Kwatz." In the end, these people have to leave because their ideas are hollow. I have to admit that a lot of garbage is posted. But every once in a while, good stuff appears. Those who take a little time to say something of interest; who have something valuable to share with others and work at it, help the cybersangha overall. One more thing. Recently, I have noticed an important development in the cybersangha and, in particular, Buddhism AOL. It is a sense of community. Other media, like television, just focus on one person with a narrow-minded core of opinions. Contrary to this, on the BBS, there is a living community, with no single individual dominating the dialogue. It is open and free. Ideas, I must say, thrive in such an environment.
Snarkyman: Do you think Buddhism will change as a result of the BBS? Will practice be as necessary as it has been?
Zenmar: The BBS cannot change Buddhism's content. The BBS can only increase the velocity of information transfer. That is, it can do away with information lag that is inherent in a slower medium, such as a book or a journal. Most of us, for instance, learned of Taizan Maezumi Roshi's death very quickly. However, I should emphasize that the swiftness of the so-called information highway does not generate wisdom. And if nothing is utilizing this highway except high-speed kitsch, then we can expect nothing great from such a highway. Overall, we can expect the BBS to be of great benefit, giving our ideas both access and speed by which to engage other ideas. Scholars can exchange information and ideas and banter on at a higher velocity.

*********************

Zen Master Thich Nhat Hanh was born and brought up in Central Vietnam, a Buddhist monk, poet, scholar and peace activist. He was the founder of Van Hanh University in Saigon and has taught at Columbia University in America and the Sorbonne in France. Many people in the West know him through his books and public talks around the world and through retreats in Plum Village, France where he currently resides. This Zen master who is 76 years old talked about his religion as engaged, contemporary Buddhism, Buddhism that can be applied in daily life.
In May 2001, Reporter Nguyen Giang interviewed Zen master Thich Nhat Hanh, as he was in London, UK for a teaching tour. This interview was broadcast via the Vietnamese BBC Radio section: 'THOUGHTS OF THE CENTURY'.

Nguyen Giang (NG): As a Buddhist monk and as a Vietnamese what do you think of the issues relating to the environment and the violence in modern society and the whole question of globalisation which has spread to Vietnam. Could your answer be simply: 'Each step will bring peace' or might it be something else?
Zen master Thich Nhat Hanh (TNH): I see that the sufferings which people carry have been lightened a lot as people have a certain capacity to be awake and people also want to do something to stop the spread of violence. In meditation, we learn to look deeply to understand the root cause of violence in order to find the solution to transform the situation and to prevent violence. We will not resolve the problem if we only touch the surface of it. For example, if parents do not know how to live in harmony, without happiness or love, and continue to make each other suffer daily, then no doubt, the parents' suffering and violence will seep into the minds of their children. In school, we are not taught how to recognise, embrace and transform violence, and at home, we do not learn the techniques either, and that is why violence keeps spreading.
NG: Dear Thay, do you expect everyone to work every second, every minute for peace and against violence? Why should we need to concentrate our minds at every moment like that? Where does our real life lie?
TNH: Our true life lies in every second. For example: parents need to be awake, clearly to recognise that if they do not treat each other with love, with compassionate speech and with deep listening, their relationship will not be good, they themselves will suffer and their children will continue to suffer. Therefore, couples should practise together. The wife can say to her husband: "Dear, from now on, I will only use loving language, I will not water the seeds of suffering, anger, resentment or violence in you. And please would you treat me in the same way, as there are seeds of suffering, violence and hatred in me. Be careful not to water the negative seeds and only water the seeds of compassion in me with kind words so that our conversation will be peaceful and our life joyful and happy. As a result, our children will be influenced positively". In every home, the parents can adopt such an attitude straight away.
In Buddhism, there are many "dharma practices" that can be applied right away at this very moment and the practice should continue every day. This is not rhetoric. There must be - There are - solutions and ways to live. Every time our monastic community travels the world to offer teaching, retreats of 5, 7 or 21 days, we only teach how to recognise the seeds of violence, the seeds of suffering in us, how to embrace and transform them; how to recognise the seeds of love, compassion, understanding and wisdom in us and how to learn to water them daily so that our lives will have a better quality. Couples can help each other water the seeds of love, generosity and happiness that already exist in both partners. We know that one only needs to practise for a few days; people will notice the transformation in us when we smile or when we use kind words because the good seeds in us are being watered and manifested.
NG: However that is, people still have desires such as wanting to advance oneself in society, to earn a lot of money and to possess things that they themselves regard as good and beneficial. Please, comment.
TNH: We have to remind ourselves that true happiness comes when we have true love, true compassion, true generosity and true inclusiveness. Therefore, we must reflect, re-examine our idea of happiness. And as we awaken we will realise that we will find more happiness if we can live together with compassion and in harmony. It is all right to drive a small car, live in a small house but we can look, smile, have time for and love each other. That can be done, however it requires us to wake up, to be mindful.
NG: You often talk about the search for happiness. Why should we search for happiness?
TNH: Happiness is primarily the absence of suffering. When we have no enmity, hatred or violence in ourselves, we will immediately have happiness and peace, and at that moment, we will truly perceive that we are in touch with the miracle of life such as the blue sky, the white clouds, the bird singing, the waving pine branches…etc…The elements nurture us, help us to smile and we can then bring happiness to the people around us. Otherwise, we would be focussed on our anger, violence and hatred and obviously we will cause pain and suffering to the people around us.
NG: Dear Thay, searching for happiness is a "desire", and according to my Buddhist understanding, "desire" is perhaps not a good thing.
TNH: There are many kinds of "desire". For example, your desire to help people is not an unwholesome desire. The historical Buddha also desired to help people, so he was ordained, practised and when enlightened, he travelled and taught everywhere for 45 years to help people. That is a good desire. We only need to transform negative desire; we do not want to transform positive desire. The desire for wisdom and love and for bringing happiness to all living beings is necessary and is what Shakyamuni Buddha taught and what he manifested by the way he lived.
In Buddhism, awakening or mindfulness is a basic practice because in Buddhism, the term 'Bud' means 'awakening' and the word Buddha means 'the person who awakes' and Buddhism is 'the path of awakening'. During my teaching travels around the world, I notice that many young Western people are "awakened"; they live a simpler life in order to have more time to live and to love. There are more and more people, also young people, intellectuals...etc… beginning to eat less meat because they know that eating meat does not make them healthier, to put it in another way, they do not get weak having a diet without meat. We can replace meat with tofu, Soya products and vegetables. Presently, all supermarkets in America and Europe do sell food products made of Soya. Today, almost all flights ban smoking with the sign 'Non smoking flight' and on cigarette packets, there is always a notice "Precaution: cigarettes may cause harm to your health". We believe that these are "awakenings". There are people, many people who are already awakened and who find ways to live healthily so that the earth will remain green and pure for our children to enjoy. I think that the ability to awaken is there and developing. We should not feel too pessimistic. We must learn to see the positive and to make our contribution helping people wake up and stop the production and consumerism of the products that nourish anger, hatred and greed in ourselves, especially in the young people.
……………………………………………
NG: Is there a contradiction between your Buddhist practice and the computer?
TNH: In Plum Village we have really made intelligent use of the computer. Every 15 minutes, there is a bell sound from the computer and we stop to consciously follow our breath and we know that we have not got lost in our work or project. A computer is only an instrument and we should not allow ourselves to become a victim of it but to remain our own boss.
………………………………………..
NG: Just one further step and we will get to the point of industrialised genetics, which means that artificial intelligence could be developed to control the human race. You said that you accept modernisation and progress. In that case, is there any contradiction between industrialised genetics and the Buddhist precepts that you teach?
TNH: We have intelligence and wisdom; we should use our wisdom to shine light on the problems. The problems require time for experiments to be carried out (and the same goes for our Buddhist training and practice). For example, we need more time to study and carry out experiments on the food grains grown by genetically modified methods to see if there are any negative effects on our body when using them.
NG: That means you do not object to GM foods?
TNH: No, because it would only show our ignorance if we make objection without knowing much about the problem; that cannot be called wisdom. However, after thorough experimentation, we can be certain that the new methods would not cause harm to our body and mind, then we can accept the validity of their use.
………………………………………………….
NG: The points you just mentioned, about the lack of environmental awareness and consumerism, and the harm this causes to us and generally have been talked about by many people and organisations, including state governments. They all state that their policies are to protect the environment, human rights and democracy. So, how far will your message travel?
TNH: I think that if the Vietnamese Government, for example, knew how to apply the strength of spirituality, they would easily lead the country out of the present difficult period. I believe they know very well that there is serious corruption within the Party and the Government. We can say that the fuel, which runs the Party and the Government machine, is fear, corruption and suspicion. There is a fear of being thrown out of office, fear of harassment or worse. There is much evidence to show their fear and suspicion. Their imagination takes over.
NG: Do you believe that Buddhism is more powerful than Marxism?
TNH: Buddhism is a very inclusive religion. Everywhere Buddhism goes, it accepts the country's religion, beliefs and culture. Buddhism has never fought against anyone. In the past, I have written much on "Buddhism and Marxism"; from these texts, we can learn many things about Marxism and which elements of Marxism can enrich Vietnamese culture, because Vietnamese culture has the capacity to tolerate, conciliate, harmonise and adapt very well. We had already brought together in harmony Buddhism, Taoism, Confucius as well as many other cultures. We did have the capacity to adapt elements from the sources to serve our people. Why do we not take some elements from Marxist theory to enhance our rich country heritage? Our spirit of harmony does not just come to us today but it has been there over the centuries.
NG: Do you worry that the Vietnamese culture is declining and unable now to overcome the modern globalisation process - if we see globalisation in a negative light?
TNH: I do not think that we lack the strength for this. If you go to the Vietnamese countryside, you will still see people showing the spirit of Vietnamese culture. Just look at the way they live, the way they think; we know that traditional Vietnamese culture is there.
NG: In many charitable assignments that we carried out in Vietnam, we noticed that the young people over there do not bother about anything at all except smoking and pursuing leisure activities. There are many villages built in brick and mortar, without bamboo, very little or no green trees. Can you tell us the reason behind this?
TNH: If you come to Plum Village in France, you will meet the monks and nuns, most of them young with University degrees, doctors, scientists. If they stayed in the world with such qualifications, they would have possibilities to develop a most comfortable life. So, why did they choose a monastic life and come to study in the Plum Village community? Because they know that they will have more happiness on this path and they can help more people. It is the same for Vietnam; the young people over there lead a fast life and run after money because they have not found a healthy and beautiful path and they are not shown a good path to follow. Meanwhile the Vietnamese government does not allow us to return home to offer teaching to the young people, to show them the right path to follow. The government only allows us to come back to perform traditional rituals and ceremonies. The government cannot drop their psychological fear and does not have enough wisdom to see that we are their allies, helping them and the Vietnamese people spiritually.
Unfortunately, their psychological fear is just too great and this stops them from seeing the truth. They can see that we have motivated millions of young people in the West to come to our retreats and to take the five precepts; our books have reached countless Western readers. Can they not see that we have the ability to speak to and reach the young Vietnamese, to guide them on the spiritual path and help them return to the culture of the country? This is extremely important, because without spiritual elements and national culture, economic development will destroy the entire country.
NG: On behalf of the BBC radio listeners, we thank you, Thay.

*********************

Buddhism and Animal Rights - an interview with Dr Tony Page
Buddhism is world-famous for its philosophy of compassion towards all people. Yet what precisely do its scriptures teach on the subject of humanity's rightful relationship with the animal kingdom? Dr Tony Page recently wrote a book on that very subject calling it "Buddhism and Animals".
Interview by Claudette Vaughan, August 2000.

CLAUDETTE: How difficult was it to carry out research into Buddhist morality and the practice of flesh-eating?
TONY: It was surprisingly easy. I have been a student of Buddhism for 20 years but have always been disturbed by the number of modern Buddhists who ate meat, as well as by the relative neglect of animal rights as an issue with Buddhist groups in the West. I have always understood that, given its strong principle of "ahimsa" or non-violence, Buddhism frowned upon meat-eating, since meat-eating inevitably meant doing violence to animals that were slaughtered for food.
But I found that a lot of present day Buddhist teachers and practitioners were actually trying to justify meat-eating. Clearly, something was wrong. So I decided to see what the scriptural basis for meat-eating was. I resolved largely to by-pass what later commentators on Buddhism had said on the subject (many were meat-eaters) and go back to the original scriptures to see what the Buddha himself had stated. It is always best to go back to the source, as far as possible. And I was tremendously encouraged to see that there was a wealth of evidence showing that Buddha Shakyamundi was himself against the eating of meat and was in fact a strong advocate of vegetarianism and compassion towards animals. This formed the basis of my book.
CLAUDETTE: Isn't there a difference regarding the question of meat-eating between the two big schools of Buddhism, "Theravada" and "Mahayama"?
TONY: Yes, seemingly so. The Pali scriptures of the Theravada school report the Buddha as having died from eating some rotten pork at the end of his life, and also claim that he said it was OK to eat flesh as long as you yourself have not seen, heard, or suspected that the animal was killed especially for you. On the first point, when one investigates the Pali work "sukara-maddava" - translated by meat eaters as pork - the evidence suggests that it actually means "pig's delight", ie. a type of food favoured by pigs, probably truffles, rather than pig's meat.
On the second point, if one reads the relevant Pali scripture carefully, one sees that the phrase "killed especially for oneself" is not used by the Buddha. It is interpolated (in parentheses) by later commentators. All the Buddha says is that meat might not be eaten if it is seen, heard, or suspected, it may be used. It seems very clear to me that what this means is that any meat put into a Buddhist monk's begging bowl (along with various other food items) should not be eaten if the monk actually sees, hears from others, or suspects for himself that what has been given to him is indeed meat. If by chance he does not notice this and unrealisingly goes ahead and swallows what actually turns out to be meat (it presumably being mixed in with other pieces of food), he is not committing an offence against Buddhist morality, since he is acting unwittingly.
CLAUDETTE: What does the Mahayama school say?
TONY: Things are even clearer in these scriptures. The Buddha emphatically condemns as "twisters of truth" those people who go around saying that the Buddha allows meat eating. He says in no uncertain terms that flesh eating is incompatible with the Buddhist principle of compassion.
CLAUDETTE: You mentioned the principle of non-violence or non-harming (ahimsa) as being important in Buddhism. What does this mean in a practical everyday sense to you?
TONY: It means respecting all beings - humans and animals - as having feelings, as being sentient, and not deserving deliberately to be hurt. So a Buddhist would never swat a fly or purposefully step on an ant or spider. "Non-harming" also means that one should not work in a profession that involves harming others, for example, a butcher or soldier. The main point to remember is to try and show kindness to all creatures, including of course humans. But animals are part of it too. After all, the Buddha took birth many times as an animal - sometimes a deer, or a monkey, or a fish, or a dog etc. He knew what it was like to be an animal. He also taught that we have been animals in our past lives and in fact all the animals are related to us, quite literally. At some point in the past they have been our mothers, fathers, sisters, cousins. So if we harm animals, we are actually harming members of our own family.
CLAUDETTE: What is your understanding of karma and eating animals?
TONY: Karma is the spiritual law of justice which makes us experience the good and the bad effects of what we do to others. So, if we harm animals by killing them, eating them, or experimenting on them, we will have to suffer analogous experiences ourselves in the future - or at least have to undergo some form of suffering. Only when we ourselves go through what the animals have been through will we definitely know that hurting animals is wrong. So eventually we will develop an empathy, a belief in our kinship with all sentient beings, including animals. It is interesting to note, also, that if we are kind to animals, kindness and happiness flow back to us. The Buddha says that if you perform one act of kindness to an animal, you will be recompensed a hundredfold.
CLAUDETTE: What is the climate like now in England now with regard to animal rights?
TONY: I think it is growing more favourable. More and more people are hearing about the wrongs of meat-eating, hunting and vivisecting, and recently some major animal experimental centres have been closed down. Young people in particular are turning against animal exploitation. But the Blair Government is little better than the Conservatives when it comes to animal rights. We must continue to put pressure (non-violent of course) on the MPs.
CLAUDETTE: Tell us about your own organisation Tony?
TONY: A few years ago I set up the UK Antivivisection Information Service, which is just a very small, unsalaried organisation aimed at getting the truth out about vivisection - especially how the practice is of no medical value due to its many unpredictable physiological differences between animals and humans. I have written books on the theme, plus Buddhist books, which approach the question of animal rights from a more moral/spiritual angle. So I try to distribute this kind of information to whomever is interested in it.
CLAUDETTE: In your opinion how can we best avoid oppressing our fellow non-human creatures?
TONY: Through educating as many people as possible about the suffering that animals are unfairly subjected to, and letting people know that there is an alternative - vegetarianism, veganism and natural medicines. And we must practice non-harming in our daily lives.
We should be a living example of what we preach. It is no good, in my view, to preach Compassion and then discount human suffering, for example. Animals and humans should be viewed as equally capable of suffering, so we should care about human rights and animal rights equally. This definitely gets more respect from the public. But never should any being be sacrificed involuntarily-wise, and the dire karmic consequences of harming any being, no matter what the alleged (but deluded) motives are.
CLAUDETTE: Do you have a Utopian dream regarding animals and humans?
TONY: Yes. I dream of a world where animals are viewed as sentient and sensitive people, whose right to be free from human-enforced suffering is respected and where our only relations with animals are motivated by the wish to love and help them. At the same time, I want to see a society that respects other humans much more, too, and our IQ variants, sex and sexuality. We also need to change the education system, so we are not churning out robotic, fact-and figure-filled automata, but feelingful, caring, creative and truly human beings, in touch with what the Buddha calls our innermost Buddha-Mind of Wisdom and Compassion.
CLAUDETTE: Any final thoughts?
TONY: Just to thank you all in Australia for the great work you are doing. Whether you know it or not, by being vegetarian, or better still, vegan and gently encouraging others to support animal rights you are truly helping to bring a little bit of Paradise down to our Earth. And what could be more rewarding for us all than that?
Dr Tony Page can be contacted at:
UKAVIS Publications
PO Box 4746
London SE11 4XF

*********************

Water, if you don't stir it, becomes clear, says a Tibetan proverb. Similarly, the mind, if you don't stir it, finds peace, says Sogyal Rinpoche, the renowned Buddhist teacher

The auditorium at the Hungarian Cultural Centre in Delhi, India, is packed to capacity by a select audience-comprising mostly seekers from the Western hemisphere, with a fair sprinkling of Indians-assembled on an evening to receive what is described on the invitation as a "teaching" from the renowned Tibetan Buddhist master, Sogyal Rinpoche.

On the facing wall, behind the podium from which the master is to speak, is a set of five paintings depicting the Buddha in meditation: imparting an ambience of serenity, solemnity and tranquillity to the occasion. These paintings are by Elizabeth Brunner, Hungary-born artist who adopted this country as her home during World War II. Now ninety, she is also present to receive the wisdom.

As the expectant crowd waits, suddenly, Rinpoche, dressed in a traditional beige gown, enters and hurriedly takes his place on the podium. Sogyal Rinpoche is the author of The Tibetan Book of Living and Dying, an extraordinary work of spiritual significance, and chief of the Rigpa, an international organisation devoted to imparting Buddhist teachings. He travels extensively across the world, having begun teaching in the West in 1974, addressing audiences and holding spiritual retreats.

He begins by inviting questions from members of the audience. Someone asks: "Can the mind be controlled at all?" "We will see," says Rinpoche. After a pause, he adds: "That is kind of an Indian answer. But actually we will see. We'll try to see the question and how you develop yourself. But the answer is possible." Another asks: "What is the mind?" "Something that knows," replies Rinpoche. "But unfortunately we misuse it for grasping. Its goal is to let go of grasping, and realise its pure nature."

The Buddha's teaching, he says, is both vast-comprising 108 Tibetan volumes-and of the essence. It is described as zav gyacheva-vast and profound. "Vast is the approach of the pundit and the learned." The lamas and monks sometimes devote 13 years to studying it. "Profound is the path of the yogi." Yet the teaching can be summed up in three lines: Commit not a single unwholesome action. Cultivate a wealth of virtue. And tame this mind of ours.

He lists the Ten Unwholesome Actions. They are of body (stealing, killing and sexual misbehaviour), speech (lying, harsh words, slander and gossip) and mind (malice, avarice and wrong view). "Who decides what is unwholesome?" someone wants to know. "Your mind," he responds, amidst laughter, adding: "They are considered unwholesome because they are the cause of suffering, dukka, of oneself and others." To abandon the unwholesome acts, and adopt the wholesome ones is, then, dhamma.

It is motivation that provides the key. "In a sense, it doesn't really matter what you do, if your motivation is correct," he says, with perspicacity. At the root of all human phenomena is the mind. "Seek not to cut the root of phenomena, but to cut the root of the mind." He explains: "If you have to cross a field covered with thorns, and you try to cover the field with leather, you won't succeed. It is far simpler to cover your feet with leather."

Rinpoche, born and brought up in Tibet, was taught by some of the great masters and lamas of the Buddhist tradition, in particular, the late Jamyang Khyentse. With the Chinese occupation, he took exile. In 1971, he went to Cambridge University to study comparative religion. Since he began teaching, he has become increasingly popular among seekers in Europe, the USA, Australia and Asia.

His work, The Tibetan Book of Living and Dying is a modern spiritual classic, focusing on the reality of death which becomes a celebration and search for the very ground of life. With simplicity and insight, it blends personal experience, ancient wisdom and recent findings on death and dying, to inspire a "quiet revolution in the whole way we look at life and the whole way we look at death."

An excerpt: "Perhaps the deepest reason why we are afraid of death is because we do not know who we are. We believe in a personal, unique, and separate identity; but if we dare to examine it, we find that this identity depends entirely on an endless collection of things to prop it up: our name, our 'biography', our partners, family, home, job, friends, credit cards... It is on their fragile and transient support that we rely for our security. So when they are all taken away, will we have any idea of who we really are?

"Without our familiar props, we are faced with just ourselves, a person who we do not know, an unnerving stranger with whom we have been living all the time but we never really wanted to meet. Isn't that why we have tried to fill every moment of time with noise and activity, however boring or trivial, to ensure that we are never left in silence with this stranger on our own?"

"Samsara is the mind projected outwardly, lost in its projection. Nirvana is the mind turned inwardly, recognising its true nature."

Water, if you don't stir it, becomes clear, says a Tibetan proverb. Similarly, the mind, if you don't stir it, finds peace. "The trouble is, we stir it," Rinpoche says, amidst a ripple of laughter. If you leave the mind in its true, natural state, it'll find peace or bliss. Settling the mind is called kshamta; removing the dirt is called vipassana, or meditation. Allow the mind to settle, then in that quiet you can experience goodness, which is our true nature.

Adding a footnote, he says, "In the West, people are over-educated, they know too much. I would call it 'being too clever for one's own good'." The audience is obviously amused. "I think that's where ignorance would be bliss."

Many psychologists admit that the gist of psychology is this: the basic cause of all mental troubles is too much thinking. Only thinking creates hope, fear, anxiety and suffering. "A little thought becomes a worry, you build up all kinds of imaginary scenarios, are unable to get to sleep until 2 a.m.-and accomplish nothing!" he says.

It's best to begin simply. "Be spacious," they say in Tibet. The Indian way, he adds by way of an aside, is 'be happy-go-lucky'. We're so used to thinking, sometimes if people don't think for a minute, they worry there's something wrong with them! A French philosopher said that the root cause of man's unhappiness is that he cannot sit quietly in a room by himself.

So at first, just let your mind quietly, spaciously be. "Sometimes, instructions are given just to make you think less and in the end, there's nothing left to say," he says, with irony. "Sometimes, the instruction given has to be emotionally satisfying, and, removing the clutter, you begin to see the wisdom of that. Sometimes, you have to go to monasteries to quieten the mind and its grasping nature."

When in the presence of the masters and the teaching, the mind is quiet, but when one is alone, the mind is back to its clamorous and neurotic activity. So we need practices, like watching the breath. In meditation, 25 per cent attention is on the breath, 25 per cent on staying alert and wakeful, and 50 per cent on spaciousness. Sometimes, one becomes fixated, which defeats the purpose.

As the mind settles, gradually all fragmented aspects of the mind become whole, all inner conflict ceases, the ego and grasping dissolve, hope and fear dissolve, and the mind settles in mind. In that space, you discover your true nature and peace. Sometimes, the problem is not with other people, but with ourselves. So be in touch with yourself, find yourself, it's very nourishing.

If you practise, it brings a healthy self-esteem. And not only the barriers within, barriers from outside also fall away. The sense of separateness falls away.

One listener asks: "I've also at times had glimpses of a spiritual nature, and then I'm comfortable with myself. But soon my everyday self comes back, and again I'm uncomfortable with myself. Why?"

The reply: "Because the battery has worn out, and it has to be recharged-preferably before it wears out. (Laughter) The glimpses are too weak or too small, and the pressure of everyday reality is too strong. It's like an elastic-you stretch it, it goes back. But if you persist, someday it will pop."

Sometimes (he says) we have to go through suffering, attachment, to realize that it's all completely worthless. Suppose you are bewitched, fall in love with somebody, lose your dignity and go through all that. Then one day you meet the same person and say to yourself: "Why did I fall in love with this person?"

With a ready smile, which lights up his face, and a puckish sense of humour, Rinpoche says: "I'm really impressed by you all-especially by those of you who did not speak." There is laughter, and he adds: "I'm just teasing, sometimes we have to tease each other."

Let me spell it out (he continues). First, just be spacious. (Teaching is important, it eliminates ignorance). Then create the environment of the mind-incense, music, lighting, if possible, proximity to nature. Then you practise, it's not really meditation, but creating the right environment for it. Preferably, keep a book of wisdom with you.

Sometimes, dying is difficult if you're attached to life. Often, we associate dying with losing, which can create pain. But the truth of life is that we cannot hold on to anything. Not what is held, and even the holder is changing all the time. "You cannot wash your hands in the same river twice," goes a Tibetan saying. Sometimes letting go is kind, enjoyable; it brings a different kind of appreciation, not of attachment but of letting go.

Sometimes, in the presence of your masters or during practice, you become aware of the mind beyond your mind, and in that moment you feel, "even if I die now in this state, I'll be happy". And in that state there is a letting go. But the problem is, it doesn't last. So keep having these little glimpses. And in that practice there is a letting go, like losing the cloud but gaining the sky.

By Ajay Ahuja

*********************

Stories that illustrate the foundation of Zen philosophy
A says he is not afraid. B says he is not afraid and he is also not afraid of being afraid. The first statement comes from a mind that is tight and assertive-a mind that clings to fixed viewpoints. The second statement comes from a mind that is nimble and free-flowing-a mind that does not cling to fixed viewpoints. The first type of mind is an 180º mind, represented by a half circle. The second is a 360º mind, represented by a full circle.

The 360º mind does not have any preconceived notions-not even the preconceived notion that there should not be any preconceived notions. The 360º mind is open, flexible and uncontrived. It is without blocks and always change-ready.

BREAKING FREE
A spiritual seeker felt suffocated in the world. Wanting to break free, he renounced the world (tarke duniya). As a reward, he was taken to heaven. It was nice and cozy up there but, after a while, he was tired of the good things. So he renounced the heaven (tarke ukba). As a bigger reward, he was taken to God. He liked being with God, but a time came when he had had enough of God's company. So, he renounced even God (tarke maula).

Now there was nothing more to renounce. Yet the freedom that he had been seeking was nowhere in sight. After some uncertainty, he had a flash of insight and he renounced renouncing (tarke tark). And he was back into the world from where he had sought freedom in the first place. Free from being free, he had come full circle.

WHITE BELT
When a novice starts learning martial arts, he wears a white belt, symbolic of innocence. After months of practice, the white belt gets dirty and turns brown, symbolic of the first degree of attainment. After more practice, the belt gets soiled and eventually turns black, symbolic of full attainment.

If the practitioner does not stop learning even after full attainment, the black belt starts getting frayed, turning almost white, symbolic of return to innocence. The frayed white belt represents technical competence of an experienced martial artist, combined with the innocence and receptivity of a beginner. It signifies going beyond technique and embracing no-technique-coming full circle.

PICASSO'S CHILD
Once Picasso said: "I used to draw like Raphael. But it has taken me a lifetime to draw like a child."

Picasso was a competent artist when he drew like Raphael. He became a great artist only when he awakened the child in him and started drawing without any pre-determined technique.

The same is true of every art. For example, the contribution of technique in the work of a competent musician is 100 per cent. But the contribution of technique in the work of a great musician is only 10 per cent or so-the remaining 90 per cent being contributed by the child in the musician. Only when you transcend technique, you become great in your field. You come full circle.

KABIR'S COMPLAINT
Kabir never accepted any gift from his disciples. But his son Kamal never refused anything that his disciples brought to him. This made Kabir unhappy.

One day, he reproached his son: "I do not accept any gift because gifts mean nothing to me. But it pains me to know that you grab all that your disciples bring to you."

Kamal said: "Father, if gifts mean nothing to you, why are you bothered whether I accept them or reject them?" Here, Kabir had an 180º mind, and Kamal a 360º one.

THE VIRTUOUS KING
When Boddhidharma visited China in the sixth century, he was invited to the King's court. The king was proud of his spirituality and the good deeds he had done for his people. He narrated what all he had done to promote religion and then asked Boddhidharma's opinion about the merit he earned.

Boddhidharma's reply was blunt: "No merit."

Obviously, being virtuous was not a great virtue in Boddhidharma's scheme of things. The king had an 180º mind. Boddhidharma had a 360º mind. You can't be spiritual as long as you wear the badge of spirituality. Taking off the badge is coming full circle.

NOTHING GREAT
Once there was a conference of religions to which all faiths sent their representatives. Every representative stated forcefully that his religion was great. When it was the turn of Zen's representative, he stated truthfully: "There is nothing great in Zen."

A member of the audience with a deep understanding of Zen got up and said: "Your saying that there is nothing great in Zen actually makes Zen sound as something great. So you should not have said there is nothing great in Zen."

The Zen representative had an 180º mind. The member of the audience had a 360º mind.

A GOB OF SPIT
For years, Henry Miller lived the life of a would-be writer. He was 45 when he wrote his first book Tropic of Cancer in 1934. Here is what he writes in the opening page:

"I have no money, no resources, no hopes. I am the happiest man alive. A year ago, six months ago, I thought that I was an artist. I no longer think about it, I am (author's italic). Everything that was literature has fallen from me. There are no more books to be written, thank God. "This then? This is not a book. This is libel, slander, and defamation of character. This is not a book, in the ordinary sense of the word. No, this is a prolonged insult, a gob of spit in the face of Art, a kick in the pants of God, Man, Destiny, Time, Love, Beauty… what you will. I am going to sing for you, a little off-key perhaps, but I will sing. I will sing while you croak, I will dance over your dirty corpse."

Henry Miller could not have written such a powerful book if he had not got over his romanticized visions of becoming a writer. He could write the book he did precisely because there were "no more books to be written."

With the transition from "I thought that I was an artist" to "I am", he had come full circle. As a result, he went beyond mere writing to "singing"-not to mention the funny things he did to God and the like in the process.

THE MUMBAI SCHOOL
Do you have free will? If you believe you do, then you have an 180º mind. The alternative seems to be the view that there is no free will.

A well-meaning, emerging school of thought in Mumbai, India, has been hammering into people's minds that free will is an illusion and without God's will you can't make the slightest move. This viewpoint is also indicative of an 180º mind.

Insisting that it is all God's will is as much a concept as insisting that you have free will. When something happens, it just happens. Sometimes it seems that you made it happen. Some other times it seems that it happened by itself or God (or the totality) made it happen. How you view the happening is more a question of perspective than of fact. Which means that the so-called your will and God's will are both labels. Taking off the labels is to come full circle.

THE MIRROR EFFECT
Certain things in the world appeal to you. Certain other things don't. You have your considered opinions about different issues. You may believe there is a benign power somewhere that cares for you. You may believe it's a chaotic world without any rhyme or reason.

How you perceive the world not only tells about the world, but it is also a reflection of how your mind works. Seeing your own mind in how the world appears to you is to come full circle.

ONE THING
When you strive for enlightenment, it's an 180º vision. When enlightenment happens, it's nothing like what you believed it to be. Then enlightenment and unenlightenment don't seem to be two things.

Saying that they are two things and saying that they are one thing also don't appear to be two things. With that realization, you come full circle. The core of this piece does not lie in what has been stated but in what has not been stated. When you see that, you'll come full circle.

With that hint, it is time for this piece to come full circle as well.

By P.S. Wasu

*********************

Sulak Sivaraksa is an activist-economist-philosopher from Thailand who has been using Buddhist ethics for social and spiritual transformation in his country and beyond for the past 40 years. Twice nominated for the Nobel Peace Prize, he is a legend in his own time. He gave an exclusive interview to Swati Chopra

Each one of us carries within seeds-potentialities-for love, anger, happiness, violence, and peace. These lie dormant until we water them with our actions. Being angry sprouts the seeds of sadness and discontent. Living in awareness sprouts seeds of peace. This is the vision Thai Buddhist activist Sulak Sivaraksa has employed to effectively bring about social transformation, for, he believes, each individual is a seed in himself and by practicing it can encourage goodness in others.
Sulak, now widely regarded as a "Thai institution, in a class by himself", is an activist-economist-philosopher from Thailand, or Siam, as he prefers to call his country. Educated in England and Wales, he spent nine years working for the BBC before returning home. In the 1960s, at age 28, he founded Sangkhomsaat Paritat (Social Science Review) that soon became the foremost intellectual publication in Siam. Editing it led Sulak to minutely examine issues related to development and its impact on the grassroots, and then dawned the realization that true change could only be brought about through social activism. Thus began his search for development models based on Buddhist ideals.
For over four decades, Sulak's outspoken denunciation of dictatorial regimes, consumerism and MNCs has made him 'Public Enemy No. 1' for successive Thai governments. He has been the central figure in Thailand's NGO movement and has founded the International Network of Engaged Buddhists.
Sulak was recently in Delhi to deliver a lecture, and to network with Indian friends. Age has not dulled his enthusiasm for exploring ideas and sharing thoughts.

Excerpts from an interview:

What is 'socially engaged Buddhism'?
'Buddhism' is an English word; the Buddha never used it! When 'Buddhism' became popular in the West, it became the notion that Buddhists should meditate, be calm and tranquil, divorced from society. This is very different from the way I was brought up in my country where firstly, Buddhism means practicing sheela-not exploiting yourself or others. This very first step links you to society. The next step is meditation, to sow the seeds of peace within. Practicing these, you attain prajna, understanding of reality And when you understand without selfishness, prajna becomes karuna, compassion.
How did you become an activist?
It began with a desire to truly follow the Buddha's teachings, and to make Buddhism relevant to modern society. It also had to do with a feeling of social responsibility. For instance, you don't steal. But if you let a few collect wealth at the expense of the poor, that is worse than stealing.
How can spirituality be made socially responsible today?
By knowing that 'spiritual' essentially means caring for yourself, not just at the material level, but for your spirit too. In Buddhism, taking care of the spirit means that you should breathe properly. Just by being conscious of your breath, you become aware that every other living creature also breathes. If you care about your breath, you automatically care about others too.
You have used the concept of the Buddhist sangha to build ideal modern economies.
The sangha is a community where you learn to respect the other and live in harmony. It need not necessarily be a group of monks or nuns. Even a family could be a sangha. In modern life, we are alienated from our sangha, even from our selves. That's why we are controlled by machines and seduced by advertisements to want more and more things. In the sangha, 'to be' is important, not 'to have'.

Has your activism ever detracted from your personal dharma practice?
For me, my social activism is my dharma practice. Through it, I have learned to be mindful, not to give way under pressure. Harsh words have been spoken against me. I have been jailed, exiled. During all this, contemplation really helped. Without it, you become hateful and lose your balance. By contemplating on compassion and loving-kindness, you help yourself and hopefully, others too. In Siam, until two years ago, the army regarded me as their Enemy Number One. And now they invite me to teach at the Army College! I have been asked to help teach the army top brass reconciliation methods, and to develop meditation for social action.
That must be an interesting project.
My actual projects are at the grassroots level. Today, the social structure is such that both the poor and the rich suffer, though in different ways.
The answer lies in empowering the poor to be proud of being poor and teaching the middle class to adopt a simple lifestyle. In the past five years, we have extended our projects to Burma, Laos and Cambodia.

Recently, many Dalits converted to Buddhism in India. Is it right to use religion to make a political statement?
Why not? When you have been oppressed for centuries you can use anything to make a statement. But merely by changing your religion, you will not become free. Conversion is only symbolic. By doing so, you are saying that you are no longer untouchable. The Buddha said clearly that whether you are the child of a king or a prostitute, you are equal. This is probably why the Dalits feel a psychological satisfaction by becoming Buddhists. To really empower themselves spiritually, however, they need to practice mindfulness, and learn not to hate the oppressor.
So, mental perspective matters more than just converting.
Yes, but conversion has its importance as a first step. The neo-Buddhists must now form a sangha in which they can practice mindfulness, let go of hatred and become self-reliant. After all, religion cannot be divorced from economics, society, culture, politics and all other areas of human activity.
Education is a key word in Buddhism; it means to learn not to be violent, to yourself and others. Some neo-Buddhists are violent. But you cannot blame them. Sometimes I am violent too.

What is your vision of education?
Western education lays undue stress on proof. But how do you scientifically prove goodness? Or beauty? From the Buddhist perspective, you can gain true knowledge just by concentrating on your breath; you don't even have to believe in the Buddha. The first thing we teach in our holistic education program is correct breathing. Education must teach people to become aware and realize their potential. There should be an ongoing exchange of ideas between teachers and students. These aspects are integrated in our courses, in which we have professionals from Germany, research scholars from Canada, illiterate nuns from Laos, even the boy from the temple.
Effecting transformation through mere breathing?
Breath is intrinsic to life. Buddhism doesn't require you to believe in anything much, it is enough if you breathe mindfully.
How is it to be practiced?
It is very simple. When you wake up in the morning, spend five minutes becoming aware of your breath. Breathe in and out. If you believe in God, invite Him into you. If a Buddhist, you can invite the Buddha into you. The point is not who you pray to but that you feel they are a part of you.
When the breath is in control, mindfulness arises, which can be developed into compassion, loving-kindness, and so on. I remember a Tibetan monk who had been tortured in a Chinese prison for 22 years. When he reached Dharamsala, the Dalai Lama asked him: "What were you scared of the most in prison?" He replied: "I was afraid that I might lose my compassion towards the torturers." The seed of this sort of strength lies in mindfulness and correct breathing.
I read about creative ways being used in Thailand for ecological conservation, like trees being ordained as monks to prevent them from being cut. You have been at the forefront of many such movements.
A tree being ordained as a monk is a symbolic affirmation of the Buddhist ethic of the sacredness of life. Those who cut trees see them as a source of money. If I go and tell them that to harm trees is to harm ourselves, they will not listen. But they belong to the same Buddhist culture as me in which a monk's yellow robes are symbolic of the Buddha. So when we put them on the trees after performing a ceremony, nobody dares touch them. This is an example of what in Buddhism is called upaya, or skillful means.
Any other memorable upayas?
Our lakes were polluted. The government wouldn't do anything. So we decided to perform the tutanka, which is walking in mindfulness and silence, near the lakes. We invited the villagers to walk with us and be aware of the life of the lakes, their breath. They were so moved by the experience that they cleaned the lakes themselves.
Similarly, a monk in eastern Siam realized that to preach to people to be good does not work. He formed a banking system with the villagers. If anyone wants to borrow money, six others have to guarantee that he doesn't smoke, drink or steal. The monk has used money skillfully to help people be aware of their conduct, even though as a monk, he cannot touch money!
Another monk is helping people develop an alternative currency to make their village sangha self-reliant. This currency is meaningful only in their village, so they buy only what is absolutely essential and multinationals lose their power over them. So you see, Buddhism is not only meditation but also effective in social action.
What is the Buddhist solution to the violence in the world today?
After September 11, George Bush should have said: "I am very sad that 5,000 people died, but let's forgive those who did this. Let's come together and talk about it." Unfortunately, violence is being touted as the answer today, it cannot be. I feel that this madness will eventually abate. Everyone deep down has the Buddhanature. To solve conflict, you have to be reasonable, get rid of your prejudices and listen to the other's view. Find out the truth. If at fault, ask for forgiveness.
In Burma, Aung San Suu Kyi has taken on the entire junta. After being kept in house arrest for six years, when she was released, she said: "I forgive them all." Similarly, the Chinese refused to talk with the Dalai Lama for 40 years, but he kept on cultivating compassion and forgiveness. This is why we respect him, even though he does not have any real authority, even over us Buddhists.
How did you cope with your persecution?
Well, compared to the Tibetans, I have undergone nothing. When you are being persecuted, look to someone who is much worse off than you and help them.
What have your 'Ariya Vinaya' meetings with the Dalai Lama been about?
'Ariya vinaya' is the Buddha's term for 'noble discipline'. It is right thinking that helps you get rid of afflictive emotions. Politically, it would mean eschewing violence no matter what; educationally, promoting ethics and values; and economically "as if human beings matter", as Schumacher said in Small is Beautiful. The Dalai Lama is backing my effort for a world based on ariya vinaya. We have started with Buddhists and want to eventually move on to people of other faiths too.
What sort of a relationship do you have with His Holiness?
He is my hero, my 'kalyana mitta'. The Buddha used this term for a guide or mentor whose advice you take seriously. Once, his primary disciple, Ananda said: "The kalyana mitta is equivalent to half the spiritual life." A Buddha said: "No, he is the entire spiritual life." The kalyana mitta is the voice of your conscience.
What has been your greatest achievement?
That I have a large number of kalyana mittas! My strength is making friends. I have good friends in Siam, India, Bangladesh, Laos, Cambodia. Networking is extremely important in today's world because we live in a global village. Although I am not keen on modern technology, I think things like the internet can be used to band like-minded people together.

By Swati Chopra

*********************

DEATH, MY TEACHER
Rigpa house is a large, two-storied building in the west Delhi colony of Inderpuri. The sitting room on the first floor, opening onto a spacious veranda, is tastefully, though somewhat conservatively, furnished, with a row of thankas-traditional Buddhist paintings-adorning the walls. Sogyal Rinpoche, 54, arrives with a beaming smile, an arm outstretched in greeting and amidst profuse apologies for keeping us waiting. He's accompanied by Mauro De March, the youthful-looking Italian director of Rigpa in New Delhi. Excerpts from an interview:

The Tibetan Book of Living and Dying talks about the importance of learning about death while still alive. Can we go into this?
"Learn to die and thou shall learn how to live. There shall none learn how to live that has not learnt to die." These words from The Tibetan Book of the Dead often come to mind when one seeks to understand death and its relationship to life. Among the Christian monks, there is a saying, memento mori-remember dying. Then you might apprehend life. So the extraordinary thing is that death is like a mirror in which the true meaning of life is reflected. Sometimes I tell people, "If you're worried about dying, don't worry, we'll all die successfully! (Laughs) Then what are you afraid of?" We are afraid to face ourselves. Coming to terms with death is coming to face the truth of ourselves. So death really is a moment of truth. The moment of death is, as many religions describe it, the moment of judgement-a time of reckoning. Death that way is the summation, totality of life.

What comes to your mind when you die are two things: how you've lived, and the state of your mind. One reason why people are afraid of death is because they're afraid of how they've lived. It's connected with karma. When you start remembering death, you know, it inspires people to live a good life. Research has shown that people also die as they have lived. The Dalai Lama says if you want to die a peaceful death, you have to live a peaceful life. So death really brings life into focus. Death, now, is no longer seen in a morbid sense, but is the greatest teacher. We're sure we're all going to die one day, but we're not sure how or when. So we think it's not going to happen to us. I always say: "If you breathe out, and you can't breathe in, that's death"-death could happen any moment. That's how fragile, how precious this life is. Realizing this, I must sort out my priorities. Death reminds us that everything is impermanent, transitory, it makes us realise that it is futile to grasp. And in the letting go there is compassion. Sometimes, we think letting go is losing; it's not so. Letting go is actually having, in life itself. For example, relationships, especially today, where the more you grasp, the more you don't have. It's when you are open, respectful to the other person, that your relationship also grows. The whole idea is that you begin to appreciate the transitory, the impermanent, and then actually appreciate life better. It's the grasping that blocks all possibilities. So death teaches us to sort out our priorities, and its lesson of impermanence teaches us the futility of grasping.

Today, there is a tendency to consider everything, including values, as relative. Can there be anything absolute, sacred in such a world?
There is this sacredness, but we fail to recognise or see it. We have lost the sacred outlook, and tend to reduce everything to a relative state. Actually, the relative and absolute are always together. It's a bit like the two wings of a bird. Because there is no absolute without the relative. What is the absolute? The ultimate truth, which pervades everything. That may be spoken of in different ways. For example, in Hinduism and other religions, the divine presence or divine intervention, in Buddhism, interdependence and so on, or the ultimate truth of the shunyata. Yet it actually manifests in the world of form, and the relative appearance. And sometimes when you really see the sacred, then you can actually appreciate the relative in a very profound, much more intense way. When you simply see the relative on its own, even that's relative. (Laughs).

You said that "a thing (in the realm of phenomena) exists and does not exist at the same time." So, can we speak of the nature of illusion?
Look at this cup, it's quite solid. You drop it, break it into tiny particles, even that you break down, where is the cup? It's non-existent. As modern physics has shown, there's no such thing as even atoms. So, in a sense, of course it does appear, does exist, it really seems real for us, but its ultimate nature is non-existence. Yet, we cannot say it doesn't exist. But then you might say, how can something exist and not exist at the same time? Actually, the beauty is that it's true, that's non-dual. Like it exists, but its nature is empty. Form is empty, but the emptiness is form. It's like a movie, when you're watching it, you can be completely entrapped in it and it's kind of real. But when it's over, it's just an illusion! So it doesn't mean that it doesn't really exist-the moments seem to manifest, but when you look at it ultimately, it's non-existent or isn't real.

You said: "The mind is the cause of samsara, of happiness and suffering." Can you explain the concept of samsara?
Basically, it means the cyclical nature (of birth and death, illusion)-you go round in circles-and it's because of ignorance, because we do not recognize our true nature. Our fundamental nature is buddha (awakened) but that's been obscured. Ignorance brings negative emotions, instigated by them we act negatively, creating suffering. We want happiness but we do everything that brings suffering. We don't want suffering but we do not avoid the unwholesome actions that are the cause of suffering. We want happiness but we do not do what causes happiness, which are the wholesome actions. So, the aim and actions are not together. And because of delusion we are in this kind of a repetitive cycle-we've lost our true nature.

The field of consciousness, or samsara, also includes things outside-people, relationships, the realm of matter and technology-which actually makes samsara seem quite complex. Can an individual still be free of samsara?
It is complex. One of the qualities of samsara is its complexity and complicated nature. But it's possible for an individual to be free. That's why some people become monks and nuns-the word is lotang, which means, to cut; detachment. They enter into a different environment, they live by the spiritual principles and not the world or its intricacy. That doesn't mean they are out of this world or not in touch; they are not fools. But they are free. They choose to live like that. For example, myself, I have lived life very much-but then as I become older, I find that it releases me in some way. See the pointlessness of it, realize that the world is only form, and what you really seek is some peace, some happiness. But we make this world so intricate, so complex. That's why so many people in the West become attracted to our teaching. They're looking for simplicity, the natural, wholesome and holistic. As in healing and so on-realizing the downswing of modern civilization. The East, you know, is going in for development; the West having experienced it, realizes that it does not bring happiness, after all.

Living in this world without renouncing it, is it possible to be free?
It is possible, but difficult.

You said that meditation can take place at two levels: conceptual and non-conceptual. Of these, the latter is true meditation, vipassana or mahavipassana. Can you explain?
When you say conceptual, it involves mental processes, for example, focusing on an object, or watching the breath, and allowing your mind to just settle. Once you've been able to establish that one-pointedness, then you're entering into the dimension of the openness, where it goes... beyond the mind, beyond comprehension.

So the conceptual clears the way for the non-conceptual?
Sometimes concepts are used skilfully in order to go beyond concept. Mind is used as a vehicle to go beyond the mind. You know, the great masters because they've experienced the direct, the experiential, they've seen the secret know-how, through their upadesha, they give you a certain way to cut through the context. And in Zen, they have certain rather astounding statements (koans), like "Let's hear the sound of one hand clap."

You said that in an advanced state of meditation, the sense of separateness falls away. How?
I think that happens even at the initial stages, but in moments. Actually when you reach a higher level then you become in union, in yoga-like in Hinduism, one with God. Put simply, when you really practise meditation, suddenly the ego, the grasping is dissolved-the sense of holding onto yourself merges with the greater universe, you also feel one with others, and feel open. Sometimes, when you're selfish, you think of yourself only, just locked up in yourself, and when you're open you suddenly begin to realize others and their point of view, that they want happiness just as you do. It is appreciation and cherishing of the other, replacing the self-cherishing, and the holding onto the self dissolves-temporarily!

What is the concept of shunyata?
I don't think we can even call it a concept. It's not a concept; we can call it a Buddhist view. I think shunyata is when you really break down everything-there is nothing that exists independently. Impermanence shows how everything is interdependent. And that shows you the nature of emptiness. But the emptiness does not mean that it's nothing! (Laughs) In fact, because of emptiness, everything can exist. I mean, without the space, nothing could exist. So emptiness is the great openness within which everything is made manifest. It's difficult to understand emptiness conceptually, it needs to be experienced and then realised. When you really realise, there's a sense of dissolution of self and a merging with and an understanding of the great openness, and compassion.

So shunyata may also be called "fullness" or "openness".
We can call it openness, definitely.

Because emptiness implies that something is taken away.
That's right. Emptiness is a more nihilistic concept! That's why I don't like it. It's as much fullness as it is emptiness. (Laughs) It's not empty in the sense of nothing is existing, rather that it's free.

According to Buddhist philosophy, what is the final goal of life?
Enlightenment or the eradication of ignorance, which is not knowing or recognizing one's true nature. You eliminate that, which is also the removal of negative emotions, which result in all one's negative karma. You therefore uproot the cause of suffering, thereby attaining sangye, the Tibetan word for awakened, the Buddha-open, awakened from ignorance and free from negative emotions, purified. You're open to all knowledge, knowledge of self. Basically it means that you're free of suffering, and the cause of suffering, and you have the ultimate happiness, which is enlightenment. That's what the goal is. All of us actually want to become enlightened, even if we don't know that, because we think that enlightenment is only for holy men and women. But actually, deep down, we all want to be happy; and no happiness that we have in this world is lasting, it's only temporary. The only way to bring about ultimate happiness is the eradication of ignorance, negative emotions and negative karma. Once you remove the cause of suffering, you're awakened; once you remove the clouds, the sky is clear and the sun shines. Because the sky, the sun is always there, like our Buddha nature, and that nature, our wisdom nature and compassionate nature can shine forth.

*********************

Interview: Buddhist lama speaks on meditation
By Kelli Goldman
Editor-In-Chief
The Sakyong Jamgon Mipham Rinpoche, an author and Buddhist lama, recently spoke at Emory University on April 3. The topic was on his new book Turning the Mind Into an Ally.

Collegian: What exactly is meditation?
Rinpoche: The Tibetan word for meditation is "gom," which means familiarity. Essentially, we are always becoming familiar with something: anger, jealousy, irritation, compassion, love. In meditation we become familiar with an object (in the beginning, the breath) in order to strengthen and stabilize our mind.
Collegian: How can meditation enhance every day life?
Rinpoche: It can help us strengthen our focus on whatever we are doing: school, work, family. Furthermore, it can help us find the mental space to see our reactions before we act them out. For example, we can train in seeing our anger arise. Then we have a choice about whether we strike out with spiteful words or actions. Or we can see our attachment or jealousy arise. Then we have the choice to indulge it by going after something or somebody, or let it pass.
Collegian: What do you believe about spiritual paths, and how does a person go about discovering their spiritual path?
Rinpoche: There are a variety of ways in which one might discover one's spiritual path. However, often it comes about through realizing a certain level of bewilderment and suffering, and wanting to investigate it. I think that everybody has enlightened qualities, you know, in Buddhism we call it Buddha nature…Buddha just means enlightenment.
Collegian: I'm very curious, especially with the current situation, between the United States and Iraq, and all this war and fighting, how can meditation inspire compassion?
Rinpoche: If you know who you are, [and] you know your own mind, you won't be easily distracted, and you won't be easily manipulated. And I think when we have fear, we don't trust ourselves, and we might lose sight of our compassion. The thing with meditation is teaching what's most important, whatever you have in your present mind. So I think the notion here is everybody can have peace in their mind - having confidence to look at daily life, and what happens when you do that is you realize that other people are suffering, other people are having struggles, the Iraqi's are also dying. All of the sudden the response is not to kill them more but to have compassion, ask how can we resolve this? It shouldn't result in destruction otherwise the destruction makes matters worse.
Collegian: So would it be safe to say that compassion and wisdom not only work hand in hand but they promote each other?
Rinpoche: Oh, definitely. If you're compassionate and you don't have wisdom, you kind of become a doormat. You think you should do something, but you don't know why. You have to have a little bit of wisdom for better utilizing [compassion] and you also have to know yourself. What can I handle? You can't always be extending if you don't have enough reserve - you'll wear yourself out. So it's kind of the sense of building up, building up to a combination. And if you just have wisdom and no compassion, you become too cold. Without compassion, you may use the wisdom as a way of responding the wrong way.
Collegian: Finally, what do you hope the readers of Turning the Mind Into an Ally will learn and take away with them?
Rinpoche: My hope is that the book becomes kind of a source for people to feel confident that they can meditate, that they can look at that and say, 'I can learn enough from here to be able to handle my mind'. I think it's possible, I think a lot of reasons why people don't meditate or don't feel like they have time is because they kind of feel their minds are unruly and don't even want to think about doing it because they think it's too much, it's a waste of time. And all of the sudden you can say if you think it's a waste of time, than you think you're a waste of time. And I feel like that's how it works, you know, you're relating with your mind anyway, and you've sort of figured out how to keep it at bay but it's never really going to get much better. From day to day we learn…But everybody can handle their mind. In my book I go through a section about how to do different contemplative mediations, how to take meditation into your life. I think there's a tendency for people to think that mediation is sort of the end-all. It's the medium step in-between. You meditate and you become strong, and you clear and learn how to handle your mind, and then you get up from the meditation so you can apply it to your life. So meditation is not the end-all, it's like training. You work out not to work out more, but to go out and do things. So meditation is kind of helping you regenerate your battery so you can go out and do things. I think that people tend to think, 'Oh, if you're going to meditate, you're going to go off and waste your time sitting by yourself.' No, that's not really the point.

*********************

Interview with the Buddhist monk
Pra Maha Thongrattana
The following is an extract from an article which appeared the Italian weekly "Voce Trentina" on 28 June 1998. It is an interview with the Buddhist monk Pra Maha Thongrattana, given during a visit to the city of Trent, birthplace of the Focolare Movement, and where he also met the mayor, Lorenzo Dellai.


What are the common values that form the basis of this relationship with the Focolare Movement?
"The two most important points are suffering and happiness. Also, Chiara's choice of virginity, a choice made by Buddhist monks too, has helped create a deep unity."

Through your contact with the Movement you have also got to know the Roman Catholic Church.
"Yes, without Chiara Lubich I would never have met the Pope, a man who is full of goodness and who is working for peace."

What do the other Buddhist monks say about this relationship of mutual respect?
"In the beginning they were wary. They said: 'Chiara wants to give you a brainwashing!' But when they got to know more about her, and when Chiara came to Thailand, they realised that she is a woman who is full of light, rich in goodness and that she is helping build peace throughout the world. In her talk at the United Nations she invited us to remove the word enemy from our vocabulary and to live like brothers and sisters."

What creates obstacles in the relationship between Christianity and Buddhism?
"One of the obstacles stems from the fact that Buddhists don't understand the meaning of the Jesus's death on the cross and Christians don't understand 'Nirvana'. If everyone studied these two things, they would open up to one another and they would understand. I would ask Christians not to write about 'Nirvana' if they don't understand it properly, because it can hurt the feelings of Buddhists."

As a Buddhist, how do you see the meaning of the cross?
What helped me was watching the Pope and Chiara in front of the cross. I saw that they were moved and they explained to me that the cross is suffering; it is death out of love. If we don't love one another, our lives can end negatively."

In practical terms, how can this mutual respect be fulfilled?
"Religious of both faiths should live more together. I would like Buddhist monks to look closely at the Mass and Christian priests to do the same with Buddhist rites. If we are able, together, to discover wisdom in the different rites, then we can begin to dialogue."

Interest in Buddhism in recent years in Western Europe has become fashionable. What do you think about this?
"I think it is a common heritage for the whole human race. Everyone wants to know why they were born, why they must grow old and why they have to die. This is the starting point of Buddha's teaching. If we know these three things, whether we wear a little cross round our neck or whether we are Muslims, something of Buddhism has entered into our soul. We believe that it isn't only Buddhists who can understand these things, because our laws are nature's laws; they are the truth that is in the heart of every man and woman. It's not a fashion.

*********************

"Just sit"
An interview with
Ken Ireland, Zen priest
November 4, 2000
Morgan Zo Callahan


Mr. Bloom lived a short distance from his body.
--James Joyce
The journey into the inner self is not just the important one. It is the only one. We need to listen to the sound beyond the silence.
--W.B. Yeats
MZC
I'm in San Francisco, California with Ken Ireland. It is November the fourth, 2000. Last year I was in San Francisco to visit the Glide Memorial Church and Reverend Cecil Williams. I noticed a flyer announcing the Tender Zendo at the YMCA Zen meditation being offered, and that's where I met Ken Ireland who's a Zen Buddhist Priest. So this interview is with Ken. And first I wanted to, Ken, thank you very much for agreeing to share with us.
KEN IRELAND
I'm working. That's great.
MZC
I wanted to ask you Ken about some of your background, especially what led you through the process of becoming a Zen Buddhist Priest and what your studies involved, what teachers you met and if you wouldn't mind contrasting that as opposed to say as with a Catholic priest.
KEN IRELAND
Well, thank you very much, Morgan.
You know, the first thing I want to say is that I've been really blessed. I had a chat with Tom Marshall, who's our mutual friend and a Jesuit brother. I really want to thank you for introducing him to me. We were talking and reminiscing over the number of people that I met in my, oh, 30 years of being a Buddhist practitioner and how great it is. I mean, what a great experience to have had the blessing.
The traditional Buddhist would say blessing. I'd say the great luck to meet some really wonderful teachers and to talk with them, to be able to ask them questions, to listen to them, to practice with them, to sit with them. So, in terms of that lineage, I mean, a huge number of people. We pulled out these pictures of my first teacher here, Mr. Chen, a Master; we used to call him Master Yogi Chen.
Yogi Chen (1906-87)
Exposed to Hinayana, Mahayana, and Vajrayana expressions of Buddhism, he studied tantric Buddhism for five years in Tibet and XiKham. He was in retreat in one room for twenty-five years in Kalimpong, India. In 1972, he went to Berkeley until his Parinirvana.
He wrote "all is changing, minute to minute…impermanence applies both to subject and object, it is impossible to speak definitely of either…one lives day by day and traces remain; life is a continuous process of traces with nothing that can be held to."
In the early days, in the '70s when I first met him, he was just called Mr. Chen, Mr. Chen. And he attracted a certain number of Caucasian people around him. He was very Chinese. He had
been in Tibet for 40, 45 years or something, left with the Karmapa before the Dalai Lama left Tibet and he lived in Northern India. He tells the story or he told the story that one day a dragon king appeared to him in a meditation and said, you're going to go to America.
And he wound up in a little apartment on Shattuck Avenue near the University of California at Berkeley and I met him through Claudio Naranjo and a Jesuit Priest named Bob Ochs who was with me at the Jesuit School of Theology in Berkeley at the time in the early '70s. And I would go to see Yogi Chen -- Mr. Chen -- quite often and sit with him and talk with him about meditation. And he was a totally unassuming, natural guy. Although he'd done every bit of meditation that there was to be done in Tibet, practice every practice, sat with every teacher, and was friends with the Karmapa.
He was well known to many people as a really experienced practitioner and yogi. And so I basically stuck with that practice for a long time, that practice and sort of more psychological practice because I think that, from my point of view, a lot of psychological groundwork had to be done with myself before I was even able to sit still for a while. I did Tai Chi. You know, I remember one of my teachers early on when I was sitting, starting to sit in the Zendo and Philip Whalen after a long session or a long period of sitting just remarked.
"For somebody who hasn't sat Zen meditation, you sit pretty well. I guess it must be some of that work that you've done. Because apparently when people begin to sit and they haven't done a lot of work on themselves, parts of their mind or their pasts are very troubling; they'll bounce around a lot and just start to jump and grind and shimmy around and twist and turn different ways, as memories from the past come up that are unresolved or troublesome.
So I spent a number of years just looking at that in various forms as therapy; We call it in California, "personal development or transformational work," just looking at that kind of material coming up and being able to deal with it in some way, handle it in a responsible way. So that was a big part of the teaching for me. But that's only the groundwork. It's only just the spadework that you do in the beginning, before you really can start to sit. And when you really start to sit, all you do is you sit.
You just sit. You just practice sitting. And you sit down and you see what arises in your own mind and in the minds around you. And if you're lucky you find someone that you can talk with about those experiences. And I think that's the heart of the matter. What else did you ask me?
MZC
How do you feel about just being in the world, being a Zen priest? What does that mean to you? And we both come from a Roman Catholic tradition that has priests and I'm sure it's quite different.
KEN IRELAND
Well, you're sure it's quite different but actually (LAUGH) I don't think it's very different at all. (INTERVIEWER LAUGHS) I don't think it's quite different at all. You know, I think that we, you see we're Americans who are Buddhists. And, just like when Buddhism went to China and they invited Zen Buddhism and then the Japanese went to China and brought Zen Buddhism back to Japan and the Koreans went to China and brought Zen back to Korea; they all transformed it in terms of their culture.
So certain parts of it remain. And certain parts of the way in which the sanga was constructed remained. But certain parts of the elder tradition or the Theravadin tradition are cultural expressions of the Southeast Asian culture too, very much so. And certain elements of what come to us as Zen Buddhism in this country are Japanese dharma. It's not the dharma; it's Japanese dharma. And I think that as we begin to invent the priesthood and lay priesthood and people who take precepts and people who are in service and practice Buddhism in one form or another in this country, we'll develop our own sense of what self-less service is, what the dharma is.
So the short answer to your question is that it's actually no different at all because it means that you're in service to the community. The longer answer is that in some ways it's not any different at all too because it signifies your commitment to a certain way or a certain, I don't want to say "form." I think probably commitment is the best word, commitment to a kind of intensive practice which you allow into all of your life. You just allow it to happen in all of your life so that a certain person who's a lay person, at least in the traditional understanding and particularly in the traditional understanding in the East, when the lay person comes into the Sanga for instruction or for ceremonies; at certain points of their lives they need to have these points marked by something a ceremony can convey.
They come in and people actually want the priests to perform those ceremonies. So some of that's available. I mean I don't do too much with the ceremonies. I've never married anyone. I've developed kind of welcoming ceremonies for babies and, living in San Francisco through the AIDS epidemic and having spent a lot of time with people who were dying, we do ceremonies at the end of life, cremation, completion ceremonies, ordination ceremonies for people when they're dying.
And that's a function of the priesthood, the function of the priesthood is that kind of ceremonial function. Priests are people who are picked out of the community to mark these events in life. So that in some ways is no different than what Catholic priests or pastors or ministers do in this country. And yet it's also very different, because they're Buddhist ceremonies and they have a different flavor to them. And although we have some of the same cultural things go on, it's basically, they're Buddhist and we try to be Buddhists by bringing our Buddhist practice to these events.
MZC
Uh, Ken, would you talk a bit about Zen meditation, a little more deeply as far as your own practice of it? A lot of people, when they're starting their meditation, the teacher, as you, always says, well, just sit. Can you go past that?
KEN IRELAND
(LAUGH) No, I can't. I've got to say, I've just got to say that when you begin your meditation you just sit and you find any way you can to convince yourself that sitting down is a good idea. That staying seated for a long period of time to get beyond being discomfortable, uncomfortable, discomforting, whatever, "discomfortable." I made up a new word. It is a good idea. You're going to have to get beyond that resistance and you're going to have to sit until, till your body gets used to it, till your mind gets used to it, till you get adjusted and you can begin to get into the posture and the breathing.
It really takes almost a year, a year and a half before people even begin to learn to sit. So sitting is posture and breath. It's posture and breath.
It's not any particular exercise of your mind. It's not a yoga. It's not focusing. It's not counting your breath.
It's posture and breath. There's lots of things you can do along the way to kind of get you over that hump of just learning to sit and learning to be with the discomfort in your knees. Yet not bullying or criticizing yourself. And Zen practitioners, particularly the ones who are trained in the Japanese Zen tradition are taught to think that sitting is the pain in your knees and you breathe through your knees and you sit with your back straight. I don't know if I ever would have gotten into sitting or, quite so easily if I hadn't done that sort of psychological work which I described earlier.
Another important thing was learning Tai Chi and being aware that my body was something other then a thing that I carried around and sat down in front of the chair, in front of the T.V. and, you know, sat in the car and drove to work and did whatever else I did with it. But that there was something, there was energy that moved in the body. So I actually had some experience of that doing Tai Chi before I started to sit zazen. So that was just me. We're all different.
I don't think you have to do that but I think that was useful 'cause the body is something more then just this thing that sits down. I think Westerners don't like that idea because they have a feeling that the body somehow carries the soul around for a particular period of time. And then when you give up breathing your soul pops out of your head and off you go. But there's something about the whole interaction of your body and what we in the West call the soul or the spirit or the consciousness. That's something that you can explore while you're sitting.
But it isn't that your mind explores it. It's your body that explores it. That "all of you" explores it. So I don't know if we're actually on the track at all about what sitting zazen is or how you get into it or how you do it. But what you do is you just sit. You just sit. And that's sort of like the beginning and the end of meditation. In between other things may happen (LAUGH) but ultimately you're just sitting and when you're sitting you're Buddha. And, that's really true.
Your experience of that deepens. I don't know if that makes any sense at all. But I'm going to let you ask another question. This is much more of a, I prefer to look at this and I prefer actually to look at what we're doing and what the kind of a thing that we're involved in here in this country, and as we Westerners learn about Buddhist practice, is a conversation. So we have a conversation with these people who have come to us, who practice meditation usually from other cultures.
And if they aren't from other cultures, they're Westerners who have kind of been in other cultures and totally adapted to the other cultures. And then they come back and maybe like Jack Kornfield, they change the name to Insight Meditation. And they take off their robes and they practice psychology and they build temples and they sit for periods of time and then talk about that experience with one another. So I think that's really important.
And, included in this conversation are the teachers and people we know who brought us Buddhism from the East. But part of it is, a conversation that we're having. It's not somebody coming and handing us some dogma saying, you know, here's the way, here's how it is, here's how to do it because we're not going to buy that, number one and number two it doesn't work because we're not even asking our own questions then. So one of the things that's going to happen when you sit in meditation over a long period of time I think is that you're, like, questions are going to start to occur to you, you know.
The usual questions are like, "What's it all about, Alfie?" and what's the meaning of life? And after we've looked at Monty Python we realized that our take on it is a little bit different than people who were raised in the Eastern cultures, Japan or China. We both have some experience dealing very closely with people who were raised in Eastern cultures. So the take is very different. The questions are very different. But we as Westerners have our own questions.
And the purpose of meditation is not to ask the questions which are appropriate for people who are raised in an Eastern culture but to ask our own questions and to ask them inside the context of our sitting practice. So that's what I'm interested in. I don't know if anybody else is. But that's what I'm interested in and those are usually the people that I talk to in practice, the teachers that I talk to or people who are interested in that. The practice that I do is interested in that. That's what I talk about when I talk with people that I sit with.
MZC
Often in, you mentioned Jack Kornfield and some of the Vipassana practitioners. They'll talk also about posture and breath, paying attention. But they'll always add, they'll say, "Do it mindfully." In other words that just posture in itself and just breathing in itself isn't useful if you don't relate it to your consciousness in the moment.
KEN IRELAND
Right. So when you were having that I had this, (LAUGH) I remember this story. You were going to ask me this story. There was a friend of mine who's really a great guy. Um, Dutch, another Dutch, an American Dutchman who sat at Hartford Street (Zen center in San Francisco) for a long time. He's also a body worker and he's very much into his body and he's really good. You know, he sits well and he's very erect and very alert. And so when we were doing sesshin there or something or maybe just a daily retreat, after we ate the Orioki food, someone would have to go in and wash the dishes. The practice at the time at least the way we thought the practice was, that we would go in and we'd pick up the dishes slowly. And we'd put them in the water slowly. And we'd pick them up and we'd take the washcloth inside the pan. And we'd follow our breath and we'd clean it all like this. And, you know that was mindful. Okay, that's called being "mindful."
That's what we said. And we'd clean the dishes and put them away and go back to sitting. It was all a very lovely, slow experience. So my friend went one time to that place in Santa Cruz, the Tibetan one. And he was working with a Tibetan lama down there, a teacher. And they had maybe 80 people sitting a day retreat. And he got assigned to the kitchen after dinner, after the lunch.
And he was supposed to go in there and wash the dishes. So he walks in and he starts and he picks up the towel and he very slowly starts to go through the thing. And the teacher comes in and he says, what are you doing? Well, I'm being very mindful as I wash these dishes. The guy says, "We got 80 people to feed." What are you talking about, being mindful? Be mindful. We got 80 people to feed. Speed it up. So there's ideas that we have even about being mindful that are, half of them, wacked out and crazy.
Being mindful is paying attention. How can you not be mindful? Your mind is present when you're doing what you're doing. Right? And so if you're going slowly when you're washing dishes, okay. But that's not being mindful. If you're going fast when you're washing dishes maybe that's mindful, maybe not. I don't know. There's another quality that you bring to it, uh, a quality of attention, paying attention. So you can pay attention when you're driving at 90 miles an hour and I hope you are if I'm on the same road with you.
And there's a quality of attention that you can bring driving 25 miles an hour and I hope you are when I pass you. So it's a quality. There's a certain quality. Now, the question is how do we think, with so many of these words that we become enthralled with, so many Buddhist terms like this? Whether they remain in Sanskrit or whether they remain in Japanese or in Chinese or we translate them into English or we do a lot of work on the philology of them?
I don't think we really even understand them until we can, begin to practice, until we begin to sit, until we begin to realize that what it is we do is when we bring mindfulness to a situation, might not even be at all what we think it looks like. We have an idea what it might look like. And then we sit down and we practice and we sit for a couple of weeks and then a couple of weeks stretches into some months and some years and six months, six years, twelve years, twenty six years later you're looking back on it.
And you realize that actually you have developed a certain quality of paying attention and you notice that when you rub your fingers together you feel your skin differently than you did when you were 18; or that you listen to your voice differently than you did or when you speak in front of a crowd, you hear yourself differently. Or when you don't speak and you listen you're hearing yourself and the other person differently. So it's a quality almost of noticing, noticing something that's changing. And I don't know if that speaks to your question at all. But that's what I have to say about it.
MZC
There's another quality too about meditation that beginners talk about. That is a quality of being a witness. You mentioned psychological things come up. I think all of us when we started meditating could see a lot of emotion come up or memories from the past or projections into the future and then we could tap into some quality, some point within ourselves that was feeling the emotion or thought and we'd watch it and could let it go, other times not, other times getting caught and going off to the past or the future.
Is there in your practice, does the arising of thoughts and feelings become more easily witnessed as you progress in your meditation?
KEN IRELAND
(LAUGH) This is one of those questions that you know, I want to say, witness shmitness bullshit, you know, I really do. Part of me wants to say I don't even know what that means. And the other part of me wants to be able to listen in a conversation and try and listen to another person's experience of developing a witness consciousness and certainly we've all heard about this.
I think Dean Ornish wrote a big, long book about it and I was very involved with Claudio Naranjo and that whole group in the seventies. And they all wanted to practice zazen; and we practiced meditation in order to perform certain psychological functions and they all thought you'd be better off if you had a witness and you were able to see what you were doing from a more objective point of view. Well, number one is you're never going to see it from an objective point of view period.
That's impossible. It's always a personal point of view. You can become somewhat more detached from what you're doing. But whether you're developing a witness consciousness I don't know. So the short answer to the question is, I don't know. I don't really know what witness consciousness is. And then the long answer to it is that I want to take very seriously the people who practice meditation and do psychological work because I think they made a valuable contribution.
They've sort of developed their own kind of language for it and some of it may in fact be appropriate meditation language and some of it may be an adaptation of traditional meditation language. But, you know, the long and the short of it is, the more I meditate -- I've been meditating close to 30 years now -- the more I meditated I'm less and less interested in whether or not I have a witness consciousness, you know, that's sort of hovering over my shoulder watching my every move.
I do what I do and that's that. And it becomes less problematic for me. At one point I think I probably did spend a lot of time actually trying to consciously look at what I was doing, what the roots were, where the causes of my actions were, just to kind of figure things out, figure myself out a little bit. So if you want to call that witness consciousness, I'll go with that. But I don't like when we talk about acting with a kind of "mindfulness" or "awareness;" those are funny words.
They seem to me to be English words and they're not coming from a meditation practice as such. Mindfulness is a different case but I think witness consciousness is particularly coming from a psychological place. And so as I said, I'm a little bit split on whether or not to pay too much attention to the psychological side of things or to spend some time paying attention to the actual practice of meditation and see what that brings up. See it's going to create its own stuff that we don't even, when we're looking at it, we don't even know what's going to happen. But it certainly happens.
MZC
So for you, the heart and soul of meditation is in the practice of it, the vital place (LAUGH), I want to change the theme a bit, Ken. I want to come back to meditation in terms of Suzuki Roshi, beginner's mind later on. But, I wanted to ask you since you've had experience studying the life of Jesus and studying the life and teaching also of the Buddha, do you relate to Jesus and do you relate to Buddha, in any way, in your practice and in your considerations?
KEN IRELAND
Sure. I think you have to. And I don't know, see both of these figures exist as historical figures and they exist as mythological figures. So there's things that they might have said and then there's the things that people make up about what they have said. So I don't know, like, this is very interesting, the whole question of the Jesus Seminar and what we can actually pin down as the words of Jesus and the historical Jesus. And if we look for him as the son of God to have come down and said certain things which will change our life and that we have to accurately hear and understand what he said in order to get the message for us; I think at some point we're looking in the wrong place.
That's not the place to look. And even if we look in the teachings of the church in terms of looking at what people have made up about what he said and how they've interpreted it and what they take it to be, we're still missing the point. So the point is what? My teacher, my first teacher Yogi Chen once said to me, he said he was very interested in Catholicism and "I have heard that there are people in this world who have the wounds of Jesus in their hands, in their feet, the stigmata."
He said, is that true? And I said, well, I've heard it's true also, you know, and he said, that's the person I would make Pope. (LAUGH) (INTERVIEWER LAUGHS) So, I kind of got it. I said, oh right. So you'd look for somebody who has the real signs of embodying the teaching of Jesus in their person, mm. This is interesting, very interesting. So now to move into Buddhism a little bit and I approach the historical Buddha from the point of view of being this rather crusty intellectual, you know, agnostic Jesuit type.
So you're looking at the words of the Buddha and somebody then will tell you what fact Jesus or the Buddha said. Doesn't make any difference. They're making something up. I once was talking to a monk. And I said to him, but how can you possibly believe that because here you are, you're looking at texts that have been written four or five hundred years after Lord Buddha walked the face of the earth in India.
How can we possibly say that this word comes from the past directly from Buddha; it's been passed down and everything. And the person, the monk just looked at me, he said, 2400 years later, big deal. That's still Buddha speaking. And I kind of got sort of the same thing of, you know, the woman or the person, the human being with a stigmata in their hands being the Pope and the spokesperson for Jesus in this day and age being like the person who wrote the Mahayana text out of his experience of meditation. And, the miracles or whatever he produced in his own life, his enlightened life being the source of the text we could say, yes, that's Buddha speaking. Does that answer the question?
MZC
Ken, you study, if I'm correct, Rinsai Zen as opposed to Soto Zen.
Would you care to distinguish between the two?
KEN IRELAND
Mm boy. So the particular school that I do practice with at the current time is from Robert Aitken who is the American source of this school, which is a combination of Rinsai and Soto practice. So the teachers received Inka transmission in both the Soto tradition and the Rinsai tradition. Now traditionally the Soto tradition is practicing Shikantaza, just sitting, so that they go and they sit and they sit and they sit and nobody bothers much with what's going on in their mind. It's all Buddha mind and that's lovely. And the Rinsai tradition, at least the modern version of that in Japan felt meditators needed the pinprick or the, grit or the "something" of koan practice. So koans are in Rinsai practice. You do sit with a koan. And you work with a teacher in terms of understanding and demonstrating the answer to the koan. And there are 3,000 or so in the curriculum.
But the Soto people have koans also. They're just not as close to the surface and you may work on them from time to time with your teacher although lots of little things about the practice are different. Your connection and your communication with the teacher during sesshin or the long sittings is not as intense in Soto practice as it is in Rinsai practice where you'll see the teacher two or three times during the day; In Soto tradition, we may see a teacher once or twice during the session.
At least, that's been my experience. So whether or not you have a question, you're dealing with something; see I prefer myself having done both, my personal preference is to work with a koan, to work with a teacher, to work intensively during meditation. Now it's also true that it's all made up, about whether you're actually doing any work because you may be doing some work or you may not be doing some work.
Your meditation may be producing results or it may not be. Um, you're just sitting. However, you know, I'll say that the Rinsai practice does have a different flavor. It's a little bit more intentional; I found it more focused. It's interesting, you know, long ago, the name I was given was the answer to a koan. So my Soto teacher at the time felt I was destined for koan training. So that's what I'm doing.
MZC
Well, you know, there's so many different kinds of Buddhist practice that we have in the West now. Mahayana and Theravadin, the Zen, Vajrayana from the Tibetans. Would you say there's a common thread, Ken, in all these. I ask other teachers that and most of the teachers say the common thread in Buddhism are the four noble truths, that through all Buddhism, the four noble truths are supreme, that that's what the Dalai Lama would say also when he talks.
I don't know if this is an important question. I don't know if I'm just trying to categorize, trying to get a key on what might be the essence of a Buddhist, someone that says she is a Buddhist.
KEN IRELAND
Yeah, this is a really good question. It's really a good question. I just met a person, a woman teacher, south of here who is really an anonymous teacher. She has received her transmission from her teacher; she says: "I have the stick." And she does. She received her transmission and I know the teacher and I know the lineage and she checks out. And yet she doesn't have a Zendo.
And she doesn't have students. And she talks to people who come and sit in her living room and she doesn't ask them whether or not they're Buddhist and she doesn't ask them whether or not they believe in whatever they believe in. She simply asks them questions about their lives. And they're not always the most comfortable questions either. They're interesting questions. They come from her experience and her loving and her own dealing with her daughter's death and her own dealing with her own cancer and, she's really quite a wonderful woman.
And I would say in my experience, and I don't even know if I'm correct in saying that, that she's a Buddhist teacher. I know that a Buddhist teacher from Japan, very respected, gave her the authority to teach Buddhism and Buddhist practice. And she doesn't consider herself any of those things. She simply talks with people who come and sit down in her living room. But there's the thread of something genuine there, something genuine about human living, something genuine about the way in which she approaches her life and the problems of her life and the delicacy and the joy of living and taking each breath carefully.
I say that her practice of mindfulness is different because she's a cancer-survivor. And her practice of meditation is such that when you go in to the room she's able to express that in a way in which you understand your own life better. So I don't know whether she's Theravadin. I don't know whether she's Vipassana. I don't know whether she's Zen. I don't know whether she's Rinsai or Soto Zen. She can tell you a lot about the various practices that she's done in her life if you ask her those questions.
But she's coming from a very different place. She's not coming from a particularly dogmatic or institutional place or she's not trying to teach you a particular practice so that you come around and she can collect the money for your having done the practice or so she can teach you what she knows. She's coming from the place of authentically living her life in such a way that's selfless, that she is very willing to share it if you in fact want to ask her questions and want to work with her and want to talk with her. So she can guide you in a way.
I'm going to continue my conversations with her because I think she's great. And I think there's something really genuine about her and I also have disagreed with her from time to time. But we've enjoyed lots of interesting moments together, discussions about literature and so forth. And I said I don't know what it is. I think it's Buddhism. It might be humanism. It might be something else. I don't know.
MZC
It sounds like you're talking about a quality, a human quality that maybe is even more fundamental then all our categories, a way of relating from the heart. I don't necessarily want to use a word for mindfulness, but (LAUGH) or from . . .
KEN IRELAND
I want to say something about this mindfulness. All right, look at it. If you're going to use a word like "mindfulness," you actually have in your mind something that you think it means and if you're using this as a practice, for God's sake go ahead and use it. I mean, if you want to wash dishes slowly, do it if it means something to you. I think the key is with any practice that you have someone you can talk to this about, you know. Otherwise you're going to be expecting the dishes to wash themselves or to pop up clean and germless and such things which just aren't necessarily so, which won't happen.
So to isolate or to understand a quality of your own mind so that something about your life-experience becomes accessible to you, do it. That's what practice is about. I don't want to step on anybody's practice and I think Thich Nhat Hanh's a great guy actually, who's the sort of main modern exponent of mindfulness teaching. And he's done a lot of good in terms of being able to make other people aware of their own traditions, what mindfulness is all about, you know.
Catholic priests doing ceremonies mindfully are much more communicative of the essence of the numinous or what they call the divine or the transcendent than people who just take the book and jam through it, like you're cutting through traffic. Or you're an ambulance with the siren on getting to the hospital.
MZC
Yeah, Thich Nhat Hanh, now there's an example of someone who on his altar places Buddha and Jesus. For him that's meaningful. I like what you say too. You don't necessarily have to agree with everybody. I found that some of his practices for me wouldn't be useful. One example was he was telling the people to smile. You may not feel like smiling but smile. That's the greatest mantra. And, it may work for somebody but if it's not coming from out of you that wouldn't be a very useful practice, right?
I like what you said about mindfulness, if that comes from our practice, it's useful. It's used a lot at Spirit Rock, mindfulness practice.
KEN IRELAND
But there you can have a teacher who understands something. You have a teacher who understands something, who actually has some particular quality of mind and understanding that he wishes to communicate to you. And he communicates it to you through his body language and through your voluntarily undertaking a kind of practice in conversation with him which will then hopefully yield some experience which will change the quality of the way you experience your life.
MZC
You know, you've mentioned often, Ken, about the importance of a teacher to you, as far as now and for getting back to different kinds of Buddhism. I know in the Vajrayana practice, the siddha teacher is the Buddha to them.
KEN IRELAND
Yeah.
MZC
In what way do you think that the teacher is so important in your practice and do you think it is possible to practice Buddhism without a teacher?
KEN IRELAND
I don't know if it's possible to practice Buddhism without a teacher but I don't think it is. Maybe, maybe the community would function as a kind of teacher. I think the most important thing about having a teacher is that the teacher prevents you from entering into a kind of solipsistic world where your own thoughts justify what you're doing and your own thinking justifies your thinking and justifies your obsessions and your passions and your prejudices and whatever. So the part of being in conversation as human beings is we're not separate but we actually have some place outside ourselves to direct our language and our thoughts and our ideas and our conceptions to get some feedback.
And it's not so that you may think that, when you see all these people sitting in the zendo facing the wall or facing out and their eyes are half closed and they're not moving and they're not looking at one another when they do their meditation or their walking, that we live in a solipsistic world. And that what you're doing is getting into a completely isolated environment. And that's exactly opposite of what's happening. You're entering deeply into yourself only to enter into communication with the world in a completely different way.
And the teacher is the pole for that to come back and to be the question. You said something really great when we were having our Chinese noodles that Robert Kennedy S.J. said to you at one point, "Well, what's the question behind that question.?" You see, I don't know if that would have occurred to you unless it were in a conversation, but in a conversation with a teacher who's been trained and who has really been trained to a certain degree and his training and his experience has been authenticated by his teacher.
What happens is that person then is the guy you go to to say, you know, direct me to look at the question behind the question that I have in my mind. Direct me there. I invite you to come into my life. I invite you into my life to ask that question which means the world to me, which unlocks my life, which makes me a human being, which allows the dharma to come alive in me. That's the invitation that you ask when you enter into a relationship with a teacher.
It's not to get something. It's not so that that person can give you some truth that exists outside yourself. It's not so that they can hand a dogma to you that's going to save you or make you free or make you happy. It's to enter into a real conversation with you so that you can begin to explore yourself at a completely different level. You know, so we get people to sit when they start; we start saying just sit and you just sit. You just sit and you get bored and you give up and you come back and you give up. And you do all the things that you do when you start sitting. And particularly when I started with Soto practice.
Yeah, I was always amazed in Soto practice when I went in to get meditation instruction the first time. When I was with other Vajrayana practitioners who would try to lay out their whole philosophy of mind in front of me then say, and here's the reason why you sit. And, um, (LAUGH) and, in contrast, I remember at Hartford Street when I first went there for meditation instruction, somebody said, there's the cushion. Okay, we sit down on the cushion.
What we do is we face the wall, okay. We sit this way. We hold our hands this way. We fold our feet this way. Um, we follow our breath and the bell rings. And then the bell rings and it's over. But during that time you don't move. I said, oh, okay, that's it, huh, this is meditation instruction. He said, yeah, that's it. Then I got up afterwards, and I kind of stumbled on the floor and they handed me something.
And I sort of mumbled some things about, you know, "no eyes, no ears, no nose, no tongue, no body, no mind, no color, no sound." I said what the hell is going on here? This is totally incomprehensible. And yet I still continued to sit. I just went back to just sit. And it took a long time, well, not so long but it took a while. And then at, at some point I just said, oh right, okay, I'm just sitting. And you get up and you say this thing which seems to be a negation of many of the particular things.
But it follows along with a certain kind of product, projection or, uh, I don't mean projection. I mean it has a certain pace and intention. And nobody even bothered to explain that to me. You just sort of chanted it. Sometimes you even chanted it in Sino-Japanese, which was even harder because it was really incomprehensible then. And it didn't make any sense and it was hard to do. And you wondered why you were doing that. And then at some point, at some point, ahh, nothing here, nothing here, right.
Nothing here, nothing here. There was a part of me involved in that, at some point very heavily like yeah, there's nothing here. And I got into that whole process of just following down the via negativa. It's not this, it's not this, it's not this, it's not this. And the question is what is it? It's not all this. Then what is it? And nobody's handing you an answer saying, it's this.
This is it. This is what to believe. No, it's not that. It's not that. It's not that conception. So you're just like sort of repairing yourself. You're just kind of getting the groundwork ready for some meditation experiences which at some point do become available for human beings.
MZC
Ken, if you don't mind I'd like to read a little bit about meditation from Shunryu Suzuki, about Zen mind, "beginner's mind."
KEN IRELAND
Suzuki Roshi, yes.
MZC
He also of course talks about Buddhism as within the practice. That's the heart of Buddhism; our practice of it, actual practice. This has been very useful for me. I'm not saying I understand it. I'd really like your comment on a few of these things that he says.
He writes, it's very difficult to keep our mind pure and our practice pure in its fundamental sense. Yet he says if you continue to practice one, two, three years or more, you may improve some. But you may also lose that a limitless meaning of original mind. "For a Zen practitioner the most important thing is not to be dualistic. Our original mind includes everything within itself. It is also rich and sufficient within itself. You should not lose your self-sufficient state of mind. If your mind is empty it is always ready for anything. It is open to everything. The beginner's mind is always appropriate. In the beginner's mind there are many possibilities. In the expert's mind there are few. When your mind becomes demanding, when you long for something too much, you will end up violating your own precepts, not to tell lies, not to steal, not to kill, not to be immoral and so forth. Yet if you keep your original mind, the precepts will keep themselves."
KEN IRELAND
Can I see this again? So he says yeah, beginner's mind. He says that we have a hard time, yeah. I'm interested in helping you keep your practice from becoming impure. I guess that's what he's talking about. I mean, this is a great book. I think that people, people rightly have taken this to be one of the key texts for introducing meditation to Westerners. It's so rich, you know. Issan Dorsey read it over and over and over again and always found something new in it every time he read it.
Philip Whalen who's incredibly intelligent reads this and says, when you begin to read it, you see the whole underlying basis of things, this great training and the sutras and all levels of practice. And a real example of beginner's mind. He's saying it like for the first time. Here's the Buddha speaking about meditation instruction right here. This is just coming from him right now. It has a whole history and an erudition and a training to it.
But when it's spoken it's so simple, it's so direct. It's so crisp and yet it's so profound. You keep on coming back and it's got more there, more there all the time. So he says so many things here. I couldn't even begin to comment on them. I do have one quibble when he says that it's hard to keep your practice from becoming impure. Well, I suppose if you get some idea about what's going to happen, when (LAUGH), you have, if you do, if you're fortunate enough or unlucky enough to have certain kinds of experience in meditation that "blow your socks off" kind of experiences, you want to repeat them again.
You want to go back and have them all over again, because they were so fabulous and alive. But guess what, you can't do that! So then you get into a routine of like, because we're taught not to expect anything. And it's your mind-set when you have talked to your teacher, you're saying, I want this to happen and the basic conversation is, well, why do you want that to happen. Well, that's not going to happen. How do you know that that's going to happen.
So almost your expectation prevents you from seeing what's really happening because when you're sitting, whatever is happening is happening. And, it's all great. But there's something, there's a key here that he really has; he really puts it on. And it's worth some study. And I think the key lies in beginner's mind and the infinite possibilities, infinite possibilities available to a beginner's mind.
You know, when we're kids and we start to be taught in school and start to be put on the path of specialization by the time we're in the ninth grade. And you're preparing for exams and you've learned the language and you've learned algebra and you've learned some calculus. And you begin to think that these are the techniques that yield the answers in life. And you don't realize that they're just techniques have been developed to solve certain problems that some particular people had some good results with. I don't know if I want to say anything more about that. Keep reading the book, it's a great book.
MZC
Thank you. There was one thing too that, not only for Suzuki Roshi but a lot of the Zen teachers especially in the Vipassana that I've read: how important your physical body is in meditation, paying attention to your physical body.
KEN IRELAND
What else are you going to pay attention to?
MZC
Well, there you go back to Suzuki Roshi saying that because in our mind we always make everything dualistic.
KEN IRELAND
So you don't think that the thoughts that are being produced by your mind about your cravings for dinner or for hot gruel or great sex or memories of the last time you had great sex six years ago you don't think that's a production of your body and the neurons just flashing off in your mind? That's all it is. Pay attention to your body. Pay attention to this happening. This is a, this isn't sort of a, you know, it's the, the life of fantasy and calculation and justification and forming opinions and rationalization is not a spiritual activity. It's a physical activity of the circuitry of the brain working in its unhinged fashion. And pay attention to it just like you're paying attention to your knees. That's all it is. It's your knees giving you problems, you know, misfirings between your ears. No secrets to it.
MZC
Ken, in working with young people, and I know you work with young people also, I don't know exactly why it's so important to me but I'm not saying this is intellectual to me. It's more experiential to me. When I'm working with young people and, well, we all have our troubles and we all kind of, sometimes lose our way. Yet, I can see their goodness. I can really see the goodness of the person.
And I intuit and feel that they don't often see that goodness. You know, they don't see it. I don't know if this is really related to Buddhism. But I want to read from, Trungpa Rinpoche about basic goodness and have you comment on it.
Chogyam Trungpa Rinpoche said "there's always the primordial dot, that spark of goodness that exists even before you think. We are worthy of that. Everybody possesses that unconditioned possibility of cheerfulness. We possess what is known as basic goodness. Then we develop an overlay of unnecessary tricks or occupations. We develop little tricks to shield ourselves from being embarrassed or feeling too painful or too naked. Those are habitual tendencies but they are not fundamental. It's a veneer of some kind called "habitual tendencies" which have to be renounced definitely. Seeing the basic goodness in oneself is being willing to make some kind of a sacrifice, namely rejecting our self-indulgence, or purely exclusively pleasing one's self. Similarly when we experience depression it is possible to cheer up. That situation is genuine but quite workable. From morning depression, for example, and its terror we can step right into basic goodness. We can learn to reject the terror of depression and step into morning, the morning of basic goodness right on the spot."
KEN IRELAND
That's really great. He says that the spark of goodness exists even before you think, right?
KEN IRELAND
Yeah, even before you think, even before anything and your question was, like, there's always the primordial dot, that spark of goodness that exists even before you think. We are worthy of that. Everybody possesses that unconditioned possibility of cheerfulness. That's such a great statement. That really rings true, you know. When you were speaking, I was reminded of one of Aitkin Roshi's book called Encouraging Words.
And we talked a little bit about the role of the teacher. So that role could go to be the role of the parent or the role of the teacher or the role of the citizen or anybody. It's just, I think one of the basic things that we overlook, that when we get sloppy with ourselves and self-indulgent we allow ourselves to just go into judgment and criticism and opinions and stuff and forget that what kids need is just encouragement, encouragement 'cause they're great.
I mean, kids are great, young kids are great. Everybody goes around saying kids are not doing this. They're this and that, yeah, people have said that about the younger generation from before time, from before Jesus, from before Buddha. The Zoroastrians were probably saying that about their kids. Just no good, not going to measure up to anything and I think that the key to this is just encouraging words.
We don't even know what the results are going to be. Who knows what they're going to be able to produce? Who could have predicted, in this century, we unleashed relativity and developed the computer and rock music and Stravinski and all the wonderful things that were produced in this past century? Well, their parents probably said the same things about them. Then, on the other hand, there were also kids that were nourished in encouragement. You know, I remember reading somewhere that, one of the Kennedy kids; maybe it's a Shriver, the one who's married to Arnold Schwarzenegger.
MZC
Maria Shriver.
KEN IRELAND
Maria Shriver said that when she was home at the family dinner table and they sat around the dinner table, it was always encouraging words. Like, what did you do today? And when the kids were encouraged they were encouraged to do more things and more things and more things. And we all take them to be, or at least a number of people would take them to be arrogant and over-achieving and full of themselves but, you know, Kennedy was responsible for the Peace Corps and Vista, a whole number of projects that grew out of that idealism.
Grew out of encouragement, encourage what's there. There's nothing that isn't there. Every human being has that spark of ingenuity; every human being has that spark of goodness. We encourage it, not slap it down and try and specialize it or stultify it or correct it or channel it or anything. Encourage it.
MZC
Ken, I have two more topics. I know you have worked with dying people. And I'd like to ask you about that. If you'd like to. One thing that comes up in our questions about dying, in relationship to Buddhism, Buddhism regards "the soul", in quotes, much differently than say the Catholic does. Or the Hindu does; the Hindu and the Catholic take the soul to have a certain immortality to it, right, in at least one way. There's of course mystery to it.
Buddhism would say there is a soul, but it's certainly not immortal. It doesn't have it's own essential independent existence as a soul. Does your theology of a soul make any difference in how you relate to a dying person? Secondly, how do you relate with someone who's dying?
KEN IRELAND
Can I just start with the first, the last question first? Because that's far more interesting to me. So I never work with dying people, they work with me. (LAUGH) You know the first thing that you become aware of, when you start working with people who have a grave diagnosis and who are very close to death, that when those considerations fall away about what's going to happen tomorrow? And am I going to be around? And what have I done for the future, and what's going to happen to me in the future?
When those considerations fall away, the real selves start to come out. And I was going to say that the thing that you have to sort of get yourself ready for is to be the subject of totally unreasonable demands and requests. And somehow you wind up fulfilling those. You know, I want to be, I want to have my relationship with my family straightened out before I die. I, I want to, you know, make peace with so and so before I die. I want to die in a particular way. On the other hand, what also happens is that the process of dying has its own; it has its own agenda.
It's nature, it's nature that we can't fight in a lot of ways, although medicine tries to do it. And nature takes over, and the process of nature takes over. And the body begins to shut down in its own way, and begins to defend itself. And you begin to prepare for the pain of life ceasing. And so those mechanisms are physiological and mental and psychological mechanisms, like, move into place and take over the function of life. So you're also at the effect of those processes and you can't, you just don't, you have to kind of like give up control it.
You just say, okay, I'm in this particular space with this person, I've decided to be here. I've committed myself to be here, to be of service. To ease the transition, to alleviate the pain. And so you just allow yourself to be in that, and you do what's necessary. So it's absolutely fabulous in terms of training, and in terms of human reward. I mean the reward is enormous. And it's incredibly difficult from the point of view of your ideas about what things should be like. You don't hold onto those, you know, you can't, they're gone. So it's great practice, I recommend it to anyone.
And, you know, the person who really wants to find out what life's all about, we talked on the Companion's List for some time at one point about the meditation on death and meditation on the disintegration of the body, and the meditation of the permanence, and Yogi Chen, who's picture is here, lived in the cemetery for three years. He actually lived in the charnel grounds where dead people were brought in Tibet to have the flesh stripped from their bones by people who were specially trained to be practitioners to do that, because they couldn't dig into the ground to bury people, and they didn't have fires and so forth.
They usually left the flesh for the vultures, and then did something else with the bones. So it was a religious practice which we would in some ways consider kind of barbaric. But you live through that, you go through that practice, you learn how to deal with your body that way, and learn how to deal with humans that way with respect, with love. And something in you changes. Something inside of you changes in terms of your relationship to people. Your relationship to your own life, your relationship to your passing years. Your relationship to the physical breakdowns in your own body.
And your achy knees and your graying hair, and your sagging belly, and your fading eyes. Those things change. And it's not just acceptance, it's not just acceptance or resignation, because those words don't even echo, I mean, they're common in culture. But it's a total transformation of that; it's, as a matter of fact, those words I don't even think occur. I don't even know why they occurred to me at that particular point, but they did. I wanted to say something about that. So I actually, you see, I think that the whole question of that sort of funny theological question, that scholastic question about how we live on, whether we live on, whether this personality lives on.
Whether it takes on a new body, whether our particular spark, I actually think those are sort of avoidance questions. I mean, they may be a question, but they're a question which I'd like to ask yeah, but what's behind that question, you know? What's behind that question? Does it mean something like, are we really concerned with what our lives are about? Yeah, what are we concerned about? What are our lives about? What do we want to make our lives about? That's even more important. What do we want to do with our lives in the time that we have?
How can I do something that's of value? And so this is when I would say that, you know, that, well the four noble truths, or more importantly the vows of refuge come in, you know. Yeah, I'm going to make something that's going to benefit all beings. That's what I'm going to do with my life, I'm going to be a benefit to all beings, because I'm part of the universe. So I don't find the theological questions very interesting. I mean, I suppose they are on some degree, and at some point I was very interested in them. But I'm not now.
And I'm not even really interested in the questions of sort of like the, I think what we get into these days with a lot of hospice questions and so forth, is, like, how can I survive it? You know, how can I actually take care of a person in the way in which they expect and survive and protect myself and take care of myself. Well you'll figure that out; that's not the point. The point is to be of service to somebody when they're at the end of their lives. And then the question becomes something like, instead of avoiding the end of life, how does life become full and complete from beginning to end?
And not just, not something we make up things about, but the whole thing's alive and well, and it breathes and pulsates, as we breathe from beginning to end.
MZC
One night I got this call; it was maybe 1:00 in the morning, and a member of our school board called me very desperately, and said this Japanese gentleman who was a Buddhist was dying, they wanted to pull the plug and so forth. He had a heart attack after swimming. And he said would you call your local Buddhist monastery and come to the hospital. I said, my gosh, it's 1:00 in the morning. But anyways, I said I'll do it. So I called them up; they were very upset at first. But it ended up three monks went to the bedside, and chanted and so forth.
And after, I asked them, I said what were you doing? And they said, well the most important thing they were doing was for the family. One of the monks said his feeling was, he had no idea about reincarnation, about what the soul is. They said the theology's great, you know, that after we die we're the same but we're not the same. We're like a fax that goes through. It's not exactly the same thing, but there is something that will go into continuation. He said in his practice he felt that all we do intentionally from compassion has some influence that's healthy.
His practice wasn't for any result, but he felt he could transfer blessings in his chant, was may I transfer the results of the good that I've done in my life for whatever journey this person is going to go on.
KEN IRELAND
I like these folks you hang out with, the more I hear about them. That's great.
Yeah, that's what we do. Yeah. I don't have any idea about reincarnation. The Buddha didn't either, apparently, from what we can read in the texts. I don't know about that, I'm not going to answer that question. But the fact that they got out of bed, and they went for the family, and they went through the ritual of the passing and the end of life. And they chanted the chants that were chanted. And they sat with the person who was passing. And they were present with him when all this was going on. It's a great thing.
And it's something that we don't do in our culture. We don't do that. We turn that over to the doctors, we turn that over to the crematorium workers. We turn it over to the hospice nurses and people and walk out of the room when it's happening, because we don't want to face it. But we want to be present for that; we want to experience what we experience. Yeah, I think there are certain points of life which are so mysterious. That really, the best way to approach them is with a ritual. This is where ritual makes sense.
I have been at the bedside with families, and the dying person is there. And I remember with Al Einhorne who called himself a Jewish convert. And he was ordained before he died, and he was a songster and a wonderful fellow who died of leukemia. But I know that he was of Jewish parentage. He was pretty Jewish himself, actually. (LAUGH) There was a tradition that the Hassadim Jew would reach over and make sure that you whispered the Shema Israel in the ear of the person who was dead.
And so I leaned over after he had died, and chanted into his ear, softly. No eyes, no ears, no tongue, no body, no color, no sound, no smell, no taste, no touch, the heart sutra . And did that again when he was cremated. You know, I don't know why I did that. I mean, it was part of the ritual, and, part of it being that he had been reading the Tibetan Book of Living and Dying by Sogyal Rinpoche, and had thought that the consciousness was leaving, and that the ears were still around after your heart stopped.
And that he wanted to hear the Teaching one more time, so I let him hear the Teaching one more time. And yet it wasn't like a ritual thing where I dressed up and lit incense; it was still part of a ritual, it was part of a very ancient ritual of dying.

MZC
Let's finish our conversation with Nyogen Senzaki (1876-1958), Zen painter, Zen poet. Three of his poems:
early in the morning
in the western sky
one star blinks at me
I love its green light
My friends do you say
you could not sleep last night?
The heat of this late summer bothered you
You could not find any cooler place?
Why did the Bodhidharma come to China?
The question, I know, also bothered you.
Wait until the evening sun colors the mountains with its gentle ray
You get more than coolness at that moment
You meet the blue eyed monk face to face.
As a Wanderer in this strange land forty-two years,
I commemorate my teacher each autumn.
Now, on the sixth floor of this hotel,
He gazes at me as severely as ever.
"How is the work, Awkward One?" He might be saying to me.
"America has Zen all the time, why, my Teacher,
should I meddle?"
KEN IRELAND
God, that's so fabulous. (WEEPING) Why did he come to California from Japan? Why did he come to wash dishes? He came to set up folding chairs in his apartment to teach zazen. He came to teach Robert Aitken meditation. He came to be our ancestor (CRYING).
I'm incredibly moved by this man's life. I think of him teaching zazen in a little apartment in Los Angeles. Robert Aitken told me about it. He started practicing really in the 40s, and he, Robert Aitken, had just gotten out of a concentration camp in Japan where he met Blithe in the concentration camp, and came back and decided that he wanted to learn meditation.
And before he went back to Hawaii and met his Japanese teachers, he started working with, Nyogen Sensaki in Los Angeles. He said the guy just had this little apartment, and when he got home at the end of the day after washing dishes in the Japanese restaurant, which is about the only job he could find; he did many jobs. He was a caretaker and a housekeeper and a number of other things.
He set up folding chairs in his little apartment near Japan Town, and people came in and he would give very basic meditation instruction. And, that's the dharma opening in this country. Unfolding chairs in Los Angeles. Amen, God bless him.
MZC
Did he come, Ken, also because he was a lover . . .?
KEN IRELAND
Curious.
MZC
. . . he was curious, and he wanted to, and he didn't come because he knew, he just came. He was attracted to come. Thank you…
What is this koan Mu?
KEN IRELAND
You want to find out about Mu? You want to find Mu? Get yourself a teacher, get the koan, go sit on the floor until your knees hurt, and discover Mu. That's what you have to do. There's no way out, only you can do it.
MZC
And it's something to be realized rather than be talked about.
KEN IRELAND
Yes. Let's go listen to the symphony. This was fun.
Not Christian or Jew or Muslim, not Hindu, Buddhist, Sufi, or Zen. Not any religion or cultural system. I am not from the East or the West, not out of the ocean or up from the ground, not natural or ethereal, not composed of elements at all. I do not exist, am not entity in this world or the next, did not descend from Adam and Eve or any origin story. My place is placeless, a trace of the traceless. Neither body or soul. I belong to the beloved, have seen the two world as one and call to that and know, first last, outer, inner, only that breath breathing human being.
-- Rumi

*********************

Leigh Brasington Interview
from Northwest Dharma News, January 1997, by Gail Gokey
The following contains excerpts from an interview conducted by Steve Johnston in October 1996 in the San Francisco Bay area. In the interview, Leigh discusses the Jhanas and gives his thoughts about the meditation retreat he will be teaching in February.

What I want to teach is the Jhanas as a meditation technique and how that technique fits into the broader spectrum of sila, samadhi, pannya: morality, concentration, wisdom.
When I say "do" the Jhanas, it's more of how you don't get in their way since the Jhanas are naturally arising mental phenomena. The doing of the Jhanas consists of not blocking them from arising. It's setting up the conditions and just sitting there watching them arise.
My favorite way of thinking about the Jhanas is thinking of Manjushri, who is the Tibetan's Bodhisattva of Wisdom. He is always pictured with a sword in his hand. The sword is used to cut through the bonds of ignorance. Doing the Jhanas is sharpening the sword.
If I give you a two-by-four and a butter knife and tell you to cut your way through it, you're going to have long, hard work. If you take that butter knife and put an edge on it first, it's still going to be a lot of long, hard work, but it'll go a lot faster. That's what the Jhanas are all about. Doing the Jhanas enables you to get your ego quiet and leaves you with a mind that can much more clearly see things as they are. Then, when you begin doing your insight practice, you can do it much more efficiently. Just because you've got a sharp sword doesn't mean you're going to wield it correctly. It's very necessary to learn at least one effective insight practice.
I prefer people to have done at least two one-week or longer meditation retreats before attending this one, and to have a close-to-daily sitting practice. Other than that, and a willingness to learn, there is probably no other requirement. I also want people who come on this retreat to realize that probably less than half the people will get into the Jhanas.
If you want to practice for the retreat, I would say sit at least for 45 minutes six days a week and be in that mode for more than a few days before you come on retreat. If you've had metta experience, you should brush that up. If you want to go in through pure concentration on the breath, you want to watch the breath in the most subtle way you can. Ven. Ayya Khema always suggests that people watch it in the nose. If you've been watching the breath in a place other than the inside the nose, it would probably be beneficial to switch to watching in the nose simply because it's going to require more concentration.
The key to getting into the first Jhana is not trying to make it happen. There's a beautiful insight that comes when you see that the first Jhana arises when you manage not to grasp, when you just let it ride.
Ayya Khema talks about the Jhanas and how they're becoming a lost art. This knowledge needs to be passed on. She thinks I have enough knowledge to pass it on. I don't know the way to the top of the mountain. I don't know how to tell people to get enlightened. I do know eight altered states of consciousness, each of which produces more concentration than the previous one. A concentrated mind is much more useful in gaining insight than an unconcentrated mind. This is all I really know. Hopefully, somebody can take their concentrated mind and gain a lot more insight than me and show me the way.

Leigh Brasington has been practicing meditaion since 1985 and is the senior American student of Venerable Ayya Khema. He lead a Meditative Absorption (Jhana) retreat February 7-13, 1997, at Cloud Mountain

**********************

Loving and learning
Interview by Atiya Achukulwisut (Bangkok Post, December 18, 1999)

Buddhist monk Matthieu Richard talks about his way of life and the conversations with his father that resulted in 'The Monk and the Philosopher', recently published in Thai.
"Happiness is knowing we have been able to spend our life actualising the potential that we all have in us, and to have understood the true and ultimate nature of the mind," so writes Matthieu Ricard in his book The Monk and the Philosopher.

The statement is a reflection of the author's life.
Born in France, Mr Ricard left behind what other people might consider immense potential. He finished a PhD at the age of 26 and had a promising career in science in the research team of Francois Jacob, eminent winner of the Nobel Prize for biology, at the prestigious Instituit Pasteur. Instead he became a Tibetan Buddhist monk in the Himalayas.
Mr Ricard is now 53, and has been in the monkhood for more than 20 years.
He notes that, "For someone who knows to give meaning to life, every instant is like an arrow flying towards its target."
His latest book, The Monk and the Philosopher is a collaboration between Mr Ricard and his father, Jean-Francois Revel - a prominent philosopher and newspaper editor. The book is an international best-seller.
The dialogue, based on conversations between the father and son during their 10-day retreat in Kathmandu, Nepal, explored some of the most fundamental questions in human belief systems. Is Buddhism really nihilistic at the core, as many Westerners believe? Why has it become increasingly popular in the West? What is the real meaning of incarnation?

Mr Ricard recently visited Thailand to speak at the book launch - and talked about his life.

Q: What prompted you to leave a promising career in science and become a Buddhist monk?

A: I didn't become a monk right away. In the beginning, I met a remarkable spiritual teacher. I went back and forth between the Himalayas and the Instituit Pasteur, where I was doing some scientific research.
For me, the discovery of the meaning of life or a truthful path was a natural continuation of natural science. After some years of going back and forth, I decided I would like to devote my time to the deeper contemplation - a study of how the mind works.

Q: What sparked your interest in Buddhism in the first place?
A: A friend of mine made a documentary movie about some spiritual teachers who fled the Chinese invasion of Tibet. I saw the images and I was struck by what we didn't often find in Paris-great sages. You may come across great intellectuals. I myself grew up among great scientists, artists, philosophers and musicians. They are very good in their own fields. But that doesn't make them better human beings. You can be a great pianist and very angry. You can be an accomplished scientist but a bad family member. A spiritual master cannot be like that. You can't be a spiritual teacher and angry all the time, because that goes against the main point of being a sage. Therefore, I felt I could find in great spiritual teachers a whole picture of what constitutes a very good human being. They become a living example by what they are, not what they are good at.
When I was going back and forth, people around me didn't know what to make out of it. They could see I was interested in those things, but they didn't know to what extent. When I decided to stay there, my father was a bit shocked. As for my colleagues at the Pasteur institute, perhaps they just thought it was up to me. I went back to the institute last year after 25 years. I met my old colleagues. I found out some of them are still doing exactly the same subject of research. We sort of looked at each other. Seeing them, I thought, what I am doing is quite okay!

Q: What made you stay?

A: It wasn't just one thing. You can attain an inner peace by sitting near a lake, going for a walk, reading a nice book or listening to music. But that is superficial. You just calm down the waves. But to be disturbed by no more waves, you have to calm down the wind. The wind comes from the mind's work. Why do we have destructive emotions, like anger, pride, jealousy, greed, or attachment? That is the wind. You can calm it down a bit. But unless you deal with the root cause, it will blow again and destroy us. We should explore deeper into the root of the mind and where these emotions come from. That is what I call contemplative science - how the mind works. What is the nature of reality? Why do I think this is beautiful? Is it beautiful for everyone? Is it my mind that makes it beautiful?
It is a deep transformation. If that happens, then you have a different vision of reality. It will remain no matter how pleasant or unpleasant, favourable or unfavourable the circumstances are. It gives greater freedom, too, to interact with others because you are no longer dealing with them in terms of attraction, repulsion or other strong feelings of ego.
The transformation takes time. We have been used to functioning that (old) way for so long. It is like a roll of paper. If you roll it for a long time, and you try to flatten it, it will curl again.

Q: Is there any conflict between your background in Western scientific belief and a quest for the nature of the mind?

A: Science means knowledge. Why should it be limited to phenomena which we can measure? Authentic knowledge is science of the mind. Do we want some kind of fulfillment or happiness? Do we want to be free from fear and frustration? That is as interesting to know as how the plants grow. It is even more central to us because we deal with our mind all the time from morning to evening. If you know how it works, it suddenly makes a difference.

Q: The impression from the book is that your father does not seem convinced Buddhism can do that for all of us.

A: My father is a brilliant intellectual. He appreciates completely the wisdom aspect of Buddhism. Experience aside, many people have some kind of inertia. It is a bit like swimming. You can be an expert in the theory of swimming but if you don't go into the water to actually swim, the knowledge is not complete.
I have come closer to my father by doing this. We've developed a kind of complicity. The only thing I don't understand is what makes him not want to try something. He doesn't have to become a Buddhist, of course. Maybe there is something hidden in every sentient being, including him, a deep reflection, that they do not feel like speaking about or bringing outside.
When the book came out, we had a conference in Japan mostly with scientists. One astronomer mentioned that we are fearful to go really deep into ourselves to see what we are. There is a kind of fear to cross that threshold. Why? Maybe it is the ego. It may cause us a lot of troubles but we just love the enemy (the ego). Or perhaps we are not sure if our life will be worse without it. It is better to live with an enemy we know. I think it is a pity. From my experience, it is very liberating.

Q: Do you believe in incarnation?

A: What is more interesting is whether consciousness is only the brain. If it is just the brain, then no person can be reincarnated. Many neuro biologists would say that it is just the brain. Slowly, more and more complex structures happen and eventually you are conscious of something, beginning with intuitive reaction then more sophisticated intelligence, such as a value in beauty.
That is very much like a computer. But can computers think? Computers may work so well they can beat Garry Kasparov at chess but there are a few things they can never do. A computer never asks what is going to happen to it after the plug is pulled. It can beat Mr Kasparov at chess, but it has no idea how to play chess. I believe the faculty of consciousness is a sign that there is something more than a computer-like brain.
There is a cause and effect to everything. You sow rice seeds, you get rice plants. Each moment of consciousness must he preceded by a cause of conscious nature. That is why Buddhist philosophy stipulates there must be a stream of consciousness just as there is a stream of matter. A mind cannot create a stone and a stone cannot create a mind.

Q: Have you had doubts about the path you chose?

A: When you have a sense of direction, even if the way is long and sometimes hard, you have the joy to walk. You know that every step brings you closer.
I would have had doubts if I found something inconsistent, stupid or crazy. But I did not. I know the extent of my defects. I have no doubt about them. I want to change them.
Maybe I have some doubt about my capacity to do it. But then, it depends upon my determination and strength. I may have to acknowledge that I don't have enough but that is my condition. It is nothing to get depressed about.
As far as the rightfulness of the means, so far I have found it wonderful.

Q: How did the book project begin?

A: I hadn't gone back to France at all after several years in Tibet. I visited my father from time to time. We discussed mostly the situation in Tibet. He is a writer (Mr Revel was a former editor of the news magazine Le Point), one of the first French journalists to write about Tibet in the 1970s when nobody really cared about it.
We never thought about an in-depth project (together). I respect his position and do not want to impose my own ideas on him. When a publisher called me in Nepal and proposed I have a dialogue with my father, my thought was he wouldn't do it. But I told the person to check with him. I thought I would never hear about this again.
One month passed. The publisher called again from France and said he was happy to do it. I was both happy and surprised. He came to Nepal. We spent 10 days in the mountains at a small inn. We walked in the forest and talked in the morning, or in the afternoon. It was the first time we had had a deep discussion. It was very fresh and constructive for both of us.

Q: You mentioned that one of the book's proposes is to promote the cause of Tibet and to help the country find peace at long last.

A: The only hope is the fact China itself is changing. The international community obviously does not have the political will to come up with a strong position, not as strong as they have for trade issues. Most Western leaders only tell the Chinese that they have to preserve Tibetan culture and not be too harsh in terms of human rights, but no one has said that according to international law they are occupying a foreign country.
Tibet is a nation that ruled out, from the beginning, the idea of using terrorist activities. They are penalised for that. If some groups do something bad, plant bombs or blow up plants, the international community says we have to solve the problem. So how about those nice Tibetans? Well let's hope for the best for them.
What Tibetans are asking for is very simple. The Dalai Lama has said more than 100 times: Let's consider if we could be part of China and have some kind of autonomy of internal affairs, maybe like Hong Kong, so Tibetan people can live the way they want. That is not asking too much.

Q: What are the best things you received from the collaboration with your father?

A: I think it helps me clarify some ideas, which I have learned from my teacher and try to practice. But when you have to express them in front of a strong Western intellectual, you have to be clearer and much more precise.
I was also very happy to share these ideas because they are dear to me. I don't want to keep them just for myself and pretend to be a great practitioner. I treasure what I believe. I am also happy I could share it with a lot of people through the book.
My father is an open person, not like many people in the West who just bash down religions. He has his ideas but he listens. He answers. He asks questions. I think that is intellectual integrity.

*********************

No Escape for the Ego
An interview with Venerable Master Sheng-yen
by Carter Phipps

Introduction
When it comes to the topic of ego in the spiritual life, few people have staked out the territory like the Zen masters. To read their stories is to enter another world, one where commitment, humility, devotion and insight take on larger-than-life significance and one thing matters above all else: to slay the ego once and for all, and in doing so to achieve enlightenment, to deeply realize one's Buddha-nature in this life. Indeed, few people could honestly claim the fortitude of spirit required to withstand the ego-destroying tactics of the Zen teachers of yore who, in stories that have become legends, resorted to often outrageous acts of enlightened wisdom in order to shock, jolt and awaken their students from the nightmare of ego-centered existence. Zen Buddhism, it would seem, has never been a path for the faint of heart, a testament, perhaps, to the First Zen Patriarch, Bodhidharma, who sat facing a wall for nine years to demonstrate his commitment to the path of enlightenment. Even in the modern era, we find echoes of Bodhidharma's resolve in the stories of contemporary practitioners, like the now-classic My Struggle to Become a Zen Monk by Morinaga Soko. In his efforts to gain admittance into the monastic life, Soko spent three days crouched beneath the wooden steps at the entrance to one Japanese Zen monastery enduring what is called "niwazume," a test of character designed to ensure that only the most determined make it through the outer gates. Exposed to the cold wind and snow, he withstood verbal assaults, psychological pressure and even physical beatings before he was finally able, with numb legs and a bloody face as evidence, to convince the monks inside that he had the humility and resolve to take up the austere life of a Zen monk. Those who aren't willing to pay the high price of slaying the ego, in other words, need not even apply.
So from the moment we decided to present an investigation of the nature of ego in this issue, we began a search for a Zen master who could speak from his own experience of the trials of this rigorous path beyond the ego a search that eventually led us to the venerable master of Ch'an Buddhism Sheng-yen. The word "ch'an" is the Chinese translation of the Indian term "dhyana," a Sanskrit word meaning meditation, and as Buddhism eventually spread from China to Japan, it was translated there as the term much more familiar to our Western ears: Zen. Sheng-yen, according to his most recent book, Subtle Wisdom, was initiated into Ch'an Buddhism at the age of thirteen when he left behind home and family to take up the robes of a novice in a Shanghai monastery, an austere and traditional temple that would be his home for the next six years. In 1949, however, a wind of revolution and change swept through China, and the Communist takeover of the mainland cut short his career as a young monk. Conscripted into the Nationalist army, Sheng-yen soon headed for Taiwan, and it was there, almost ten years later, that he experienced a powerful spiritual awakening at the age of twenty-eight. It was, he says, "the most important experience in my life up to that point." In a story that could have been taken from the pages of classic Zen literature, the young Sheng-yen was on a brief sabbatical from the military, visiting local Ch'an teachers when, while up late one night meditating, he found himself sitting near an older man, also a guest of the monastery, who impressed Sheng-yen with his steady and peaceful demeanor. Asking the elderly monk if he would answer a question or two, Sheng-yen proceeded to pour out his heart for two hours, giving voice to all of the questions that no one had been able to help him with during his many years of spiritual practice. And at the end of each question, the monk, whom Sheng-yen would later find out was actually a revered Ch'an master, would simply ask, "Is that all?" Finally, Sheng-yen had exhausted his litany of questions and, in a moment of confusion, hesitated, not knowing what to do. Bang! The monk struck the platform they were sitting on and roared, "Take all of your questions and put them down! Who has all of these questions?" The effect on Sheng-yen was immediate and profound. "In that instant all of my questions were gone," he writes. "The whole world had changed. My body ran with perspiration but felt extraordinarily light. The person I had been was laughable. I felt like I had dropped a thousand-pound burden." The words of the Buddhist sutras [scriptures], which once seemed foreign and impenetrable, now came alive as Sheng-yen's own experience. "I understood them immediately, without explanation," he writes. "I felt as if they were my own words."
As fascinating and inspiring as this story is, what was most intriguing to us, from the perspective of our investigation of ego, was not the experience itself but the way Sheng-yen responded to it. In a time when it seems that so many, after similar experiences of profound awakening, have quickly assumed that the dangers of the ego have been forever left behind, Sheng-yen came to a very different conclusion. While knowing that his perspective on life had radically changed, he also recognized that his "vexations," or the character deficiencies arising from his own ego, had not disappeared and could, under the right circumstances, still cause him trouble. It was not time to rest or to teach, he decided far from it. It was time to practice to rededicate himself to spiritual purification with all of the resolve, inspiration and determination that this deep glimpse into his true nature had given him.
So with deepened faith in the reality of a life beyond the attachments of ego, Sheng-yen once again took up the robes of a bhiksu [monk], obtaining early release from his military duties. He began to study with a well-known Ch'an master named Tung-chu, who had earned a reputation as a very demanding teacher, even by the high standards of Ch'an. Tung-chu pushed Sheng-yen hard, one day challenging him to perform prostrations, and then days later reprimanding him for the same; telling him to write, and then tearing up his essays when they were completed; even going so far as instructing him to close up the door to his room just so he could create a new one in the opposite wall.
Grateful to have returned to his interrupted monkhood and discipleship, and for the freedom to devote himself full time to spiritual life, Sheng-yen pursued his spiritual practice with great intensity. On his own initiative, he eventually decided to begin a three-year solitary retreat high in the mountains of Taiwan. Living in a small cliff-top hut with no running water or electricity and subsisting on wild potato leaves that he grew himself in his backyard, he worked to uproot the deep vexations of his own mind, to bring the full power of the Buddhist dharma to bear upon his attachment to the ego. Beginning with a half-year of prostrations (doing one for each of the almost 80,000 characters in the Lotus Sutra) he then concentrated on sitting meditation and, in his spare time, wrote and completed two books on the Buddhist teachings. Three years later, feeling at home in the quiet calm of solitary practice but convinced that his efforts to cultivate freedom from "greed, anger, arrogance and ignorance" were still incomplete, he decided to double the time of his retreat, extending to a total of six years this period of seclusion, contemplation, practice and study.
It was after he emerged from this second retreat that Sheng-yen began to feel that the time was right for him to take up the mantle of a Ch'an teacher in his own right and spread the Buddhist dharma. But having long been troubled by the extreme lack of education he had often seen among the monks and nuns of Taiwan, he first set out to obtain the formal schooling that he himself had never received during his years of retreat and practice. He headed for Japan and attended a university there, immersing himself in the subtle intricacies of the Buddhist dharma, earning a doctorate in Buddhist philosophy and, in the process, studying with well-known teachers from almost all the major schools of Japanese Zen. Later, accepting an invitation from a Buddhist association in the United States, he headed for the unfamiliar territory of America, where he launched a teaching career that would grow rapidly until it eventually encompassed communities of students from both the West and the Far East.
Currently Master Sheng-yen resides primarily in Taiwan but spends several months each year visiting his centers in the United States. As the founder of a liberal arts college near Taipei as well as several monasteries and meditation centers, the author of more than ninety books in ten different languages, a lineage holder in the two major schools of Ch'an Buddhism and personal spiritual guide to thousands of devoted students, Sheng-yen is a master who wears the threads of a great many responsibilities in the fabric of his simple monk's robe. He is credited by some for sparking a revival of Chinese Buddhism, a tradition that is today in exile from its home country a place where Sheng-yen may visit but cannot teach and where an illicit underground network is the only way to distribute Buddhist literature to the population. In a role that has some similarities to that of H.H. the Dalai Lama, Sheng-yen has spent much of his life working hard to help save and invigorate what was once an extraordinarily rich tradition, despite its continuing oppression in the very homeland where it first blossomed more than a thousand years ago.
Morinaga Soko, in writing about the lessons learned during his ordeal outside the Zen monastery where he became a monk, states, "Until you have subjected yourself to some discipline, you should not put too much faith in your own willpower. When I saw my own will crumbling at the monastery entrance, I suddenly felt I understood the reason for niwazume. As one crouches by the bench on the dirt floor, one's resolve is put to the test time and time again. . . . At the entrance to the monastery, I had learned the meaning of the courage which has its roots in faith and which remains undaunted whenever resistance is encountered." It doesn't take more than a cursory look at the spiritual life to recognize that to truly free oneself from the fetters of the ego takes courage, determination and resolve in no small measure. Sheng-yen, it seemed, was someone who had spent much of his life attempting to cultivate these very qualities. Indeed, he was a man who had been tested in the fire of the Zen path, who had given his heart and soul to a tradition that demands much of the spiritual aspirant and has a reputation for offering little if any quarter to the needs and concerns of the ego. So what would he, as a person who had truly lived and breathed the experiences most people only read about, have to say about this ancient enemy of the spiritual life? Would he be filled with the fire, intensity and passion for ego death that so many in his lineage have expressed down through the ages? Or would he, in his current role as a teacher and the public face of Buddhism to thousands of people around the world, be more palliative in his relationship to the ego, more accepting of those for whom the idea of ego death is going just a little too far and more accommodating to a Western spiritual culture in which the ego seems to have fallen from its preeminent position as the one and only obstacle between us and the gates of nirvana?

Interview
Sheng-yen talked with me one November afternoon on the second floor of his Queens, New York, meditation center. As the soft chanting of the Buddhist sutras drifted up through the wooden floorboards from the room below us, we sat together and spoke through a translator for an hour.
WIE: What is the ego according to Ch'an Buddhism?
Master Sheng-yen: In Ch'an Buddhism the idea of ego revolves around the idea of attachment or clinging. The ego originally does not exist. It is created as a result of attachment to the body and attachment to one's ideas or one's own viewpoint. But because both the body and the mind are impermanent and constantly changing over time, our attachments to them are always changing as well. And as these attachments change, the ego also changes. So from the perspective of Ch'an, the ego does not exist in the sense of being a permanent, unchanging entity. The ego does not exist independent of one's changing attachments to one's body and one's ideas.
WIE: What does it mean to go beyond the ego?
SY: There are two different ways to accomplish this transcendence of the ego. One is experiential, through experiencing the transcendence of the self. And this can be done through practice, the practice of sitting meditation and the investigation of a koan [paradoxical question]. It is possible to attain this experience without a practice, but that's very rare; most people need to do the practice. The point of this kind of practice is to essentially push the ego into a corner so that it has nowhere else to go. It cannot escape anywhere.
So the ego and the method that you are using to transcend the ego are in direct opposition to each other. As I said, the ego is based on attachment our attachment to the body and to ideas. Therefore, the method of transcending the ego is to deal with this attachment, to put down this attachment. When the ego is cornered and has nowhere to go, the only thing one can do is to put it down. And when one puts down the ego, then that is enlightenment.
WIE: Could you explain further how facing into a koan helps to "corner the ego"?
SY: In this method, you're actually not trying to solve the koan. Rather, the method involves asking the koan to give you the answer. A koan may be like, "What is wu [nothingness]?" So you keep asking and asking the koan to give you the answer to that question. But actually, it's impossible to answer. Of course, in the process of asking, your mind will give you answers, but whatever answer you get you have to reject. And you just stay with this method keep asking and keep rejecting whatever answer comes up in your mind. In the end you will develop a sense of doubt. You will not be able to ask the koan anymore. In fact, it'll be meaningless to ask anymore. Then there is nothing to do except to finally put down the self and that is when enlightenment appears in front of you. But if you ask the koan and you simply get tired, if you can't get an answer and so you just stop, that's not enlightenment. That's just laziness.
The second way to transcend the ego is the conceptual way. It happens when there's a sudden and complete change in one's viewpoint. It can happen, for example, when one's reading a sutra [Buddhist scripture] or listening to a dharma talk. In an instant, one can become enlightened. But for this to really work, a person has to already want to know the answer to the question, "What is ego, what is the self?" They have to already be engaged with this question in their own mind. And then, when they come across a particular sentence, they can suddenly recognize the answer and instantaneously realize enlightenment. One very good example is the Sixth Patriarch, Hui Neng. He heard one sentence from the Diamond Sutra and got enlightened. However, for people who never think about these issues and questions in their daily life, who don't care about what the ego is and have no desire to know what the self is, this won't work. Listening to a dharma lecture or reading the sutras isn't going to help them.
WIE: What is the role of the teacher in liberating the student from his or her ego?
SY: First of all, the most important thing is that the student has to really want to know what the nature of the ego is. They need to have this burning desire to know. Then, what the teacher can do is to give the students a method or a tool to investigate and show them how to go about practicing the method. Many students may have a method and not be able to use it well. So the teacher can show a student how to use their method properly and can also show the right attitude and conceptual understanding they need in going about their practice. And if the student has a strong desire to understand the nature of their real self, then the method will be helpful. They will be able to see that this self that's based on attachment is illusory. It's not real. And when they realize this, they will also see that there's no such thing as the ego.
WIE: In your recent book Subtle Wisdom, you write, "Sometimes the mind experiences something that it takes to be enlightenment, but it is actually just the ego in a very happy state." Could you explain the difference between these two experiences between genuine enlightenment and a condition where the ego is simply, as you said, "in a very happy state"?
SY: The experience of happiness can also be a part of enlightenment; a person can feel happy whether they are enlightened or not. But usually when one is in this blissful, happy state, it is because, in that moment, one is no longer feeling burdened by one's body or by one's mind and emotions, and so one feels very at ease. However, this is not the same as liberation. One may feel very light; it doesn't mean anything. A very peaceful, blissful, happy feeling is not the same as enlightenment. Enlightenment is not being attached to any viewpoint or having any attachment to the body. There's no burden at all, and that's why one would feel happy. For example, Shakyamuni Buddha, after his enlightenment, sat under the bodhi tree for seven days to enjoy this happiness, this dharma joy from his liberation. But one can feel happiness whether one is enlightened or is not enlightened. So we need to be able to distinguish.
WIE: In your book you go on to say that this experience of the ego being in a very happy state could occur because "the ego may even be identified with the universe as a whole or with divinity." Could you explain what you mean by that?
SY: That feeling of unification with the universe is actually one kind of samadhi [meditative absorption], a result of a deep state of concentration, and when a person is at this stage, they recognize that the entire universe is the same as themselves. What happens is that one expands one's small ego outward, to include all viewpoints, to include all of the universe and everything in it. So at this point, one would no longer have individual selfish ideas or individual selfish thoughts that normally arise from the narrow, selfish ego. In fact, one may experience a tremendous power that would result from this samadhi, a power that would come from the idea that "the universe is the same as me." People who have had this kind of realization can often become very great religious leaders.
But the Buddha, after his enlightenment, did not say, "I'm the center of the universe." Neither did he say that he represented the entire universe. What he said is that the Buddha is here to encourage all sentient beings to see that ego comes from attachment, and if we can all put down this attachment, then we will be liberated. And so the Buddha sees himself as a friend, a wise friend to all sentient beings, encouraging them to understand that ego comes from attachment and encouraging everybody to practice, to put down this attachment.
So in the Buddha's nirvana, there's no more arising and no more extinguishing. There's no self no big Self, no small self and that is the true enlightenment. That's the enlightenment of the Buddha.
WIE: So if an individual is identified with the universe as a whole, is there still, in that case, an ego attachment that the individual hasn't given up?
SY: Yes.
WIE: Some of the great Ch'an and Zen patriarchs were reputed to have been very fierce teachers who would go to great lengths and use very extreme measures to liberate their students from their egos. In your books, you have written about how some of your own teachers were very tough with you as well. Is it because our attachment to the ego is so deep and so strong that these revered masters needed to employ such extreme measures to get their students to go beyond the ego?
SY: Actually, not everybody needs these harsh methods. The kind of method that is used has to match the needs of the individual student and the condition of the moment. Timing is very important. For example, when I teach my students, I only use harsh methods when it is necessary. Most of the time I use a lot of encouragement, especially for beginner students. It is for those who have been practicing for a while, who have a lot of confidence in their practice already but who still have this attachment to the ego, that I will use some harsher methods to help them to move forward. But it takes a very experienced, very good master to know when the time is right to use such methods.
WIE: Another passage from your book reads, "If your sense of self is strong, solid, and formidable, then there is no way you can experience enlightenment." What do you mean by this? Why is it difficult for a person with a strong sense of self or what Westerners would call "a strong ego" to experience enlightenment?
SY: It's not necessarily true that people who have a very strong ego cannot be enlightened. In fact, those who know that they have a strong ego may, in some cases, actually be very good candidates to practice the Buddha-dharma. You see, there is a type of person who is very egocentric yet at the same time has a strong desire for enlightenment. Because of this strong desire, they are naturally going to be very unhappy and dissatisfied with having a big ego, and that attitude will be good for their practice. When you have such a strong ego, you have to be willing to do something about it. So someone like this could be a good candidate for practicing and studying Ch'an.
Then there are also individuals who have what we would call a weaker or softer ego. This can help them, but only if they still have a real desire to deal with their ego. If they don't, they are not going to be any closer to enlightenment because they won't have any confidence in the practice. They won't have diligence in the practice. But if an individual has a weaker, softer ego and still understands that they need to practice diligently to deal with it, then we could say that these individuals, because they have both a strong desire for liberation and a smaller ego, are closer to enlightenment.
WIE: Today many Western spiritual teachers believe that traditional spiritual paths, including Buddhism, do not properly address all the needs of the modern seeker. In particular, they feel that people may need psychotherapy to supplement their spiritual practice in order to work out many of their emotional attachments and problems with their ego. Do you feel that the Ch'an path is incomplete when it comes to addressing the suffering of the modern seeker and that a person would be well advised to consider this dual approach psychotherapy and spiritual practice in their pursuit of enlightenment? Or is spiritual practice alone, if it's sincere and diligent, sufficient to free us from the ego?
SY: There are two different issues here. First, individuals who have very severe psychological problems should not use the Ch'an method. It's not good for them. If they just want to learn the beginner's sitting meditation, we will teach them and they will reap benefits from that, such as improved health. However, a person with severe problems should get a doctor to help them recover before they begin the practice of Ch'an.
But generally, for individuals who do not have severe psychiatric problems, Ch'an practice is sufficient. There's no need to get help from a psychiatrist or a therapist. In fact, sometimes psychiatrists or therapists come and seek help from me.
WIE: In the last thirty years, there have been many powerful teachers who have had profound spiritual understanding and experience and have attracted large numbers of students, but who eventually fell from grace due to corruption and scandal, sometimes in very shocking ways. Is it possible that spiritual experience and understanding could, in some cases, actually empower the ego?
SY: It's hard to say. I don't really want to comment on this. It is a problem. There are some individuals who think that they are enlightened, that they are liberated, and they also have the idea that after they're liberated, they do not need any morality; they do not need to uphold the precepts [basic obligations undertaken by Buddhists] anymore. And according to my own understanding of Buddhism I can only speak for myself here we follow Shakyamuni Buddha and if we look at the Buddha after he was enlightened, he didn't go and drink. He didn't go and hang out with women, sleep around and cheat people out of their money. And so that is what we follow. The Chinese Ch'an masters emphasize the importance of upholding the precepts.
WIE: For everyone, teachers and students?
SY: In the sutras, the Buddhist scriptures, they say that if you are really genuinely enlightened, you will naturally uphold the precepts.
WIE: You are a revered teacher with students in Taiwan and also Western students here in America. Some of the Western spiritual teachers and psychologists we have spoken to for this issue have said that the ego of Westerners is different from the ego of Easterners that Westerners are more attached to an individual self and personal identity. If that's true, then theoretically, it should generally be easier for Easterners to get enlightened than it is for Westerners. Do you agree with that? Is that your experience?
SY: This is not necessarily the case. It all depends on whether you have the desire for enlightenment whether, as I was saying, you really want to understand the nature of the ego.
WIE: You're saying that's the key to success?
SY: Yes, that's the key. You may have a weak or small ego, but if you don't care about these things and you don't have a strong desire, then you're not closer to enlightenment.

*********************

On confidence in the Dharma : an interview
His Holiness the Sixteenth Karmapa

ONE EARLY MORNING [in 1980] His Holiness the Sixteenth Gyalwa Karmapa generously granted an interview to the readers of Densal. What follows is the text of that interview, word for word, as translated by Ngodup Tsering Burkhar. In it, His Holiness touches on many important aspects of spiritual practice, the Kagyu lineage, and life in the world today for the Dharma practitioner. It is a timely and most valuable teaching for Buddhists and non-Buddhists alike.
Densal: This is your third tour to America. Do you have any observations you would like to share about it, and about the growth of the Dharma in the United States?
H.H.: The responsibility of the teacher is to always give the teachings. It doesn't matter that only a short time has passed, or a long time has passed; what matters is that the teachings are continuously given. Sometimes it may seem to be more appropriate to teach because most people are at leisure and have a lot of time, and it appears to be a good time to give teachings. Maybe at other times it may appear that the teachings should not be given because people are busy and perhaps they are not interested. It is important not to discriminate in this way as to time or to place, but to make the teachings constantly available. If only one person is interested, the teaching must be made available. Whether there are just a few people, hundreds of people, or even millions of people interested, the Dharma teaching must go on without any sense of satisfaction or discouragement. The Dharma teaching must continue at all times, transcending the appearances of the time.
Another situation that might arise is that because of time or what we may have done or accomplished, we feel that maybe now we should stop practicing or listening to teachings. This is not the Dharma path. You keep going. That is the bodhisattva's way. As long as it benefits even one being you have to, without any sense of discouragement, go on.
If you have 100 percent dedication and confidence in the teachings, then every living situation can be a part of the practice. You can be living the practice instead of just doing it. Regarding the establishment of the Dharma anywhere, it happens as a result of what takes place on an individual basis; it is the practitioner's responsibility. It is understanding the Dharma properly, respecting the truth of Karma, the truth of cause and effect. One respects the truth of the teachings and knows that this is something that one must live up to and preserve. But if beings fail to respect the truth of the teachings, or the truth of cause and effect, which is also the truth of the teachings, then that would not further the establishment of the Dharma.


Densal: More than anything I think one problem Westerners in the Dharma face is the desire to achieve ultimate spiritual realization in this life, coupled with the fact that they must work full-time in the world at various time-consuming careers. How can one handle this life situation and travel the Mahamudra path effectively? It has been said that action can also be meditation. Could you please clarify this?
H.H.: We have for many, many lifetimes been caught up in the samsaric existence because of fascination with our habitual patterns, and are compelled for this reason to continue the patterns in the same way that we had in the past. At this time in our lives, as a result of whatever virtuous actions that we have formerly performed, some degree of awakening has arisen. A very precious waking state has come about in our consciousness, and that is our connection with the Dharma.
Once we are connected with the Dharma in such a way that we have some understanding, we also have some sense of direction as to where we are going. It is like wanting to go to California. You know that there is a particular train, and that train takes you to California. You have that understanding. Then it becomes a matter of individual realization of the need, the urgent need perhaps, to get to California. Then there is boarding the train. It is possible for one to do such a thing, to make the decision, "Yes, this is it, I am going to travel."
And there is the greater possibility that you will get to California once you have boarded the train. And maybe there are certain possibilities of your not getting there, of something happening on the way. And if something happens on the way, an accident or something, you know that the possibilities of getting to California are still there. You haven't yet reached it, but you have some sense of direction as far as your knowing that it is possible to get there.
And this is, one could say, like the blessings of the Dharma: that even though one is not able to realize enlightenment in one lifetime, the blessings of the practice and the Dharma are continuous. There is a sense of optimism about the possibilities of getting to California, even though you haven't arrived. That is the same situation that takes place in your Dharma life. The blessings continue, even though you could not attain the experience of enlightenment in this lifetime.
If you have a proper direction, in the state of the Bardo, there occurs what is like the meeting of the mother and son. It is an opportunity to rely on your own ability to understand and to realize, and utilize the Dharma blessings at that moment in the Bardo in order to recognize the "mother," so to speak. Your experience of clear light takes place. And realization is very much possible. This has been witnessed by the teaching and is guaranteed by the teaching. It is definitely possible for people to experience that sort of thing.


Densal: Even though people find themselves caught up in a samsaric whirlwind, they can maintain their equanimity and attain the realization in Mahamudra?
H.H.: Yes, it is possible. It is a matter of confidence in the teachings. If you have 100 percent confidence in the teachings, your realization is not purely dependent on just formal practices. If you have 100 percent dedication and confidence in the teachings, then every living situation can be a part of the practice. You can be living the practice, instead of just doing it. But the more you lack the confidence, the more you will find yourself separated from the Dharma.


Densal: Part of the training within the Kagyu lineage is the three-year, three-month retreat. At KTD already there is one nun in such a retreat. Does the monastery project include plans for a three-year retreat center here in the U.S.? And could you explain how the three-year retreat relates to the American people, many of whom are oriented toward activity and have difficulty in seeing the practicality of such an undertaking?
H.H.: Actually the monastery project itself is to facilitate the practice. The purpose of the monastery is to be able to help create a proper environment, to establish a solid, structured environment for the practice of the Dharma. And the practices of the three-year retreat are definitely included. Not only are they included as far as the outer facilities are concerned, but in terms of providing the basic needs. I have in mind much concern as to how that could be worked out.
Before I leave this country, it is my plan and vision to talk to many people about this, to awaken their interest and make Dharma benefactors aware of the situation. If they could help and support the three-year retreat, it would be very beneficial for them, though they themselves might not be able to do the three-year retreat. They will actually accumulate equal benefit, as much as those who will do the three-year retreat. As far as those who are able to do the retreat are concerned, of course they will have tremendous benefit coming as a result of the practice. And it is also my vision that gradually people will find more time in their lives to do retreat, as we are able to provide the facilities for people to take advantage of. Unlike some time ago, we now see many people trying to make time for the practice, and many are seeing the possibility for making time. There is much more interest.
I see in the future many people doing these kinds of practices, and they will be given the opportunities to do them. For those who are not able to do it, it is possible for equal experiences to be achieved. The thing is that during the three-year retreat, one does a lot of meditation practices: the fulfillment-stage and development-stage meditation practices, and the recitation of the mantras, and many other things. No matter how ignorant a person is, it is guaranteed that the three-year practice will bring a reasonable experience of the teachings to the mind of the person. The retreat situation involves relating to the preliminary practices as well as the main practices, and when one has that kind of grasp of both the preliminary and the main practices, certain experience is guaranteed. Now, if someone has a greater wisdom and capacity for penetrating the teaching, then even without doing a three-year retreat, it is possible for one to experience definite understanding and realization.


Densal: Do you see more Westerners being trained as qualified teachers and holders of the lineage?
H.H.: Yes, I feel it is important that people become able to take care of the Dharma, and are not always in a position of dependence. I am confident that we will be able to produce such teachers, and producing such people is very important.


Densal: You are the recognized head of an important lineage, the guru to many thousands of people. There are frequently misconceptions about what a spiritual teacher is in this tradition. Could you explain, in your own words, what is the guru?
H.H.: I will tell you what a guru is not. That is, somebody who is interested in fame, teaching for the sake of notoriety, for the sake of wealth; or one who, when he is in the presence of many people, puts on all the qualities of goodness that might be appropriate for a teacher: wearing the mask of the Dharma, so to speak, using whatever appearances are necessary, but insincerely. The reality is much different. When he is away from a crowd of people, he actually needs as much as anyone else needs, wants as much as anyone else wants, if not more. Discrimination between beings, selfishness, all these negative states are present in him, and have not been transformed. That is, unfortunately, what takes place too frequently these days, and that brings a negative influence to the spiritual path and spiritual friends in general. It fosters a negative view altogether among people who are ignorant as far as who should be regarded a true guru. They might encounter someone who is a true teacher, but they fail to relate to him because of having gone through other experiences of the negative nature I have already explained. Now these days, actually, to encounter a very true guru is difficult.
So if you encounter a spiritual friend who is a guru, look to see that he is willing to help himself at the same time that he is willing to help you; also, if he is capable of helping you. He should have a desire to help you as much as he helps himself. In times like these, such a one may be considered a spiritual friend. The actual quality of a guru should be a willingness to work for the benefit of others, along with the ability to work for the benefit of others. And there are different levels of gurus, as well. There are different degrees of being able to benefit others, different degrees of having the strength and the wisdom to reach beings. It is difficult, then, to be specific about qualities of gurus, yet we can come to the conclusion that as long as the teacher is selflessly benefiting beings somewhere, that this person may be worthwhile to be recognized as a spiritual friend and as a guru.


Densal: Your Holiness, do you have any special message you would like to give to your students, disciples, and interested people who will read this newsletter; some advice, perhaps, in these difficult times?
H.H.: The practice of the Dharma is a matter of serious importance; people have to realize that. It is a precious opportunity that has come about, one that has never come about before. It is very precious because it is so rare. The time in which you can use the opportunity is quite limited, and this makes it even more precious. I would like to repeat that a rare and precious opportunity which has never come before has manifested when you find the Dharma in your life. It is a historic situation, a landmark. But the time to take advantage of this opportunity, again, is limited. Therefore we have to realize the great value of the opportunity. The best way to do this is to engage oneself in the practice of the Dharma as sincerely as possible. Otherwise the opportunity could fade away. There is this danger, most certainly, that one could lose this opportunity. It could become more and more distant, and this would be a very unfortunate situation.
It is like crystals that are put together with a piece of a diamond in the same container. They are all regarded as the same. They are neglected, dust settles on them, and they cannot be appreciated any more. But if, on the other hand, they are cleaned and the diamond is placed on a gold stand with light shining on it, then you will, of course, be able to appreciate it. You will see very clearly that it is a diamond, and that it is not an ordinary crystal. That degree of understanding and recognition is very important.
Now, in sincerely practicing and studying the Dharma, whatever particular line of study and practice one is pursuing, it is important to retain respect for other schools and religions. An example can be found in Buddhism itself, where people try to discriminate between Hinayana and Mahayana. That is very much against the Dharma, an entirely wrong view. One must have equal respect for the Hinayana as well as the Mahayana teachings.
Also respect is necessary for the established religions, the religions that have been prevalent in this country for hundreds of years. These religions have played a significant part in the lives of many people. If one is going to practice some other religion, it must not be in denial of any existing religion. One has freedom of religion, freedom of practice, and so one chooses to practice a particular faith. But that choice must not include rejection, denial, or a sectarian disrespect for other paths. That is not in accordance with the practice of the Dharma.
Since an individual does have the freedom to choose, however, it is important to commit oneself to a particular spiritual practice and teacher, taking advantage of the teachings and practices that one receives, and being oneself worthy of the teachings through one's continuous practice. There is even the possibility of certain students becoming more realized than their teachers. This can happen. So one can see possibilities and take advantage of them, knowing that one has the ability to actually master the teaching.
Having a very definite relationship with the teacher, the teaching, and putting effort into studying is essential if one wishes any attainment. It is not done out of or sectarianism. If one goes to a teacher and tries to study and practice a little bit, then goes to another and does the same, one would not experience definite improvement and success. So from that perspective, consistently relating to a particular line of practice and teachings is vital.


Densal: Do you see this training in an established religion as a good ground foundation for one who then chooses the Buddhist path?
H.H.: There is a general benefit in all practices as long as they have a religious or spiritual orientation, and as long as the tenets of the tradition are followed. It is in some way or another beneficial. But it is a different path.
Again, respecting all schools of teachings is important, be it Buddhism or any other. At the same time, you are free to choose. Choose a path with a meaningful experience in mind. For instance, if something tastes sour, you will want to taste its sourness, or if it is sweet or bitter you also will want to be able to taste it. Whatever practice you do, do it to the point of being able to experience its essence. Experiencing is very important. To develop a capacity for experiencing, a relationship with a particular teaching and practice is needed.
One of the reasons all the criticism comes from people is because they are so impatient and so confused that they go here and there, they try to relate to a teaching, get infatuated and try to do something fast, and they don't get anything reasonably understood, achieve any experience. How could they? How could they experience anything? So they go to another place and spend some time, a short time, and expect that something will happen immediately. If it is a true path and a true teaching, it doesn't come about just like that. It takes time.
The Mahayana teaching, for instance, is very precious. It takes a lot of output in the way of your sincerity, your commitment, and your genuineness. It doesn't happen instantly. It is not that cheap. So then what happens is you start criticizing this or that particular school, saying that it is not worthwhile, the practice is not good, or the teaching is not good, or whatever. You don't have any ground for criticism. And besides, having such an unhealthy attitude does one no good at all. Such an unhealthy attitude expressed in the open causes a lot of harm not only to yourself but to others. You place obstacles in the path of others who are making attempts to connect themselves to some kind of higher teaching. So that becomes a problem.
Along with devotion to your study and practice, and sincere openness toward others, there is another attitude to maintain: simply, no political games. That is, not going about with a friendly appearance on the outside, while inside it is something else again. Outwardly and inwardly you must lend whatever friendship and support to others that you are able, and at the same time do your practice. Respect and live your own life, and do your own practice properly. You have to have some confidence and trust in the teaching and a sense of commitment, which is like surrendering to the teaching. That is absolutely important. It does not come in an easy way.
So what I would like to get across is this: people enter the Mahayana path and expect some instant realization, without having any confidence in the teaching, any respect toward the teaching, or any genuine commitment, and they are under a serious misunderstanding. If your commitment is sincere, and you have genuine confidence and trust, then something can take place in the way of experience. The validity of the teaching is witnessed by thousands of years of practice and continuity.
If you cannot have trust and confidence, then I can frankly say that you are fooling yourself. You must have a certain amount of patience, a certain amount of confidence. The greater the confidence and trust that you are able to have, the quicker your realization, and within one lifetime you could make a significant achievement and experience a satisfactory realization. It is something that you could feel was worth all the time and energy that you put into it. If you are able to have total trust and confidence and exert yourself, then definitely within one lifetime you could have an extraordinarily meaningful accomplishment. If you are not able to have total trust, but have some, and also do some practice, even then you can achieve something.
You have heard as well as seen monks who have done some practices, but in their daily life you do not see anything different about them, until they die and sit in meditation for three days after death. If not during one's lifetime, there is a point during the bardo when there occurs a very pure moment. When that very complete, clear moment arises, the ability you have developed may bring the realization of a higher state of mind.
The third thing that I would like to say is that people have to definitely work and support themselves. When you have the enlightened attitude you have a responsibility to the people around you, to your country. You care about them. You are always with your practice, you are inseparable from it, you seize opportunities to benefit others and you will benefit others in whatever way you can. You have been in this country, you were born in this country. Many people who will read this are from families that have been here for generations. This country has been an important place for you. You have to offer respect for your grandparents, and you must live a decent life, a dignified life that upholds the traditions of your ancestors, that meets the approval of society, your parents, and yourself.
Also you have to set a decent and dignified example for generations to come. If you are really going to serve this country and help its people, this seems like a reasonable way, rather than belonging to this party and that party, and getting involved in this competition and that competition, and all kinds of politics. As practitioners of the Dharma we don't have to deny politics and reject politics, but we don't have to play those games, either. It is not necessary, it is not important, it is not needed.
If you are working maybe in a hospital, you can see how you might have the opportunity and responsibility to help people. In the same way, whatever work you have taken, there are definitely people that you can benefit. So you should serve your people, serve your country, not expecting your country to serve you. And that's part of the practice of the Dharma. Not working is not taking responsibility. If you are a practitioner of the Dharma practicing the Mahayana teachings, that means you have something to be proud of, something to be worthy of, something to be decent about.
But many people go around like some kind of outcast. That is not in accordance with the teaching, to come on like some kind of outcast, in rags, with long hair, unwashed, as if you are a drug addict or something. This is not the proper way to present yourself. You are not maintaining self respect, you are not respecting the Dharma that you are practicing, and you are not creating the proper outlook that the excellent Dharma is worthy of.
This is the message to the practitioners of the Dharma: that they must be dignified internally as well as externally, and that their internal dignity must reflect outwardly also. We are not drug addicts. Wearing decent clothes, and being a decent human being, and serving your country, your people, serving the Dharma, and also yourself, being a self-respecting person is the Dharma path. How are you to benefit beings by looking as if you are completely discarded from the society? By putting forth that appearance, you are not being responsible or reflecting the enlightened attitude.
If you are practicing the enlightened attitude, you should naturally be able to attract people so that seeing you, people might think, "Yes, these people definitely seem to be decent people, I think I could relate to them, and could ask something of these people. They might even be able to help me." So that you appear capable of giving help, or at least capable of giving some direction toward help. We are proud of ourselves as examples of the Dharma. If you are going around in rags, not taking care of your body, and going in the world like a misfit, it makes a very bad impression of you, of the Dharma center that you are involved with, and also of you as a person of this country, which means that you bring disrespect and a bad impression to this country and its people.
These are certain points that before I leave I would like to offer for people to use. I hope that whoever hears these words, whether you are a Dharma practitioner or not, or involved in Buddhism or not, that it will make some sense to you. It comes sincerely and truly, not with any put-on, or masquerade or diplomacy, but truly straight and clean. With integrity and sincerity you can serve beings, and as you work in the Dharma you will serve many beings, and that is the greatness of the Mahayana teaching and practice. You don't have to be a dropout from the community, the society or the family. You are not. You have dignity.

*********************

"One way to understand moods is that they are just the habit patterns of our mind continually playing themselves out in different scenarios. According to the view of dharma, we don't need to understand them or consider them so much as we need to replace them with new habits. This is where practice comes in. When we practice, we involve our body, emotions and mind in a new habit which is much more "real" in the sense that it is in line with the reality of enlightenment. This is the definition of purification.
The problem with paying much attention at all to emotional states is that we must on some level believe that they are real if we are considering them at all. We reinforce our sense of egoic reality by examing and exploring them, much in the way Narcissis was enamored of his reflection. When we come to practice from that place, we create extra obstacles and encounter even greater resistance."
-- A'dzom Rinpoche

*********************

Opening Your Heart, Caring for Others
An Interview with Geshe Michael Roach
By Lisa F. Weber
On Thursday, March 25, 2004, Geshe Michael Roach appeared at Vitality Natural Foods and Yoga in Wilmette for a book signing of his recently published The Tibetan Book of Yoga . At the signing Geshe Michael demonstrated "heart yoga," the Tibetan style of yoga that he is presenting to the U.S. on his book tour. Before the demonstration, I had the pleasure of interviewing Geshe Michael about himself, his new book and heart yoga.


Geshe Michael is a fully ordained Buddhist monk. He received his undergraduate degree from Princeton University and his "Geshe" title (master of Buddhism) from Sera Mey Monastery [a Tibetan monastery transplanted to India] after 22 years of study both in India and the U.S. with Lama Khen Rinposhe Geshe Lobsang Tharchin. Geshe Michael has been teaching Buddhism since 1981 and is a scholar of Sanskrit, Tibetan and Russian. Over the last 20 years, he has incorporated the spiritual principles of Tibetan Buddhism into numerous projects. He has directed a successful diamond company in New York, translated and published numerous works, started the Diamond Mountain University and Retreat Center in rural Arizona and the Enlightened Business Institute, and much more.
LFW: What was your motivation for becoming a Buddhist monk?
GMR: Two things--my mother was dying, and I was having girlfriend problems. My motivation was suffering. My parents were alcoholics and went through a bad divorce. At the time my mom was dying, I was signed up for Christian seminary. I was having trouble understanding why two people who loved each other could turn on each other and why someone who I loved so much could die. I needed clarification. [Then] I received a scholarship to study where His Holiness the Dalai Lama lives [in Dharamsala, India, seat of the Tibetan government in exile], and I studied there for the next 22 years. I didn't reject Christianity. In fact, I still consider myself a Christian. I needed to know what the meaning of life is if everyone has to get old and die.
LFW: Many of us are experiencing this same suffering. What can you tell someone who has experienced alcoholism, death of a loved one and relationship problems that might give them tools for moving through pain and suffering?
GMR: The big thing I learned in Buddhism is that whatever we perceive in the world is a result of how we have treated other people. If you maintain the first ethical code [of the Yoga Sutras ]--the first yama [ ahimsa , nonviolence]--your world will change. The first yama says that if you take care of others, your world will change and violence in your world will disappear. Alcoholism is a type of violence; bad relationships are a type of violence. The Yoga Sutras , which is the mother of all yoga, says that if you take care of others, your world will change.
LFW: Women do that to a fault sometimes. What about codependency and boundaries in response to taking care of others? How can you have a loving and compassionate heart, but still detach?
GMR: Simply put, you cannot take care of others without taking care of yourself. A big part of yoga is taking care of your own mind and body so that you can take care of others. If you sacrifice your own health to serve others, that would be a failure.
LFW: In your book, The Tibetan Book of Yoga , the heart yoga style focuses on a different way of doing pranayama. A breath cycle begins with the exhalation, followed by the inhalation. The book says that when we are born we inhale, and when we die we exhale. By reversing this flow of breath we are putting death aside. Help me understand this concept.
GMR: In the Tibetan monastery you are not allowed to do yoga for 25 years. You first study the ethical code of life. When I told the Tibetan lamas that 20 million Americans are doing yoga they were shocked. On the other hand, they thought it was a great karmic ripening. Really, it is pranayama linked to asana linked to meditation. In the monastery, yoga in the long term is used to overcome death itself. The purpose of exhaling first and then inhaling is to open the central channel. That is to change the inner body, the chakra system. To change the body is to change the heart. When the heart chakra opens you become more of a being of light. When you exhale first and then inhale, it has an effect of concentrating prana in the central channel. We call it a harbinger. You still would inhale on the way up and exhale on the way down. In the asana practice, you begin with a movement that has you exhale first. It is really just a different way in which you count the breaths. The Tibetan lamas believe this is a huge spiritual ripening. The ultimate purpose in yoga is to help many people at one time.
LFW: In heart yoga you are paying attention to the flow of your thoughts and sending your loved ones thoughts of love, compassion and patience. The end result is a calming feeling and a compassionate heart.
GMR: Yes! And hopefully it will open it.
LFW: In this practice you visualize a rose at your heart with a diamond in the center. You are supposed to inhale your loved one's problems into the diamond and explode it, and at that moment their problems are destroyed. This scares me because if someone isn't good at this technique they could in fact hurt themselves energetically.
GMR: In the monastery they are very, very strict that you don't hold their suffering in for very long at all. You must destroy every last atom of their suffering in a heartbeat. Yes, there should be a moment of hesitation or fear when you are about to inhale the other person's problems. [Then] you must explode the diamond in a flash or it could hurt you!
LFW: In your book you mention Jivamutki. Have you worked with David Life and Sharon Gannon?
GMR: Christie [McNally, his spiritual partner] and I were trained in Tibetan yoga style together. We wanted to do a three-year silent retreat. We did it three years ago. Every four or five months during our three-year silent retreat in the mountainous desert of Arizona, David Life and Sharon Gannon would fly out to teach us. They were very kind to us and the most influential teachers in our yoga practice. They are wonderful people and very dear teachers. During this time we learned that Tibetan yoga came from the same lineage as ashtanga, Iyengar, Bikram, anusura, etc. As Sanskrit and Tibetan scholars, Christie and I discovered that 1,000 years ago Tibetan yoga broke off from India because India was attacked by the Muslims.
LFW: Are you a vegetarian? And what can you say about eating meat and karma?
GMR: Yes, [Christie and I] have been vegetarians for a long time. Tibetan lamas eat meat mostly because of where they live. They live so far above tree level. Considering karma, you can experience the karma of the food. When consuming meat, which is gotten through a violent means, your mind will become more violent. You will have more angry thoughts. You have to start from the source--from something as simple as what you eat and put into your body. The main point of the book is that if you practice yoga but are not kind to others, the yoga won't work. If you are kind to other people, that is what you will see in your world. If you harm other people, that is what you will see in your world.
LFW: The alcoholic drinks because he or she is in pain. You are suggesting that this practice is an alternative to drugs and alcohol.
GMR: In the Tibetan monastery you take a vow not to drink alcohol. In [an] ancient [Buddhist] text, 3,000 years old, they say no fermented or chemical intoxicants. I brought my mom when she was dying to see the Dalai Lama who said that if you want to get the same results as intoxicants, practice yoga, practice meditation, study ethics and take care of other people. That is the story of my life and it works. The results will be the same and they will last and they are free.
LFW: Tell me about the Enlightened Business Institute, which you started.
GMR: First, I want to tell you about The Yoga Studies Institute. We come into a yoga studio for two weeks and do an intensive study of the Yoga Sutras . The first one we did was at OM Yoga in New York. We had 100 yoga teachers from all yoga traditions. We trained them really hard on the Yoga Sutras . It was intense and it was beautiful. The next one we are doing is at Jivamutki. We hope to do one here in Chicago. The idea is to bring people the deeper meaning. When we split off one thousand years ago, the Tibetans took the internal stuff and the Indians took the external stuff, and now we are trying to bring it back together, and the planet is ripe [for this]!
EBI is a separate project. I wanted to raise money for the Tibetan refugees. I started this diamond company in New York. It was the fastest growing company in New York, and I based it only on spiritual principles. Through the Enlightened Business Institute we teach you how to conduct your business only through spiritual principles. Yoga or business, it's the same thing. Your yoga only works on your body if you take care of other people, and your business only works if you take care of other people. It is not that you need to make little money and be a failure. By conducting your business through spiritual principles, you become a super business person!
LFW: It sounds like there would be a huge audience for this.
GMR: Yes, huge. In fact, my book The Diamond Cutter , which talks about this, is on the bestseller lists in Japan and Taiwan.
GMR's c l osing comments:
We all have desires in our life. On a very primitive level we all want to eat, to have sex, we want to be successful, we want others to give us attention. On the very deepest level, we want to take care of other people. That is what yoga does. It allows you to connect with that urge. It is what we were put here to do. We don't want to be selfish cruds. We want to take care of other people.
The other thing is to live more simply. You would be happier. The more things you own is harmful. You have less energy to create and to take care of other people. Christie and I live in a hut!

You can receive the full interview on CD free by going to www.lisayoga.com . (There will be a shipping charge, and you will have the option of making a donation to Yoga for Hope.) To learn more about Geshe Michael Roach, visit www.world-view.org .
Lisa Weber is a yoga instructor and free-lance writer. She may be e-mailed at lisayoga@comcast.net.

*********************

Ordination is neither weird nor extreme

Gen Kelsang Vajra - facing a meaningful challenge
Dianne - asking the questions
Dianne Elliott caught up with Gen Kelsang Vajra soon after he was ordained as a Buddhist Monk.
Gen Kelsang Vajra is now the National Spiritual Director of Kadampa Buddhist Centres in New Zealand.

Dianne: You have been ordained for a few weeks now. How is it? Does it meet your expectations?
Vajra: It is still very early but I am starting to realise some benefits, especially in my meditation practice. It seems to be moving into different realms - expanding.
I didn't really have any expectations but the actual experience of being ordained surpasses anything I could have hoped for.
Dianne: How do you feel now about your decision to ordain?
Vajra: I am absolutely sure it was the correct decision for me. I can now settle into my practice and into being a monk - and that feels right to me.
Dianne: What was the ordination ceremony like?
Vajra: There were 26 of us who ordained together, and the ceremony was held in the Temple at Manjushri Centre. Some of the senior teachers also attended, as did some monks and nuns who wanted to renew or reinforce their vows.
The ceremony itself was very simple. Mostly it was a teaching from Geshe-la on the benefits of becoming ordained and advice on how to stay ordained. He spent a lot of time explaining the boundaries of ordination and how to stay within them in our daily life.
It was really lovely being so close to Geshe-la. He blessed us and touched our crowns. It was very personal.
Dianne: Did Geshe-la give any other advice?
Vajra: He emphasised the importance of maintaining pure moral discipline and of developing and maintaining a mind of renunciation.
He talked a lot about developing renunciation and what it is. What we are renouncing is samsaric rebirth - not our family, our friends, our work, or our life. We are renouncing suffering.
He helped us understand that the mind of renunciation is a beautiful and happy mind. We sometimes think we have to abandon our friends, family, and possessions but actually all we need to abandon is our attachment to them. The suffering we experience comes from our attachment.
When you develop a mind of renunciation you don't become like a zombie, on the contrary, your mind becomes lighter and happier.
Dianne: How does it feel to be wearing robes? How do people react?
Vajra: This is an especially pertinent question for me because I work in London and go to work in my robes. I walk down busy city streets and ride the Tube dressed as a monk.
Overall the response has been very positive. I have had no negative responses at all. At times people stare but not in a negative way. Sometimes people give me things. Many people seem to have immediate faith and respect for monks - whatever religion they belong to.
London is a very cosmopolitan city and there is a real mix of races living and working here. The most positive responses tend to come from people of eastern cultures. They are very open and seem to immediately recognise and understand what I am about.
Dianne: How was the Summer Festival for you? Did it seem more meaningful as a monk?
Vajra: It was very special - being a monk and being close to my Guru physically, mentally, and spiritually. But I don't think I can say it was more meaningful just because I was a monk.
The Summer Festival is an extraordinary phenomenon that is helpful to anyone who attends - but always in a very individual way.
Dianne: When we talked before you mentioned that the hardest part of being ordained would be doing battle with your delusions especially attachment. Have you noticed any increased minds of attachment or other delusions coming up since the ordination and if so how are you handling them?
Vajra: No I haven't but it is still very early. Right now when thoughts arise that could lead to attachment I seem to be able to get them out of the way quickly. But then I am still fresh from the Summer Festival and I have Geshe-la's blessings fresh in my mind. We'll have to see what happens as time goes on.
Dianne: Is there anything else you would like others to know about ordination?
Vajra: Yes, I would like people to know that ordination is something that can be achieved by people living in the west. It is both possible and necessary.
Monks and nuns have a very special contribution to make to our society. Ordination is not something to fear. It is neither weird nor extreme - simply another life path that some people choose. But it isn't easy. Ordination presents a meaningful challenge to anyone who decides to choose it as a way of life.

*********************


Passing Through Death
By Yvonne Zhang, Shanghai Star.

Be alert and behave respectfully, it is time for underworld souls to stay with you.
WHEN ghosts appear, it's like someone blowing cold air on the back of your neck, your hair stands on end and you breathe out cold vapour. So said the boy who could see "dead people" in the film, "The Sixth Sense".
Stay quiet for a while and see if you feel anything the boy said, because ghosts are supposed to be wandering about.
According to legends and Chinese folklore, during the seventh lunar month, the gate of hell is wide open and all the spirits are let out to stay with their living families until the end of the month when they have to return to hell.
There is a festival called Zhongyuan Jie or Yulan Jie, on the 15th day of the seventh lunar month - that is Friday, August 23. Local families used to place offerings for their ancestors - usually food, incense, candles and joss money - to be spent in the underworld.
The festival is recognized in both Taoism and Buddhism, the two dominant religions in China. In both, this day is a time when the souls of the dead are redeemed.
Yulan Festival
The word Yulan means "liberation from suffering" in the original Sanskrit (ullambana), and Yulan Jie has its roots in the Buddhist story of Mu Lian, a disciple of Buddha, who saved his own mother.
It is said that Mu Lian (Maudgalyayana in Sanskrit), who gained magical powers on reaching a high level in knowledge of Buddhist doctrines, saw his mother starving in hell, with a huge belly and very small mouth, unable to swallow any food.
His mother had been a miserly woman and had not done a single charitable deed in her lifetime. She was condemned as a hungry ghost in the lowest depths of hell. Mu immediately fed his mother some rice, but the rice was reduced to ashes when it touched her mouth.
Mu begged Buddha for help, but Buddha said the mother had committed too many sins in her lifetime. The Buddha advised Mu to collect various types of fruit and vegetables in Yulan basins as an offering to the Buddhist monks. Only by doing this could his mother's suffering be alleviated.
Future generations continue to put fruit and vegetables in Yulan basins as offerings to monks, and the festival turned into a time for remembrance of ancestors.
In many places of South China and Japan, people used to make paper boats on each of which a lit candle is placed. They would put these little lanterns into the river and let them float down the water. Children would try to follow the paths of their lanterns from the riverbank to see whose lantern could go the farthest.
One folk story said this custom is to send the lanterns as a guide for the spirits of their loved ones, to show them the road back home. Another tale said these lanterns are to send away the spirits of those who died of drowning. It is believed that these spirits, "drowned ghosts", will suffer in the water until someone else comes to take their place.
It is thought that many people have died by drowning simply because they were pulled into the water by ghosts who were eager to find a substitute for their suffering.
After Death Myth
Chinese people have had very practical attitude toward religion and faith. Religions, either initiated in China like Taoism or through other countries like Buddhism and Christianity, are perceived in a totally pragmatic way.
People build temples and worship whatever god they believe will be efficacious and will respond to prayers. They do not care much about what religion the god represents.
In folklore and general belief, all the gods are like officials in charge of different affairs, judging people from different aspects and giving out punishments and rewards.
In Buddhist books, there are vivid descriptions about the horrible hell, the torment that sinners would suffer after death. Those with the worst sins would be sent to the 18th floor of hell, which means they could never be born as a human again.
And those who died of persecution, or after being wronged, would seek revenge in the living world. They would become malicious ghosts, who would bother the living and bring bad luck to people and places affected by their hatred.
Many scary movies have been made about ghosts. Classical literature also has stories about ghosts seeking justice through super-natural means.
Dou E, a character in a drama by Guan Hanqing (circa 1210-1300) was executed on false accusations and she made some severe curses before her death: it would snow in summer and there would be drought for long years.
All her curses were realized and it was not until her father came back to the region as a high official and redressed her case under the guidance of her spirit, did the natural disasters stop.
Horrifying Images
Chinese ghosts don't always take on real forms. Often they come and leave in a breeze. But sometimes they come in horrifying images of their death: hanged ghosts have protruding tongues, drowned ghosts leave water wherever they stay, and young beautiful women make the most scary ghosts, with long hair covering their pale faces with blood trickling down.
But ghosts are not invincible. There is a figure in Chinese legends whose sole mission is to catch and devour ghosts.
The character, Zhong Kui, is often pictured as being tough and dark, with a thick beard. His portrait is often hung in the house to protect the family from being hurt by wandering ghosts.
Not all Chinese communities take the festival seriously. In a modern metropolis like Shanghai, people cannot expect to see large-scale rituals of worshipping or temple operas held on the day.
But if you look carefully, you will find in some unconspicuous corners in residential quarters, candles being lit and incense and joss paper being burned. It is the past ones of the family enjoying the offerings of the living.

*********************

Perspective
Buddhism Meets Western Science

A dialogue on the mind and consciousness
by Gay Watson


The field of cognitive science has fundamentally changed in recent decades. As Western research-ers grapple with problems of consciousness and subjective experience, an interesting dialogue opens with Buddhist tradition, which has always been overwhelmingly concerned with human experience.
While the pan-Indian doctrine of karma refers to an inevitable causal chain of actions, the great innovation of Buddhism was to ethicize and psychologize this by emphasizing intent rather than action. Buddhism has always presented an empirical psychology that both reveals and rests upon a philosophy of process. The Buddha's analysis of the human condition was that it is unsatisfactory; that craving and misunderstanding cause this suffering; that it can be ended; and that a path of emotional and cognitive realignment leads to such liberation. These truths are less a statement of fact than a call to act. We are called to understand fully that ordinary life is suffering; cease from the causes of suffering; realize liberation; and cultivate the path. Throughout, the emphasis is on human experience, and the path to liberation from suffering is seen as attainable only by understanding experience and the ways to improve it. Buddhism does not propose beliefs of the supernatural or transcendent, but offers a practice: the cultivation of cognitive, emotional, and physical practices to bring about change.
Three realizations characterize the current generation of cognitive science: the fundamental and inescapable importance of embodiment, the importance of emotion, and the extent and importance of nonconscious processes; all must be seen as contextualized and interdependent.
Western science now acknowledges that the mind is not just a program in the brain, but that its processes are distributed throughout the body. This acknowledgment ends centuries of mind/body splitting in Western discourse. More importantly, it ends overvaluation of mind at the expense of body. Indeed, in a world theoretically ever more relative and pluralist, mindful realization of our common human embodiment may provide us with a much-needed foundation.
In Buddhism the separation of body and mind has never been as thorough as in the West. In a work that brings together philosophy, ritual, and medical practice, a thirteenth century Tibetan writer proclaimed that even if the mind is understood with the greatest wisdom one will never be fully enlightened until one understands the body. In an interview in the Winter 2000 Tricycle, S.N. Goenka, a leading meditation teacher, refers to the Buddha as a superscientist, stating, "If proper attention is not given to the sensations, then we are not going to the deepest levels of the mind. The deepest level of the mind, according to Buddha, is constantly in contact with bodily sensations. And you find this by experience." By attending to sensations, one trains the mind to resist reacting nonconsciously to them.
Western science now also finds conscious reason to be the tip of the iceberg of complex processes that are largely emotional and below the level of consciousness. Buddhism has always been concerned with feelings, emotions, sensations, and cognition. The Buddha points both to cognitive and emotional causes of suffering. The emotional cause is desire and its negative opposite, aversion. The cognitive cause is ignorance of the way things truly occur, or of three marks of existence: that all things are unsatisfactory, impermanent, and without essential self. The Samyutta Nikaya, a canonical work of Buddhism, describes feelings, referring to a twofold division into those of body and of mind and continuing on to a division into 108 types. The teachings place equal importance on body, speech, and mind; practices of mindfulness are related to bodily sensations, feeling, the mind, and its objects.
Though Western research finds that many of these processes function subconsciously, Buddhism has always posited sufficient freedom of consciousness to enable choice, and Buddhists try to increase conscious choice by cultivating mindfulness. Contemporary research finds the influence of mind upon body, body upon mind, and environment upon mind and body to be complicated and interdependent. Research has shown that repeated action, learning, and memory can actually change the nervous system physically, altering both synaptic strength and connections. Such changes may be brought about by cultivated change in emotion and action; they will, in turn, change subsequent experience.
Indeed, Buddhism's centuries of exploration of subjective mind states may be a resource for Western science. As scientists discover the impossibility and the inadequacy of objectivity, and the need for new methods to study subjective experience, they may turn toward this treasury of mental practices for help. A recent issue of the Journal for Consciousness Studies explores this. Perhaps this very project within contemporary cognitive and neuroscience will unite inner and outer, body and mind, first-person and third-person descriptions, and even Eastern and Western traditions.
Buddhism, based as it is upon experience and a psychological understanding of body and mind, is one of the oldest systems of thought yet most in tune with contemporary neuroscience and with other strands of contemporary discourse. The third mark of existence, that of nonself, describes dependent origination, the central philosophy of Buddhism which states that all phenomena, including persons, are constructed and dependent upon a network of causes and conditions.
The other face of dependent origination is the doctrine of emptiness. All phenomena, including persons, are empty of any unchanging, isolated essence because of this very dependence upon a network of causes and conditions from which they cannot be separated. Such theory resonates both with the findings of contemporary neuroscience and with the postmodern insistence upon contextuality and difference. Each person and thing is, at the same time, dependent, contingent, and determined by causes and conditions both environmental and cultural, uniquely individual and unrepeatable.
Buddhist teachings do not lead to logic of either/or; rather, they attempt to forge a middle path of nonduality. Unlike Western logic its purpose is not to support theory and abstraction, but to relate to an embodied way of being. This involves a movement away from binary dualisms toward relativity, mutuality, and commensurability. Thus in Hua Yen Buddhism, a central image is the net of Indra-a net of jewels, each jewel reflecting each other and the totality. Such a guiding metaphor replaces hierarchical symbols such as the pyramid or tree.
The related questions of self and free will are too enormous to engage here. The unexplained gap between physical phenomena and subjective experience remains both in East and West, science and philosophy. The no-self of Buddhism refers to the lack of some unchanging essential self, but does not deny a contingent transactional processual self, or one based on processes, such as thinking, feeling, and acting, rather than product. Buddhism posits sufficient free will to allow for intentional practices to augment awareness, to foster wholesome thought and action, and to defuse unhealthy reactions. Beyond this we must return again to intention-in this case the intention of dharma, which is the search for liberation. Outside this, Buddhism is little concerned about free will. For Buddhism is not concerned with ontology, or indeed with knowledge for its own sake. Such are questions that the Buddha refused to answer. What Buddhism is centrally concerned with is a path to liberation and the end of suffering through realigning our experience in accordance with the way things are, rather than the way we tend to misperceive them. It suggests that the way to change our experience is through understanding that experience. To this end it has undertaken centuries of first-person exploration of experience that may now helpfully come into dialogue with the West's superior third-person research.
Gay Watson obtained her doctorate from the University of London for work on Buddhism and psychotherapy. She is the author of The Resonance of Emptiness: A Buddhist Inspiration for a Contemporary Psychotherapy, and coeditor of The Psychology of Awakening. She teaches at Sharpham College, Devon, England.

*********************

Prodigy Online Interview

In 1996 Rama did two online interviews, one with CompuServe and one with Prodigy.

This is a transcript of an online interview with Rama on Prodigy. The interview occurred on Wednesday, February 21st, 1996. The interview was in the chat format, with Rama and the participants typing their questions and answers.

Interview copyright (c) 1996 Prodigy
Moderator: SURFING THE HIMALAYAS is a compelling novel of seeking and enlightenment, featuring two unforgettable, strong willed characters. Readers have likened it in spirit, charm and wide commercial appeal to JONATHAN LIVINGSTON SEAGULL ... "

Frederick Lenz's novel is based in part on the author's death-defying experiences snowboarding at the top of the world .... please write to Dr. Lenz now with your comments and questions about his book or about other related subjects, including how martial arts and meditation helped allow our guest to get more out of life than one could imagine.

This evening's chat session is with author Frederick Lenz. His book, SURFING THE HIMALAYAS, is described by some as Eastern philosophy taught through the parable of snowboarding. Now for those not familiar with the book, that must sound a little strange! Nevertheless, the novel is winning all kinds of praise from critics and from readers. Please submit your questions now...

We'll be starting in about 10 minutes. Please continue to submit your questions for Dr. Frederick Lenz, author of SURFING THE HIMALAYAS ...
Frederick Lenz, Welcome to "Surfing the Himalayas," a book which I wrote to inspire people to learn about snowboarding and a more ecological way of living.
Moderator: Besides being a world class snowboarder and a black belt in karate, Frederick Lenz is also an author, teacher and active partner in several advanced technology companies, and has directed the development of financial, business management and educational software products.

A Phi Beta Kappa and magna cum laude graduate of the University of Connecticut, he received a Ph.D. in English Literature from SUNY Stony Brook. He lives in NYC, London and Santa Fe ... Ladies & Gentlemen, Dr. Frederick Lenz ...

OK, Dr. Lenz... Let's get to the questions....
cemeteryguy: Have you always been an "adventurous" guy?
Frederick Lenz, Yes! I've always had an interest in the unknown and edge sports.
Cjazz: Is your book popular?
Frederick Lenz, Currently, it is tied for the number one spot in Marin County. It has sold over 200,000 copies in the U.S. in hardback and it has already been sold in 12 other countries. Does that qualify as popular??
Dave L in CT: What did you learn from writing Surfing The Himalayas?
Frederick Lenz, I learned something that I had forgotten ... that it is easier to write computer code than books. It was fun to relive some of the most exciting moments of my life, and it was very challenging to put those in a dramatic form that would be both entertaining and enlightening for the reader.
Dave L in Ct: What would you say, after all the studying and living that you've done, is the key to happiness (and I know that's a tough one to give a short answer to!)?
Frederick Lenz, The key to happiness, in my opinion, is taking the time to find out what your spirit wants you to do, and doing it. In short, following your dream, never giving up, and always believing in yourself!
kmfdm: Are there any specific yogic techniques that will insure that a person will be able to reincarnate in to a higher spiritual realm?
Frederick Lenz, Certainly there are! The primary yogic technique IS meditation. There are many levels of meditation, starting with basic concentration and progressing to the complete cessation of thought.

In my sequel to Surfing, I will be presenting a number of these different techniques. Incidentally, they can also be learned first hand from qualified Buddhist Yoga teachers.
Dave L in CT: How long did it take for you to write STH? Was it a difficult book to write?
Frederick Lenz, Since I am involved in the creation of software, and I'm also in the music business, I didn't really spend my full time writing the book, so all in all, the book took about a year and a half to write, and re- write and re-write!
cemeteryguy: What did your close friends and family think of your travels? Did any of them discourage you out of fear for your safety?
Frederick Lenz, My travels to the Far East occurred primarily during the 1960s. Naturally, I have returned many times since. Of course, there was concern from my family that I was traveling to far distant lands to accomplish snowboarding activities that no one had tried yet ... but I went anyway!
cemeteryguy: What was the weirdest thing you saw up on that mountain?
Frederick Lenz, I don't think I saw anything WEIRD in the Himalayas. What I DID see was a level of beauty that I had never witnessed before.
kmfdm: I just wanted to let you know, I found that your book is one of the few around that has taken a very complicated subject that is usually convoluted by religion and made it fun and readable! Thank you.
Frederick Lenz, I have heard many people voice a similar response, and it made the time and effort of writing the book worthwhile.
Larna7: Dear Dr. Lenz, I read in your book that at the age of 29 Master Fwap met his teacher. This seems like a special time in life and I was wondering how to approach that age for myself. I am almost 29 and my outlook on life is beginning to change significantly. I am both excited by the process and somewhat confused.
Frederick Lenz, From the Far Eastern perspective, 29 is considered a very special age. Astrologically, at that age, you experience a Saturn return. It is considered an auspicious time to reboot your life. It's a chance to have a clean start and move forward into something VERY exciting. I recommend that you do so.
cemeteryguy: Do you regret ANYTHING about the voyage?
Frederick Lenz: It has taken me many years to appreciate what the Buddhist monks taught me in the Far East, and my only regret is that I could not understand more rapidly the depths of their simplicity.
Moderator: Here's a question that may take a minute to digest! Thanks for writing in Yen ...
Yen for Zen: My experiences in several spiritual communities have led me to two seemingly contradictory conclusions.

(1) I am incapable of making spiritual progress without an advanced teacher;

(2) Students in spiritual communities invariably become obsessed with the life of the master, in effect living vicariously through the master, while simply trading old diversions (sex, drugs, etc.) for new spiritual diversions. Doesn't this foster ego reinforcement and defeat the purpose of spiritual practice?
Frederick Lenz, I have studied in many different spiritual communities also, and have witnessed experiences similar to your own. My conclusion is that it is best to study from a teacher of ANY subject, as long as you focus on the teachings and NOT on the teacher. All of the real important answers to life's questions lie within your own mind. A yoga teacher's purpose is simply to teach you techniques that allow you to explore the hidden realms of light within your own mind.

I have never been a particularly social person, and that's why I think meditation, snowboarding, scuba diving and long distance running have always appealed to me. But some people do find it beneficial to have the reinforcement of regular meetings with a spiritual group or teacher. So, I think it's a matter of personal choice.
kmfdm: Master Fwap mentions both occult powers and siddha powers, in your book. Can you explain what is the difference between them?
Frederick Lenz, Siddha powers are a type of occult power, but occult powers come in many shapes and forms. The siddha powers that you most commonly hear about are things like levitation, telepathy and astral projection. Occult powers, on the other hand, are much wider-ranging. They involve the ability to change dimensional structures and the evolutionary path of sentient beings.
Dave L in CT: How on earth did you ever come up with the concept of combining snowboarding and Buddhism?
Frederick Lenz, I didn't. I went to Nepal to snowboard. While I was there, I met a particular monk, Master Fwap, who told me that snowboarding, or any activity could be improved by the practice of meditation. Since I previously had some training in Korean martial arts, I was somewhat open to the idea, but it was only after studying Buddhist meditation and seeing that it DID radically improve my snowboarding that I saw that the two were complimentary.
kmfdm: Was there any point after your meeting with Master Fwap that you realized and understood what he had said, and went back to the Himalayas to find him to let him know?
Frederick Lenz, You have to wait for the sequel to find out!
liteone: Dr. Lenz, how did you become enlightened?
Frederick Lenz, By spending many thousands of hours sitting at the feet of Buddhist Masters, and by then practicing what I learned from them in my day to day life. In other words, it was all karma from my pasts lives.
Dave L in CT: Did you by any chance see the movie Forrest Gump? What did you think of it? I found it kind of wild that the country got so swept up by the meaning of the movie.
Frederick Lenz, I saw Forrest Gump several times. I personally thought it was Tom Hanks' greatest role and I think it was one of the most eloquent love stories of our time, set against the chaotic background of the 60's.
KingChud: Did you ever think it was all over for you?
Frederick Lenz, Yes, several times. (Mostly while snowboarding). Also, while traveling in the third world and being held at gunpoint by Chinese border guards.
kmfdm: I enjoy meditating to music a great deal, but am having difficulty finding groups and albums that are clear enough to meditate to. Can you recommend any?
Frederick Lenz, Absolutely!

This week, my new album ECOLOGIE came out. It's on Miramar Records, and I think you'll find that it is very easy to meditate to. I also personally enjoy meditating to Vangelis and Kitaro.
Moderator: Jacques Demers, the former head coach of the Montreal Canadiens, says that in SURFING THE HIMALAYAS, author Frederick Lenz: "goes on a wonderful voyage. He conquers the mountains of the inner self "
liteone: What about being psychic? How psychic are you?
Frederick Lenz, I think everyone is equally psychic, but through the practice of meditation you learn to make your thoughts quiet and become more aware of your innate psychic abilities.
MightyTaco: Dr. Lenz, what could you recommend to help me strengthen my discipline?
Frederick Lenz, I think you are probably very disciplined at being undisciplined. All that is necessary is to find something that really excites you in life and then the discipline will follow.
thevelvetfog: Most people consider martial arts to be implicitly violent ... true?
Frederick Lenz, Not true.

In my opinion, the best martial artist doesn't WIN fights, but avoids fights. Martial arts is a way of gaining basic self mastery of your mind, body and emotions. It can also be very useful in combat situations.
Dave L in CT: Were you at all surprised by the book's success? What was your first clue that it was going to be bigger than most expected?
Frederick Lenz, I had no clue at all. As a matter of fact, you could say that I was clueless! I did not at all expect the book would be embraced by so many people ... more every day. I think this simply proves that there is a tremendous interest in our American culture for the wisdom of the Far East and for more info about the world's fastest growing sport, snowboarding.
liteone: You often say you are a teacher of Buddhism, yet you consistently omit many key elements of the practice, such as the Eight Fold Path and the Four Noble Truths. Why?
Frederick Lenz, I have often discussed the Four Noble Truths and the Eight Fold Path in talks I have given about meditation. But, since I also teach Hindu, Buddhist and Taoist mediation, I have a very eclectic approach to the subject. However, I can assure you that next to my bed, there is always a copy of the Dharmapada, which I read from every night.
Moderator: Peter Max, the artist, says that SURFING THE HIMALAYAS uses, "amazingly simple and completely wise approaches to living a much better and happier life ... Great reading ... Many spiritual moments."
Punk bOy2: where do you get your ideas?
Frederick Lenz, From meditation!
Punk bOy2: do you like ozzy osbourne?
Frederick Lenz, Yes. Since I am a producer of both rock and New Age music, I generally like all categories of music. In particular, I enjoy Ozzy's stage presence.
fwaznoid: One of the aspects of your book I enjoy the most is the spirit of the interaction between "young Frederick Lenz" and Master Fwap. It seems to be a combination of silliness, affection and enthusiasm, but carries a great deal of respect and love as well. Do you think it is possible for someone to learn how to bring this quality to their own interactions with teachers and other people in their lives?
Frederick Lenz, Certainly! I think it is incumbent upon the student to go to the teacher to learn. That may sound obvious, but most people approach Far Eastern teachers and teachings with many preconceived notions. I think it is best to let the experience you have with a teacher direct itself. Instead of trying to make the experience into what you might like it to be, leave it alone! Simply allow your own natural feelings to evolve as they will. Be true to yourself.
Punk bOY2: What kind of BOOKS do you read?
Frederick Lenz, My favorite novel is Ulysses by James Joyce. I also read a great deal of Shakespeare, Thoreau, and my favorite contemporary author is John LeCarre, the author of spy novels.
Moderator: Here's a concept taken from SURFING THE HIMALAYAS that you might find interesting: "Enlightenment is the complete awareness of life without any mental modification. It is only by clearing one's mind through meditation and thinking nothing that one becomes aware of everything. One cannot truly describe "enlightenment" in a vocabulary of words, because words are fences of a sort, and thus limiting ... "

Pretty interesting talk for a Wednesday night chat ...
kmfdm: I've always wondered how, when an enlightened master is in samadhi and there is no ego left, they are able to return to this world or whatever world they may be meditating in. Is this a siddha power or something like that?
Frederick Lenz Gravity.
Dave L in CT: Which do you like better: living the experiences you write about OR the actual task of sitting down and doing the writing?
Frederick Lenz, I prefer living the experiences. For example, I was recently scuba diving at 235 feet in Saba. The experience was bright, vivid, and I didn't have to rewrite it! Even though I have a Ph.D. in English, I have never found writing easy, so I think I am prone more to adventurous sports and, while recuperating from my adventures, I return to writing to try and inspire others to push their own limits in a safe and intelligent way.
KingChud: I want to say that I loved the book and was inspired and I was wondering how and when you became a Buddhist Monk.
Frederick Lenz, I was initiated as a Buddhist monk at the age of 19, but I think that initiation is simply a starting point. Being a Buddhist monk, to me, means never losing one's optimism in spite of all difficulties. It also means being harder on yourself than any of your teachers ever were. You must constantly check and re-check that you are meditating properly, and not becoming what Clint Eastwood refers to as "a legend in your own mind."
SeraSera: What is the difference between ego and personality?
Frederick Lenz, There isn't really a difference. Personality is a manifestation of ego. Ego is the central sense of separateness that a person has from the rest of the universe. Personality is the form that that separateness takes.
Punk bOy2: Can you play any instruments?
Frederick Lenz: Yes. I can play the guitar, the dulcimer, the piano, and the drums. However, I am not a performance quality musician. Rather, I am a composer and a producer.
bhud: What was the inspiration for SURFING THE HIMALAYAS?
Frederick Lenz, The actual inspiration was Generation X. I grew up in the 1960's. While it was a very turbulent time, it was quite optimistic. Unfortunately, the world that Generation X has grown up in is turbulent, but NOT optimistic. Through writing Surfing the Himalayas I hope to excite the members of Generation X in the wonders of both snowboarding and Far Eastern thought.
Dave L in CT: Will Surfing the Himalayas ever be made into a film? Could the book's message be adequately captured in a film?
Frederick Lenz, I have had several offers to make the book into a film. I don't know if the message could be accurately transmitted, and so I have been somewhat hesitant in granting film rights. Perhaps someday ....
Yen for Zen: Buddhist masters are well known for radical teaching devices that seem irrational on the surface. Example, Tilopa and Marpa, who required disciples to endure extreme physical and verbal abuse, pay exorbitant fees, and carry out a variety of bizarre tasks.

Are the crazy trials somehow an integral part of the process? Are they tests of faith? Or is it ego shock treatment designed to turn the disciple inward to learn that the teacher's chaotic behavior is symbolic of the chaotic nature of life?
Frederick Lenz, It seems to me that you just answered your own questions very well! Have you considered a career in spiritual teaching?
Frederick Lenz, By the way, I no longer teach meditation. Only software design.
kmfdm: I notice that even though I meditate and have a lot of things to focus my mind on, I'm always thinking. It gets very frustrating. Is there anything you can recommend that I can do to reduce the static in my mind?
Frederick Lenz, Yes, definitely.

Keep trying. Your experience is quite normal for a person during their first ten years of meditation. That doesn't mean that you are not making progress just because you are thinking. Meditation is a lifelong study. You are learning to be at peace with yourself. It is not simply a discipline, but also an understanding.
FreshPowPow: In your book you talk about the fall of Atlantis, but you don't go too deeply into the cause. What happened?
Frederick Lenz, There were many stages to the Atlantian civilization. During the later stages scientists became involved with advanced particle physics. In particular they were interested in reverse gravity fields. Their experiments caused them to destabilize the structure of the continent and thus Atlantis sank beneath the waves.
bhud: What did you do after your experience meeting Master Fwap? Did you immediately become a serious yoga practitioner?
Frederick Lenz, Not immediately. I returned to the US, had variety of different experiences, and then while in the mountains of Southern California I began to practice what I had been taught.
Miami Mary: What are your feelings about recreational drugs or alcohol? If done in moderation, do you feel that they can enhance a lifestyle or do you feel that in the long run that they'll only diminish or take away from true happiness?
Frederick Lenz, My feeling is that drugs and alcohol take away from the pure experience of meditation. That does not mean that occasionally a person couldn't have a glass of wine or a drink, but if alcohol is used on a regular basis, it definitely negatively effects one's ability to meditate, and drugs REALLY mess you up.
Yen for Zen: The other day on Star Trek Voyager, a member of the Q, a nearly omnipotent species, fought for the right to commit suicide because he had seen and done everything in the universe. There was nothing left for him to learn or experience.

Does enlightenment happen only after you've seen and done everything in the universe and gotten bored with it all? (Don't worry, I'm not suicidal. Just curious about enlightenment).
Frederick Lenz, Enlightenment can occur or be brought on by a number of different circumstances. Some people seek enlightenment, as you've suggested, because they feel they've seen and done it all, and need something new. Other people seek enlightenment to overcome pain, to experience ecstasy, or to find out what the true meaning of life is.
dharmakaya: You have taught meditation in the past. I would like to know what you think of Mother Meera.
Frederick Lenz, There are many great teachers of meditation, but I think only a person who's studied WITH someone for many years is in a position to voice a true opinion about a particular teacher. Since I have not studied with Mother Meera, I am not qualified to answer your question.
Sally Seeker: A book I've been checking out about Buddhist mind training says that a method to get beyond ego is to mentally take on the sufferings of others. Did Master Fwap talk about this, Dr. Lenz?
Frederick Lenz, Certainly!

One of the traditional Tantric methods of mediation is to imagine that you are taking on the suffering of all beings. There are many such methods and techniques which can be found in abundance in books on Tibetan Tantric Buddhist Yoga.
dharmakaya: What is the best way to hold onto and strengthen a spiritual realization and integrate it into one's daily conscious awareness?
Frederick Lenz, It only takes a few minutes to read a spiritual book, but it may take many years or lifetimes to realize the truths contained in such a book. Don't lose your sense of humor, keep trying!
fwaznoid: I love your music, especially your new album Ecologie. Do you have any recommendations for how someone who is a performance musician can learn to compose music?
Frederick Lenz, As you know, the group I produce, Zazen, has three current albums out: Enlightenment, Canyons of Light, and our latest, Ecologie. My own musical background is based in the blues and in classical composition. I grew up listening to Muddy Waters, John Coltrane, Miles Davis, Beethoven and Bach. If you're really interested in composition, listen repeatedly to the music that turns you on most. Learn to anticipate every note and movement in the music before it occurs.

It probably wouldn't be a bad idea to go to Juilliard if you can afford it!
liteone: Dr. Lenz, you used to claim that there were 12 enlightened beings on earth, but recently it has changed to 5. Why?
Frederick Lenz, Downsizing!
Nightcoder: Dear Dr. Lenz, would you please tell us about the name Rama you sometimes use. I saw it on the covers of your albums (Ecologie and those by Zazen). Is there a connection with Ramayana? Is there a connection with Arthur Clarke's Rama Revealed? Thanks a lot.
Frederick Lenz, Interestingly enough, Aurthur C. Clarke once autographed a copy of his book Rendezvous with Rama for me. Rama is a fairly common name in India. It symbolizes an individual who is interested both in enlightenment and martial arts. I do not claim to have any past life connection with the historical Rama. It's just a name I liked ...
mr D Kaya: How do you try hard without being attached to results?
Frederick Lenz, Keep trying!
Zilla8: Are there certain cues that you get when your energy is focused in the right place at the right time?
Frederick Lenz, Yes! Definitely!

The main clue is that you feel better. Your mind is clear and you are optimistic even in difficult situations. As Buddha points out, however, you should not rely on the opinions of others for validation of your internal progression. That's why I don't read newspaper articles about myself.
Mr D Kaya: How can you love and be detached at the same time?
Frederick Lenz, Read The Little Prince by Saint-Exupéry.
Sally Seeker: Dr. Lenz, what dreams do you want to fulfill in this life?
Frederick Lenz, I would like to create better software for the United States Government military and private industry. My particular focus at the moment is on the development of genetic algorithms and neural networks that work together to create computer architectural systems. In addition, I would like to write a good sequel to Surfing the Himalayas, and dive to 350 feet! I like it down there ... you know who the sharks are!
Miami Mary: In your opinion, can one experience great happiness, satisfaction and pleasure in life without benefit of meditation? Or is it a pre requisite for everyone?
Frederick Lenz, I think everyone meditates in their own way. Some people sit and practice formal mediation techniques for many hours a day while others spontaneously meditate while watching a sunset, listening to music, or participating in athletics. I would not attempt to define the meditative experience. That's what YOUR life is for. In Surfing the Himalayas I have introduced Tantric Buddhist concepts that will help many of you along the pathway to enlightenment. I have also tried to encourage a style of snowboarding which is similar to surfing.

One of my greatest personal heroes is Jerry Lopez. When you can snowboard like he surfs, you'll be there!

I would like to thank you for joining our chat session tonight, and for reading my book and listening to my music. I would encourage you all to follow your dreams wherever they lead you and pay for those dreams with good jobs in software programming and computer design! May the force be with you!

*********************

Promoting Vegetarianism:
An Interview with Tenzin Kunga Luding
Phayul, December 19, 2004

"Where Tibetans go, butchers flourish," they say. But this is soon to end if efforts of a lone Tibetan youth is to bear fruit. Tenzin Kunga Luding, a Tibetan youth is on a mission to bring about vegetarianism in the Tibetan society. He agrees it is a gradual process though. But he believes every Tibetan will one day embrace vegetarianism. Phayul reporter Nilza Angmo caught up with him recently, excerpts...

Could you tell us a bit about yourself first?

I was born at the Sakya Tibetan Settlement in H.P., India, where my father, then Member of Parliament of the Tibetan government-in-exile, was also serving as the General Secretary of this settlement. He was one of the pioneers who introduced the Tibetan Green Book. Since my childhood I had always been very interested in helping the helpless and the needy like collecting funds for the Somalians affected by famine, raising funds for the Tibetans in Manali who were affected by the mob violence, etc. I have served as a General Secretary of Sakyapa Ngorgon Charitable Society and Samyeling GRS Welfare Association in Majnu-Ka-Tilla. I am happy to say that during our tenure with the Welfare association, we were able to obtain licenses for shops and restaurants in our colony, sponsor needy children, build gates for the colony, improve relations with police and concerned authorities, etc. For our hard work we received community service award from the Local Assembly and Welfare Office which was presented by Kungo Tashi Wangdi la, Representative of His Holiness the Dalai Lama in Delhi.

What prompted you to start this society?

As a child I was extremely fond of meat unlike my sister who rejected meat or its soup right from her infancy. I relished meat not knowing how it came about, till I was about 10-11 years old when someone narrated me for the first time how cattle were packed in trucks, unloaded, and mercilessly slaughtered. The suffering of these animals that were forced to be our food touched my heart right through. Since then I gave up meat. To make up for it, I used to consume lots of milk and eggs, as school books, movies, health magazines and everyone around my world believed these two things were healthy and necessary and I felt the same too. Then one day I read Maneka Gandhi's book "Heads and Tails". This book was the turning point of my life and it enlightened me about lots of ugly things that I knew for the first time. I realized that these foods (eggs, meat, and animal dairy products) were not only causing so much of unnecessary sufferings to animals but were harmful to our precious health and environment as well. I then chose to be a Vegan. At that time hardly anyone had heard of this term and almost everybody opposed or was skeptical of it. But I had the full support of my family, even though they were concerned of my health. I knew what I was doing and I stuck firm to my belief. I knew that compassion and loving-kindness form the core teachings of our 'Buddha Dharma' and so propagation of vegetarianism was even more important in our community. I waited long time for a related group to come up in our community, but nothing surfaced. Finally, I took it upon myself to get things started and thus how we have the first Tibetan vegetarian and vegan group called "Tibetans For Vegetarian Society (T4VS). It is a registered non-profit charitable trust. I consider it as our community's share of contribution for the development of a globally healthy, happy and humane environment. Everybody is welcome to join us whether vegetarian or non-vegetarian, Buddhist or non Buddhist.

This society was founded in 1997, and it was registered in 2004, why?

The following were the chief reasons for the delay: In the initial stages, I was basically working all by myself and had very few people who could understand me. It was a daunting task and there were many skepticism, pressure, and economic problems. Everywhere I turned to, there were people who ate meat and thought it was just a fad on my part. Some thought I had gone crazy or fighting a loosing battle. Since it was a new thing for our community I neither got any financial nor moral support. I literary had to dig into my own pockets or borrow money from my family to carry out activities such as to rescue animals, compile a magazine for free distribution, give an appeal in the Tibetan Review, etc. It was a gradual progress. Slowly my hard work seemed to pay off and I began to receive genuine appreciation or sympathy from people. Since this year our government was celebrating Tibetan Vegetarian Year I felt it was the perfect occasion to strengthen our group and seek legitimate registration. I invited a few like-minded people to discuss the matter and they expressed their desire to join the organization. So, on the 6th August, 2004 we got ourselves registered as a non-profit Trust.

What is your opinion present dietary habit of the Tibetan community?

I can only comment on what I have seen in the Tibetan community in India specially the ones I have been to. I am happy to observe that our food variety has improved and people are getting more health conscious. Most Tibetans have now included legumes in their regular diet which is very important. More people are turning to vegetarianism, specially the younger generation, which is wonderful. We have now started to gift fruits and juices in place of eggs and butter which was in the past a regular feature when visiting a guest. In fact I feel we must introduce fruits on a daily basis. I would love to see Soya bean dairy products replace animal dairy products. It is not only healthier, and ethical, but also an environment friendly option. On the whole I think there is some awareness to remain healthy seeped in the minds of our people and we must continue to follow in the same direction.

What kind of activities do you organize to create awareness of the benefits of a vegetarianism?

We wish to explore every possible means to reach people. So far we have shown documentaries to public and students, put up posters, given appeals in Tibetan Review, distributed pamphlets and stickers, sent video CD's to every settlement and few big monastic institutions in India. We ran a signature campaign persuading all the Tibetan restaurants and meat sellers in Majnu-Ka Tilla and Budh Vihar colony to observe a meatless day on full moon's day during the Saga Dawa month. Recently we organized a very successful concert in Majnu-Ka-Tilla as a part of celebration of the Tibetan Vegetarian Year (2004-2005) with the well-known Shambala Band, Pa-Tsering, Dhondup Tashi, etc. Phuntsok Topden, a youngster of our board member, has also started a "Green Friday" movement in his settlement. On this day many people of his settlement abstain from eating meat. As we have already registered our domain i.e. www.t4vs.com we are now on the phase of developing a web site for it. Alongside this, we are working to make a new video documentary to be shown in year 2005. We also hope to organize the first Vegetarian congress in our Tibetan society in the nearby future.

Recently you had the honour of having an audience with His Holiness the Dalai Lama, tell us about that.

Despite His Holiness' tight schedule, he was kind enough to grant us an audience for about half an hour. We all felt so blessed and privileged. We were actually there to seek his blessings and capture his message on vegetarianism in video to show to people. His Holiness inquired what let me to start this group and was very pleased to know the ethical reasons behind it. During the conversation one significant thing he sadly noted was that in Tibet these days there are many meat sellers outside the Potala Palace which were never there before. His Holiness advised us to continue the efforts undeterred and felt it would be more helpful to expand the work to various parts of the world including Tibet. And just before we were departing I mustered all my courage and requested His Holiness to be our Patron-In-Chief which he has happily obliged. This is a land mark in the Tibetan history to have a Dalai Lama as Patron-In-Chief of a registered vegetarian and vegan organization. We will be showing the video recording of this message as planned in 2005 but now one can view it in Bod Gyalo's monthly video news too. In all, the audience has motivated me and the board members to work harder and consolidated our belief. (We wish to inform that majority of the financial assistance has been from the Private office of His Holiness the Dalai Lama.)

A few words to the Tibetan community?

I would like our people to read the book, "The Food Revolution" by John Robbins, which will give an insight into the need for plant based diet and my belief. One can also visit some good websites like, www.vegsource.com, www.ivu.org, and www.peta.org. What I am doing is nothing new; it has always been there in our Buddhist scriptures. I am only trying to follow these teachings by propagating love and compassion with respect for nature. His Holiness the Dalai Lama, also stresses often the need to cultivate a good human heart for the benefit of all living beings. Therefore it would be nice to see people trying to give up or reduce meat for a week, month, or for whatever limit of time they could. Meat is no more a matter of survival for us. With exposure to large varieties of nutritious vegetarian food, improved technology, better transportation system, and advancements of knowledge in various fields it is so much easier for us to give up meat than our predecessors.

Lastly, I conclude with the favorite lines of His Holiness written by Shanti Deva which has always motivated me. I hope it does the same to you all.

As long as space remains,
As long as sentient beings remain
Until then, may I too remain
And dispel the miseries of the world.